You are on page 1of 257

1- What is the percentage of adult Saudi male smoker:

A. 15%
B. 20%
C. 30 %
D. 51%
Answer: B vs C

4- male with gonorrhea, his wife present with discharge and itch what is the most accurate
dx swab :
A. Endocerix swab
B. High Vaginal swab
C. low vaginal swab
Answer: A

5- 31 yrs old 10 months history of diarrhea with anemia and unintentional wt loss no change
appetite no blood in stool What to give:
A. anti diarrhea
B. antibiotic
C. anti parasite
D. gluten free diet
Answer: C vs D

6- Cause of Polyhydramnios:
A. Down syndrome
B. bilateral renal agensis
C. IUGR
Answer: A

7- Time of ovulation:
A. 36 hours after LH surge
B. 36 hours before LH surge
C. At time of LH surge
D. something related to estradiol
Answer: A

8-10 year old boy presented to the pediatric clinic with a history of skin rash. The parents
report that the symptoms had started with a single red spot over a small scratch and have
spread over the last 4 days . There are several lesions on the cheeks. The trunk and the
upper limbs. Physical examination confirms swollen lymph nodes in the arm pits and groins
Blood pressure 100/65 mmHg HR 80/min
Respiratory rate 24/min Temp 37.4 c
Which of the following is the most appropriate treatment?
A. steroid
B. acyclovir
C. antibiotic
D. antiseptic
Answer: C

1
9-Same Q, Dx?
A. Impetigo
B. HSV
(cat scratch disease not mentioned)

10- heart rate 130, pink body and hands with cyanotic feet, weak cry, flexion of the arms
and legs, active movement and crying when stimulated. What is your patient’s APGAR
score?
A. 9
B. 10
C. 8
D. 5
Answer:

12-A 39-year-old male presents to the clinic with a 3- weeks history of painless neck
mass.on exam, the mass is located at level 2 ,not tender, measure 3*5 cm and hard. What
is the best next step in management?
A. CT with contrast
B. US
Answer: Lab test? if not in the option then most likely B.

13- Which of the following is the highest risk factor for cerebral palsy:
A- Hypoxia
B- Preterm
C- Gestational diabetes D- Neonatal sepsis
Answer: B

14- Pt came with perianal mass developed after hard stool passage 12 hours ago, mass
was bluish, tender under skin of perianal area, about 1*1 cm, what is the appropriate
management ?
A-Needle aspiration B-Conservative
C-Hemorroidectomy
D-Rubber band ligation
Answer: B

15- Treatment of chrons controlled symptoms on steroid and another drug , came with
multi-fistula ?
What to give ? No abx in options A-Infliximab
B-Azathropine C-Methotrexa D-Budesunide
Answer: A

17- UTI case, what confirms it:


A- Nitrate
B- Leukocyte
C- Specific gravity
Answer: A

18- Know case of asthma use inhaler steroids come with oral thrush what's treatment:
A- Nystatin
B- Amphotericin b

2
Answer: A

19- Female with vaginal discharge and in examination there is red strawberry cervix (I think
there is cervical motion tenderness!)
A- chlamydia
B- neisseria gonorrhoeae
C- trichomoniasis vaginalis
Answer: C

20- child with pica,pale, low socioeconomic state , socioelow hemoglobin hand lead was 2
(that the unit of lead was mol/l)threm, iron profile was normal, x- ray dense metaphyseal
bands, (they mentioned splenomegaly in examination the Q but I forget if it is positive or
negative finding), what is the treatment?
A- vit. K
B- D-penicillamine
C- Iron
Answer: B

21.Elderly on amitryptilin (30mg) at night for insomnia C/o drowsiness,


management?
A. Switch to SSRI
B. Change to taking at morning
C. Change to taking TID
D. Decrease to 10mg
Answer: C

22.Child come with empty lt testis and there is swelling in lt groin exam was
testis in groin and can return back to scrotum by hand rt testis and
srcrotum normal what the diagnosis
A. undescend testis
B. retractil testis
C. orchitis
D. Epididymitis
Answer: B

23. Child just delivered was die no resuscitation done for him when exam
him was ecchymosis and ct showed intracranial hemorrhage plt for him
very low 5 And mother healthy and her platelet normal what the
diagnosis
A.alloimmune thrombocytopenia
B.idiopathic thrombocytopenic
C.TTP
D. syndrome i forgot the name
Answer: A

24.35 man come with redness and hotness at lateral side of rt thigh What
treatment you will give (as the question prescribe cellulitis?)
A.penicillin
B.cloxacillin
C. metrandizole

3
Answer: B

25.Pt have liver cirrhosis with ascitis and present with umblical mass Hernia
reducible and uncomfortable with that what next:
A.do operation immediatly
B.do paracentesis and after that direct do operation
C.dont do operation
D. dealy untill ascitis controlled
Answer: D

26.What child can do at 4 years:


A.can count until 4 and tell story
B. can tell sentences from 2 word
Answer: A

27.Baby come at OPD and he can support his head and he can sit with
support and can reach object at any age
A.3
B.5
C.8
D.10
Answer: D

28. Elderly pt come with severe headache to ER then suddenly deteriorated and
decrease level of consciousness ct done was hematoma next:
A.admit to icu
B.give mannitol
C. evacuate in ER
Answer: B?

28 years Gestational 20 week her blood group negative and she come
with vaginal spotting what you will do
A.observation
B.give anti d now
Answer: B?

16 years old boy come to ER with history of euphoria agitation visual


hallucination, What he take:
A. cannabis
B. amphetamine
Answer: B

32.Female come complain of abdominal pain with watery and green


vaginal discharge she recently entered IUD diagnosis:
A.PID
B.uterine rupture
C.bacterial vaginosis
Answer: A

4
33- Woman with hx of miscarriage, now in 7 weeks with vaginal spotting, no hx of passing
tissue , on examination os is closed and no bleeding:
A.Threatened abortion
B. Normal pregnancy
C. Incomplete abortion
D. Inevitable Abortion
Answer: A

Pregnant HBs positive what will u give the baby in first 12 hrs ?!
A. hep b vaccine + immunoglobulins
B. hep b vaccine only
C. immunoglobulins only
Answer: A

35.Woman 35 years old , smoker 10- 15 cig per day , has history of sub
fertility and has family history of diabetes, which is the strongest
factor for the fertility ?
A.Age
B.Hx of subfertility
C.Family hx of dm
D.Smoking
Answer: A

36- Newly married 22 years old comes to you for a regular follow-up for her gynecological
health. She is medically free and has regular 28 day cycle, complaining of mobile breast
lumps bilaterally that are painful right before meneses. Examination was normal. She wants
to get pregnant, but she is worried about her breast lump. What is the
screening test to be performed?
A. Bilateral mammogram
B. Papanicolaou test
C. Breast ultrasound
Answer: B

37-A male victim presented to ER after RTA, with SOB, in examination the tracheal shifted
to the right side, in chest X-ray, the lungs are expanded and winded mediastinum. What is
the diagnosis?
A. Tension pneumothorax
B. Cardiac tamponade
C. Pleural effusion
D. Ruptured esophagus

Another recall
trauma with hypotension , x-ray : trachea shifted to the right , expanded lungs and widened
mediastinum. What is the dx ?
A- Massive haemothorax
B- Pneumothorax
C- Thoracic aorta rupture
D- Spontaneous pneumothorax
Answer: C

5
-Trauma patient with trachea deviated to the left & absent Rt lung sounds. Vital signs were
provided & he had low BP. Best management?
A. Needle decompression
B. Chest tube
C. Intubation
Answer: A

38- Pregnant women during vaginal delivery, what can make her has fourth degree perineal
tear?
A. Unrestrained legs and squatting position
B. Unrestrained legs and sitting on chair
C. Restrained legs and use of forceps and other metallic instrument
Answer: C

39- Female wants to conceive after 1 year. What you will give her?
A. Influenza vaccine
B. Rubella vaccine
C. Hepatitis
D. B immunoglobulin
Answer: B

40- PROM How to confirm its amniotic fluid?


A) Sterile speculum examination
B) Abx
C) chemical testing
D) Fetal hormones
Answer: A

41.Newborn with shifting of trachea, What to do ?


A. Intubation
B. chest tube
C. thoracotomy
Answer: B

42.Obese with chronic cholilithisos?


A. mixed
Answer: A

43. Newborn infection after delivery ( group A streptococcus was found )


management ?
Answer: incomplete Qs

44.Old man with oxygen 74% what to do ?


A.mechanical ventilation
B.oxygen mask
C.send home
Answer: A

6
RTA patient came to ER and goes through exploratory laparotomy and
you find multiple laceration of liver, patient is unstable, What will you do :
A-right hepatectomy
B-prehepatic packings
C-right hepatic artery ligation
D-indvidual hepatic artery ligation
Answer: B

Infant Has Cataract jaundice he has brother who has same


presentation don’t recall the rest but it was Galactosemia presentation ,
Urine / +ve for urine-reducing substances test (normal negative “written
like this “) , How to confirm Dx?
A.HIDA SCAN
B.urine-reducing substances test
C.Coom test
D.Alpha anti
E. alpha antitrypsin test
Answer: B

Pediatric case female has sore throat , Pain upon swallowing and neck
lymphadenopathy since 2 days, Now complaining of fever, When
examine her throat swollen tonsils white exudate. What's the important
next step to establish the diagnosis :
A.ECHO
B.Biopsy the lymph node
C.Throat culture
D. Blood test looking for agent (not ASO titer)
Answer: C

Women K/C of liver cirrhosis, + acites , confused labs: low Na


A) Fursomide
B) 0.5 NS
C) NS
D) D5
Answer: D if no albumin

I’m not sure if there was hypertonic saline or not instead of 0.5 NS or NS
I forgot
*Similar MCQ

Old patient smoker 10 packs, Has respiratory symptoms, didn’t mention weight loss has
clubbing:
A-COPD
B-Lung cancer
Answer: B

After 5 years of infertility, baby was born and vomits between meals ,
everything Norma baby is healthy ?
A. Education about feeding and reassurance

7
Answer: A
Baby after feeding cyanosis ?
A- TOF
B- ASD
Answer: A

A young man fell on an outstretched hand and there was pain at the anatomic snuff box:
A-Hamate fracture
B- Scaphoid fracture
C- Colle’s fracture
Answer: B
another recall: Falling on out stretching hand name of the fracture? Colles fracture)

A full-term C-section baby presents with intercostal retractions and cyanosis. What is the
most likely diagnosis?
A- Transient Tachypnea of Newborn
B- Meconium Aspiration Syndrome
C- Hyaline Membrane Disease
D- Bronchopulmonary dysplasia
Answer: A ( if pre-term, C)

A pre-term C-section baby presents with intercostal retractions and cyanosis. What is the
most likely diagnosis?
A- Transient Tachypnea of Newborn
B- Meconium Aspiration Syndrome
C- Hyaline Membrane Disease
D- Bronchopulmonary dysplasia
Answer: C

Child had long hidtory of purulent otitis media with effusion which was treated with
tympanostomy tube came to you today with Yellowish discharge from the tube. No fever. In
history he likes to swim (WRITTEN LIKE THIS) what is the organism?
A. STREPTOCOCCUS Pneumonia
B. STAPHYLOCOCCUS AURUS
C. PSEUDOMONAS AERGONISA
D. STAPHYLOCOCCUS EPIDERMIS
Answer: C

Patient with intracranial hemorrhage. next step?


A-Mannitol
B- Diuretics
C- Raise head of the bed
Answer: C initially.

irregular cycle + absent period for 6 weeks With pregnancy test positive since 2 week only
before it was negative How to determine the pregnancy age
A- Quantitative hcg
B- Calculate from date of pregnancy test
C- Wait 2-4 WEEKs to do ultrasound
Answer: C

8
Subdural hematoma how to manage?
Answer: Evacuation

child with edema around eye dark urine and LL edema pitting dx?
*similar Q: child with HTN, ankle and around eye edema, proteinuria, what is the most
important history?
A.Recurrent UTRI
B. Abdominal trauma last week
C. Impetigo last week
Answer: A

Appropriate site for thoracentesis ?


A- Mediclavicular line 6th ics
B- Medaxillaru line 6th ics
C- Parasternal
D- 9th ICS
Answer: D

Pt had history of ectopic pregnancy 2 years back treated with


methotrexate, now present to preconception clinic , how to minimize her next
pregnancy risk ?
A-Wait for one year more
B-Folic acid
C-Early detection or diagnosis of pregnancy
Answer: C

Pt in ICU had leukocytosis and fever with long scenario, Dx ?


A-SIRS
B-sepsis
C-Severe sepsis
Answer:
It depends.

Pregnant lady in c/a and doctor couldn’t stop bleeding he calls assistant
consultant and the conclusion is to do hysterectomy to safe her life
A-Consent from husband
B-Do only what’s written in intervention consent (basically do nothing)
C-Do it without consent
D-Idk call someone or something
Answer: C

Pregnant came with ROM and was preterm (I think 32 week) has history of
active genital herpes ulcer:
A-Speculum exam
B-CS immediately
C-Assisted delivery
D-I forgot the last one, i think induction
Answer: A

9
Baby came with clear picture of UTI and urine analysis showed UTI. Mum
said she had these symptoms multiple times. What to do?
A-Renal US
B-Voiding cystourethrogram
C-Renal US and cystourethrogram
D-I think the last one was reassurance
Answer: C

17 YO female has DM type 1 for 5 years with 3 times DKA. Mum is


asking the prognosis of the disease, Which can be used for annual screening
for nephropathy?
A-Urine analysis
B-Microalbumin in urine
C-Creatinin
D-HgA1c
(These are the exact options)]
Answer: B

Female pt (i forgot what was the main presentation) has history of multiple
Stillbirth APTT was high:
A-Antiphospholidid syndrome
B-Protein C deficiency
C-Protein S deficiency
Answer: A

Scenario and at the end they gave diagnosis of pleural effusion, What will
give exudative pattern?
A- LDH more than upper 1/3 of normal
B- LDH more than upper 2/3 of normal
C- Pleural:Serum Protein <0.5
D- Pleural:Serum LDH <0.6
Answer: B (Lights criteria)

Pt with meningitis Sxs with history of fever for 1 week


CSF analysis
Color: turbid
Glucose: normal
Protein: high
Neutrophil: high but not very much
Lymphocyte: very high
Causative organism?
A-Bacterial
B-TB
C-Viral
D-Fungal
Answer: C

10
Pt with long standing HTN not controlled. Echo showed left ventricular
hypertrophy and EF was 55%, Dx?
A-Hypertrophic cardiomyopathy
B-Diastolic dysfunction
C-Systolic dysfunction
Answer: B

32 GA came with preterm labor then had preterm rupture of membrane


and was given Abx and steroids. What to give ?
A-Nifedipine
B-Mg sulfate
Answer: B? it depends

Pregnant at 8 week of GA had painful bleeding. Cervix was open and fetal
mass was seen on the cervix, US showed crown rump length of 9 weeks and
fetal heart activity, Dx?
A-Threatened abortion
B-Incomplete abortion
C-anembryonic pregnancy
Answer:

Pt with dysmenorrhea and multiple pregnancy. One of her pregnancy was


delivered by CS and all of them was induced. Examination normal uterus not
enlarged not tender and no adenexial mass
A-Endometriosis
B-Adenomyosis
C-Fibroid
D-pelvic congestion syndrome
Answer: D?

89. Pt has neck or back pain not sure and using analgesia to reduce the pain
BP was high, Pulse was normal, Na was normal, Potassium was low
What caused HTN?
A-Essential hypertension
B-NSAID-induced
C-Pheochromocytoma
D-Primary Hyperaldosteronism
Answer: D

90. Baby presented with drooling and increased salivation, X-ray showed
coiled NGT?
A- Esophageal atraisa
Answer: A

*Another previous recall:


Pediatric pt suddenly developed SOB after Insertion of coiled orogastric tube which is
shown on the xray, what will be the complication that will most likely occur?
A-Aspiration

11
B-GERD
Answer: B

91. Pt just received chemo for leukemia and developed fever and Hyperkalemia. What do
you expect to see:
A-Hypocalcemia
B-Hypercalcemia
Answer: A

92.Pt came with 1 week history of N/V, fatigue and yellow scleria. No past
medical history no recent drug used or illegal drug use. Exam showed
jaundice , LFT were sky high , Management?
A-Check INR
B-Give dextrose
C-Refer to liver transplant
D-Reassurance
Answer: A

93.Pediatric pt with abdominal pain radiating to the back increased with eating. Tenderness
on the epigastric area. Amylase and lipase high, Next step?
A-NPO
B-IVF
C-analgesia
D-MRCP
Answer: ?

94.Question about hepatological case, then ask which of these has the
highest best prognostic value?
A-Bilirubin
B-Albumin
C-ALT
D-INR
Answer: D

95.Child with nausea vomiting, fever and UTI signs gone through renal Xray
with contrast it showed vesicoureteral reflux unilaterally. What is the Dx?
A-Vesicoureteral reflux
B-Agenesis of right ureter
Answer: A

96. A man came from Sudan a few days or weeks ago. He developed fever
(high grade 40.1) then he sweat and fever stops. He done blood smear =

12
negative , How to investigate?
A-Thick blood smear
B-Thin blood smear
C-Forgot other options
Answer: repeat blood smear q8hr for 48hr

97/98- Patient had gastric bypass then weeks later develop abdominal pain rebound
tenderness X ray showed small bowel multiple air fluid levels What is the next step?
-Exploratory
-Conservative with ABX
Answer: A?
( 2 scenarios about post sleeve, One with severe pain, Other with pain and fever and
rebound tenderness. Both were unstable both asking about next management).

99. female pregnant US photo shows twins which one of the following will
increase mortality and morbidity?
A-preterm
B-vaginal delivery
C-C/S
D-instrumental delivery
Answer: A

100. Child present with sever dehydration and fever ( sunken eyes and
decrease skin turgor ) septic work up ordered, what to do next ?
A- 20ml/kg IVF bolus
B- Maintenance IVF until results come back
Answer: A

101. What’s the type of polyp that has the highest risk of being malignant?
A. Villous
Answer: A

103- Which of the following is one of the kawasaki 5 criteria?


A. Arthritis
B. Myocarditis
C. Anterior uveitis
D. Bilatral congunctival bulbar injection
Answer: D

105- A mother presented to you with her 2 years child who was yelling and throwing himself
on the floor, how would you deal with her
A- positive reinforcement counselling

13
B- Ignorance counseling
C- Strict and firm counseling
Answer: A

107- A child came for vaccinations , his brother known for immunodeficiency , which of the
following vaccines should be postponed
A-IPV
B-Varicella
C-DTaP
D-hepatitis A
Answer: B

108- which vaccine is contraindicated in HIV pt


A. OPV
B. varicella
C. MMR
D. DTP

109- Pt with GERD came and wants consultation about esophagus and stomach cancer,
What’s the most associated risk (something like this) ?
A-Smoking
B-Barrette
C-Obesity
D-Smith unrelated
Answer: GERD= Adenocarcinoma, Smoking= SSC

110. World health organization (WHO) to determine the health of the population
in KSA what should KSA share to WHO:
A-Health determinant
B-Health promotion
Answer: A?

111. Vomiting and diarrhea, acid base changes ?

112. Pain relieved with defecation, no bleeding, dx ?


A- Celiac
B- IBS
Answer: B

113. Pan systolic murmur ?


A. VSD

14
Answer: A

114. RUQ pain, Fever, Jaundice, What to do ?


A. ERCP
Answer: A, after resuscitation

115. FLAT T Wave After vomiting and diarrhea:


A- Hyponatremia
B- hypoMg
C- HypoCa
D- can't remember
Answer: Hypokalemia

116. Lactating mother with a 5 cm breast mass with thinning of the skin and tender?
A Incision and drainage
B Conservative with antibiotics
Answer: A

117. Patient has far nipples , something face ?


A. 45x
B. 47xxx
Answer: A, Turner syndrome

118. 11 year bed wetting?


A- bed alarm
B- educate
C- Concerned and strict parents
D- positive something
Answer: educate initially then bed alarm “not sure”

119- Pregnancy 7 weeks with vaginal bleeding. And no intrauterine or extrauterine sac
A. US after 2 days
B. Reassurance
C. Discharge home
Answer: A

120- Another recall:


She is pregnant 6 weeks closed cervix but abdominal pain, vaginal ultrasound no sac or
fetal pole,, what to give ?
and another one, Similar to above but with severe bleeding ?

Answer: revise type of abortions

121. ASD WHAT TO DO ?


A-Repair now
B-closed before 3-5 years

15
Answer: B

122- Lady presenting with vaginal bleeding with fever 15 days after C/S. High WBC Most
probable Dx?
A. Wound infection
B. Retained products of conception
C. Endometritis
D. Mastitis
Answer: C

123. Patient takes aspirin, and a lot of drug , develops postural


hypotension ?
A. Medication side effect.
Answer: incomplete!

124. kid with penicillin allergy ? What not to give ?


A- Moxifloacin
B- Tazosen
C- Pipracillin
Answer: C

125- RTA , sternal fracture , decreased heart sound , hypotension. Dx?:


A-Cardiac temponade
B-Cardiac contusion
C-Ventrical rupture
Answer: A

126. TB, Mode of transmission?


A. Airborne
Answer: A

126. Asthma patient wakes up from sleep 6-7 times , already on Saba what to
add ?
A.ICS
Answer:

127- 15 year-old, his friend died in a car accident and he told his mom he's thinking of
suicide but not going to act on it, what to do?
A⁃ Asking him directly about his suicidality is gonna increase the risk of him acting on it
B⁃ Ask him in details about his suicidal thoughts and feelings
C- reassurance and never to mention suicide thoughts to patient
Answer: B?

128. In hospital , already informed to the patient that surgery is not allowed ,
what to tell ?
A. INFORM HIM IT IS NOT ALLOWED

16
Answer: A

129. Patient refused amputation ?


Sign lama/DAMA and discharge
Answer: Sign on Refusal form‫ز‬

130. Celiac with DM type 1 screening:


A- at time of dx then Every 5 years
B- at time of dx then every 2 years in asymptomatic patient
C- annually
Answer: B

131- In children of diabetic mothers, 20% glucose is given in?


A. central line
B. peripheral line
C. NGT
Answer: A

132. painful ear pain in child for 2 days and perforated tympanic membrane,
dx?
A- acute OM
B- chronic OM
Answer: A

135. Pt with hand injury clean open injury with separation of tendons and
median nerve (injured) what’s best management
A-Primary repair of injured structures
B-debrid and something
C-Debrid and secondary closure
Answer: A

136-Young female has sparse pubic hair not reaching the mons pubis, breast budding with
areolar enlargement, but with no clear distinction between breast and areola. No contour of
breast was seen. What Tanner stage is she?
A- Tanner I
B-Tanner II
C-Tanner III
D-Tanner IV
Answer:B

17
137- A 45 year old male smoker had an adenoma removed from his colon. Pathology report
shows a benign lesion. What advice should you give this patient to prevent him from getting
colon cancer?
a-Ear a low-fibre diet
b-Eat a high-protein diet
c. Colonoscopy every year
d-Stop smoking and start exercising
Answer: D

139. 7 years long scenario very normal healthy girl mother concerned of early
puberty something like that Kid has only pubic hair that’s on majora and
mons not extending no axillary or facial hair
A-Clitros normal (not CAH)
B-Breast normal (no turner)
C-Premature adrenarche
D-Normal puberty
Answer: C

141. What’s the folic acid dosage used for supplements pre conceptions
(Not sure if those are the options exactly like in exam )
A-0.4
B-0.8
C-1.4
D-1.8
Answer: A, If healthy

144. Patient received 5mg warfarin his inr is 7 , known case of AF ?


A- Reduce to 2.5 mg
B- Same dose
C- Stop warfarin till become therapy then resume +/- low dose vitamin K
Answer: C

145. Child with severe projectile vomiting with dehydration type of fluid?
A- Ns
B- Potassium
Answer: A

146. Female smoker wants to quit, what to give for smoke cessation;
A) nicotine patch
B) vareniclin
C) bupropion
Answer: A

147. smoking cessation for pt with chest pain on exertion and relieved by rest
on examination he’s anxious with nicotine staining on fingers

18
A- buspirone
B- varenicline
C- nicotine replacement therapy
Answer:

148. Smoker with anxiety wants to stop smoking intial mx :


A)bupropion
B)varencillen
C)nicotine replacement
D)motivational support
Answer: D

149. A pregnant came to your clinic because she would stop smoking, which
one of the following is a useful tool you could recommend for smoking
cessation?
A-Bupropion
B-Varenicline
C-Behavior therapy
D-Nicotine replacement therapy
Answer: C

150. 9-month Hx of MI came to clinic for smoking cessation he was in respiratory


distress raised JVP nicotine tear in his hand what is the most appropriate
drug?
A. Nicotine replacement therapy
B. Varenicline
C. Bupropion
Answer:

151. Male patient is heavy smoker and he is known to have epilepsy which is
well controlled on carbamazepine. Patient is counseled for smoking cessation.
Which of the following is contraindicated in this patient ?
A. Nicotine replacement therapy
B. Varenicline
C. Bupropion
Answer: C

152. Which among the following is NOT a side effect of bupropion?


A. Diarrhoea
B. Blurred vision
C. increased sweating
D. dry mouth
Answer: A

19
153. Patient in labor received mgso and pressure dropped what to do and
contracted also dropped ?
A. MGSO
Answer: INCOMPELET.!

155. Patient with complicated thyroid cyst part of it is solid next step in
management ?
A. FNA from solid
B. FNA from cyst
C. Core needle biopsy
Answer: A

156- Female she came complaining of itching and bad smell after delivery and episiotomy,
she went to a doctor he suspect uti and gave her antibiotic but still she didn’t improve, then
she went to another doctor after examination her said there is a missed gauze inside, what
is the mistake that the first doctor did?
A- he let the midwife help in delivery
B- second doctor communicate result to the first doctor
C- genuine something "‫"تشخيص حقيقي والفرق بين الدكتورين كيف فحصوا‬
D- didn’t check the safety protocol before leaving OR
Answer: D

157. Female Patient with nausea & vomiting with each period last one was
vomiting with minimal amounts of blood now she is stable what the
management ?
A.Arrange for urgent endoscope
B.Reassure & till her to come back if she had the symptom again
Answer: B

158. Post menopause 53y dyspareunia LMP 15 months age


A.Estrogen creaam
B.Estrogen Oral
C.Ocp
D.Depo
Answer: A

159. 30 years old female 30 weeks GA 2cm cervical dilatation 50%


effacement with PPROM the doctor give her steroids & tocolytic drugs
Tocolytic purpose ?
A.Delay labor until steroids work
B.Prolong pregnancy until 37
Answer: A

20
160. Average 3 years old what can he do?
A. Triangle
B. Climb stairs
C. Known Week days
D. Can catch ball with alternate leg ( something like that)
Answer: B

161. Postpartum Depression management?


A.Give therapy to rest of the Family
B.Calcium supplement
C.Give drug ( forget the name)
D.Continue to breastfeed the baby
Answer:

162. Bilateral distal and proximal pharangeal joints pain with nodule no
Morning, c reactive normal, Rheumatoid factor normal ?
A.polyarticular gout
B-Reactive arthritis
C-Rheumatoid arthritis
D-Osteoarthritis
Answer: D

163. Scd in pregnancy HB 97 management:


A. Simple blood transfusion
B. Close follow up throughout the pregnancy
Answer: B

164. A famous player went to the hospital and the doctor forget to log off &
passenger take photo of the screen and Posted who is fault ?
A.no one
B.the passenger
C.nurse how didt check after the doctor
D. the doctor
Answer: D?

165. Most common pneumonia in adults ?


A.streptococcus pneumonia
Answer: A

166. Female with breast mass first step in management?


A.Us both beasts
B.FNA
Answer: A

21
167- what is the most common site of intussuscption ?
A- ilioilial
B- Iliocolic
Answer: B

168- Baby blinks alot but with Oriented:


A-Tics
B-Tourette syndrome
Answer: A

169- Most common child leukemia’s ?Other recall, what is the most common type of
leukemia in children?
A- acute myeloid
B- acute lymphoblastic
Answer: B

170- which antipsychotic causes weight gain?


A) olanzapine
B) clozapine
C) risperidal
D) forgot
Answer: A vs B

171- Neonate 3m have IDA pallor and pale oral mucosa what is ttt ?
A- im iron
B- oral ferrous
C- blood transfusion
Answer: B

172- Most common cause for swimmer’s ear with yellowish discharge:
A) pseudomonas aerugionsa
B) staphylococcus aureus
Answer: A

174- Surgeon diagnosed a pt with appendicitis then he decided to do surgical


appendectomy. Intraoperatively he found appendix normal. Then doctor removed the
appendix. What is the most Appropriate action with regards to this decision?
A. Tell the patient that Dr found appendix normal and removed it according to guidelines
B. Don't tell the patient
C. Call another Surgeon to tell him
D. Inform the ethical committee

22
Answer: A

175- Case of Parkinson disease, which one is the following factors presents in patients who
are high risk of developing dementia?
A. Forgetting future appointment
B. Word finding when talking
Answer: B?

176- 30 y/o mechanic accidentally inject by pressure his left Index finger with disolve
Next:
A.oral antibiotic
B.oral antibiotics and steriod
C.surgical open and drainage
D.observation
Answer: C

177- Pregnant + did OGTT post 1hrs 8.1mmol Wt the risk of infection ?
A-candida
B-BV
C-Trachiomosis
Answer : A
Risk Factors: These include diabetes mellitus, systemic antibiotics, pregnancy, obesity, and
decreased immunity.

178- Case with scales on scalp and face but not covering the diaper edges :
A-Scapes
B-Atopic dermatitis
Answer: B

179- What is the role of antihypertensive in pre-eclampsia?


A- To prevent maternal complication as stroke
B- prevent IUGR
C- prevent fetus demise
What is the role of antihypertensive in pre-eclampsia
Answer: A

180. Pt in icu post op on TPN feeding for a while then he developed fever and
leaukocytosis, Dx ?
A.Central line infection
Answer: A

181- Pt present with perianal pain and fever, on DR exam you found posterior
fullness with mimimal discharge , Dx ?

23
A-Abcess
B-Perianal fistula
Answer: A?

182- Pt present with lower back pain with bilateral knee and ankle pain and
swelling with morning stiffness for 1 hour, no extra-articular manifestaion, RF
or imaging not mentioned Dx?
A-Ankolysing spondylitis
B-Rheumatoid arthritis
Answer: A

184. Pt present with small intestine obstruction, imaging showed 3 stricture


on the ilium, Dx ?
A-Crohns
B-GI Stromal tumor
C-Intestinal lymphoma
Answer: A

185- Child present with palpitation with HR=260 or 290 and other systemic
exam unremarkable, ECG provided (regular rythem with narrow complex)
Dx ?
A-Svt
B-Atrial flutter
Answer: A

186- Calculate child pugh score for cirrhosis pt with severe acsitis and severe
encephalopathy?
A. class A
B. class B
C. class C
D. This score Not for cirrhosis patients
Answer: C

187- Pt with Rh. Arthritis presented with unilateral knee swelling with fever,
with high wbc count on swelling fluid, management ?
A-Steroid injected to the knee
B-Iv abx
Answer: B

188- 37 weeks in labor with Intrauterine fetal death with regular contraction

24
and cervix 6cm, mgt ?
A-CS
B-Augmented labour
Answer: B?

190- SVD and placenta was attathced to the fundus and manual extraction
failed, doctor decide to do hysterectomy but pt refused as she didn’t
complete her family, so doctor planned to go with medical therapy with
methotrexate, most likely complication of medical therapy ?
A-Infection
B-DIC
Answer: B?

191- 9 wks pregnant on exam fundus was palpable just at the symphyses
pubis, what to order next ?
A-Transvagina us
B-Ct chest
Answer: A

192- During vaginal examination u felt soft structure:


A. BREACH PRESENTATION
Answer: A

193- 55 year female in ICU post hartmann procedure on the 2ed day post op, has 20ml/d~
urine output Central line pressure 10mm. Mx?
A. re explore abdomen
B. CT with contrast abdomen
C. Duplex us renal
D. MRI abdomen or us abdomen not sure.
Answer: A? “Maybe abdominal compartment”

194- Post mi came with chest pain, on auscultation sharp sound, cough with white frothy
sputum Dx?
A. tricuspid rupture.
B. PE
C. Papillary muscles rupture.
Answer: C

195- 4 year old post MVA splenectomy, Need vaccine?


A. Influenza
B. Meningococcal
C. d-Tab
D. Tetanus
Answer: B

25
196- Child post URTI has consistently dry cough at night on SABA improved slightly but
night coughs still present what to give ?
A. Oral something
B. Some type of steroid
Answer: Incomplete! maybe inhaled steroid

197- Female with heavy vaginal bleeding, Hypotensive, Hb 7. Proper next step?
A. hysterectomy
B. Treat anemia
C.? Progesterone ~not sure
D.? D&C ~ not sure
Answer:

198- Child with ALL on chemo , had some infection symptoms fever ...
A. Blood urine... culture and paracetamol
B. Blood, urine,... culture and PO Abx
C. Blood, urine,... culture and IV Abx
Answer: C

199- Pregnant at 27w stable, Cervical dilated 1cm head at -2


A. induction of labor
B. observe with CTG monitoring
No medication in the choices
Answer: B?

200- Female old urge urination frequently dysuria on PE (some type of prolapse) what is
next ?
A. urine analysis
B.C.D all about types of prolapse operations
Answer: A

201- Fever acidosis clubbing:


A.Empyema
B.Pl effusion
C.Tb effusion
Answer: A?

202- SLE on azathioprine hydrochloro prednisone c/o hip pain xray normal:
A. lupus arthritis
B. Avn
C. Septic arthritis
Answer: B

26
203- infected bedsores low bp:
A. Septic shock
Answer: A

204- Adrenalectomy periop steroid:


A.Periop hydrocortisone
B.Postop fluco
Answer: A

205- the patient was stabbed in multiple areas, came to the ER and was awake,his clothes
were full of blood, his bp is around 80/60, which of the following is best next step?
A) oxygen supplementation.
B) iv crystalloid infusion
C) Intubation
Answer: ABC

206- Pregnancy lady with heartburn which resolves with antacid , after a while symptoms
recur again what to do?
A.upper endoscopy
B.ph manometry
C.barium enema
Answer: PPI

207- A girl who has a first time tonic clonic seizure for 5 minute (I think) has a positive family
history. What is the appropriate next step?
A.CT head
B.EEG
Answer:

208- The optimal time for treatment (brucellosis)?


A.6 weeks
B.12 weeks
C.24 weeks
Answer:
Simple = 6 weeks
spondylitis = 12 weeks
neurobrucella = 6 months

209- -42w IOL w prostaglandins contractions started after an hour fully dilated +2 but
sudden fetal bradycardia 70 and keep getting worse mx?
A.C/s
B.Start pushing and consider ventose
C.Give oxytocin

27
Answer: B?

210- diarrhea and b12 deficiency dx?


A.Celiac
B.Crohns
Answer: B?

211- 32 post CS 16 days , presenting with fever not responding to antipyretic and Abx,
previous multiple Cs with significant adhesion. Ct shows small bowel fistula l, which is
appropriate next?
A. NPO and start pancreatic and gastric secretion inhibitors
B. resect and anastomose
C. restrict to soft diet
D. colostomy
Answer: A?

212- bronze skin w testicular atrophy, initial inv?


A.24 urine copper
B.Antiendomysial antibody
C.Liver biopsy
Answer: C “Dx: hemocheomatosis”

213- 8w larger than size passing vesicles dx?


A.Molar
B.Missed
C.Threatened
Answer: A

214- atelectasis and pneumonia dx?


A.Rigid bronchoscope
B.Flexible bronchoscope
C.Fluoroscopy
Answer: B

215- DKA forgot to take his bolus insulin what to give after IVF?
A.IV insulin
B.IV insulin with bolus insulin\
Answer: A

216- neck LN with follicular cells?


A.Excision
B.Refer to surgery
Answer: B

28
217- married cousins, the husband’s sister has scd, came preconception clinic, what to
advise?
A.Screening before conception
Answer:

218- Pictures: ecg svt, henoch schonlein purpura, hurchsprung, ulnar fracture, air under
diaphragm, fibroid, genital ulcer, impetigo, early deceleration

219- Pt had central abdominal stab wound, vitally stable what’s next?
A-Ct abdomen
B-Exp laparotomy
C-FAST
Answer: A

220- pt of RTA came with lower limb can't move and worm extremity, her BP low 70/60
corrected with 1L normal saline, And ask what should add to improve her Bp,, CVP was2
A.Normal saline
B.blood transfusion
C.hypertonic saline
D.mannitol
Answer: A

221- Greenish -brown discoloration of skin of 20 y pt what is the Dx?


A.Hemochromatosis
B.Wilson dz
Answer: B?

222- 50 yrs female diagnosed before 20 yrs with hashimotes thyroditis .. came now with
rapidly enlargement of thyroid and weight loss? ( there was no labs and no extra info)
A. subclinical thyroiditis
B. medullary carcinoma
Answer: B

223- Pregnant rh negative and her baby positive?


A.Give anti d to mother only
B.Give anti d to mother and fetus
Answer: A

224- When to do GBS swab?


A.35 weeks
B.25 weks
C.15 weeks

29
Answer: A, 35-47W

225- Patient has rash on buttocks and lower limbs, what is commonest joint involvement?
A.iliosacral
B.Hip
C.knee and ankle
D.Small joints of the foot
Answer: C

226- What is the dose of folic acid in milligrams for pregnant without any hx of previous
malformation or risk of neural tube defect?
A)0.2
B)0.6
C)1.4
D)2.4
Answer: B, 0.4

227- Child c/o lower limb pain during running and playing basketball, there is mild pretibial
edema, limbing, what is the diagnosis?
A)Tibioiliac band syndrome
B)patellar instability (I think!)
C)Osgood-Schlatter syndrome
Answer: C

228- Pictures: “See pic file”


1-rectal prolapse, Scenario: female c/o rectal pain and hx of passing mucus.
2-fibroid, Scenario: female in her 30s c/o vaginal bleeding for 2 years
3-microcytic hypochromic anemia asking about the management, Scenario: child on cow’s
milk based formula since 9 month of age, Answer: iron supplemantaion

229- Endometrial biopsy shows atypical hyperplasia, what is true regarding management?
(nothing in the scenario was about fertility desire!)
A.progesterone
B.Tamoxifen
Answer: A?

230- Child presented c/o lower limb pain,he has a history of upper respiratory tract infection
1 week ago, he has fever and tenderness over the right thigh and groin, high ESR, high
WBCs, what is the diagnosis?
A)Osteomyelitis
B)Septic arthritis
C)Synovitis
D)myositis
Answer: C

30
231- Mother came with her child for vaccination, he has history of anaphylaxis after 4 month
vaccination, she didn’t know what the exact vaccine he is allergic to, what is you action?
A)Delay all vaccines
B)Give all vaccines
C)Give HBV vaccine
D)Give all except Dtap
Answer: A?

232- 34 years came for antenatal care found excessive amniotic fluid.
A.Renal age sis
B.Rh isoimmunization
C.IUGR
D.uncontrold DM
Answer: D

233- Indication of peatue-food and egg to decrease the allergy?


A.10
B.14
C.18
D.22
Answer: A

234- Low grade fever + Acute onset strider


A.Acute epiglottis
B.Acute tonsillitis
C.Acute laryrngitis
D.Viral laryngotracheobronchiti
Answer: D

235- 24 women G2 p1 come to clinc for first visit , dr ask glucose intolerance test , why he
order this early ?
A.BMI 20
B.Husband DM
C.Previous Clift lip
D.Previous GDM
Answer: D

236- ‫ (†وجاني†سؤال†يقول†وصلت†كميه†محدودة†من†تطعيمة†االنفلونزا‬pcv 3 ) ‫األولويه†لمين‬


A.school child
B.Pregnant women
C.Isolated hypertension something
D.Iron deficiency pt
Answer: B

31
237- 4 years child with N/V and headache and vision problems since 6 week increased at
morning and late night improve with activity?
A.Brain MRI
B.Abd US
C.EEG
D.muscle biopsy and something
Answer: A

238- 20 years old female divorced asking for pap when to do it!
A.Now
B.After 1 y
C.After 3
D.When she has symptoms
Answer: B

239- How long should children do activities per day


A.30
B.60
C.120
D.150
Answer: B

240- Child 4 month with watery diarrhea for 2 days Vital Bb 80/ Pulse ; 130, What to do?
A. Give all vaccine
B. Give all vaccine without OPV
C. Delay all
D.forget it
Answer: A?

another recall:
Mother and her child visited the pediatrician for 6 months vaccination, however the mother
stated that her child was hospitalized after receiving the 4 months vaccination he develop
anaphylaxis, what is the right thing to do?
A. Test the child for skin test
B. Give him all vaccines except DTP
C. Vaccinate him and discharge home
D. Vaccinate him and observation for 1 hour
Answer: A

241- Female with 4 month history of mass, no sx of hypo or hyperthyroidism all was (-) US:
7 mm solid nodule with no cervical node enlargement What’s the management?
A. Follow up imaging
B. FNA
C. TST
D. ct chest
Answer: C

32
242- old patient came with subdural hematoma with signs of lateralization imaging revealed
13 mm shifting. his GCS 7/15 then was intubated and resuscitated what to do next?
A. iv mannitol
B. admit ICU and observe
C. craniotomy
D. insert intracranial monitor
Answer: C

243- Chest pain retrosternal for 3 hour with diaphoresis he is on bb, nitoroglycin and
clodeprgil, ECG show sinus tachycardia + LBBB , your action:
A.wait the cardiac enzymes result
B. thrombolysis
C. CT cardiogragh? Not sure
D. infusion lidocaine? Forget the name
Answer: B?

244- A male with RA and the disease was active.. they asked what would you give him?
A. MTX + prednisolone + hydroxychloroquine
B. MTX + sulfasalazine
C. all the other options didn’t have prednisolone
Answer: MTX + prednisolone

245- Common cause of nephrotic syndrome in 5 years old child ?


A. minimal change disease
B. Progressive glomerulonephritis
Answer: A

246- patient complaining of chest pain, diaphoresis and other symptoms of MI that started 2
hours ago. He has a history of ischemic stroke (2 months ago). what would you give him?
A.thrombolytic
B.PCI
C.ASA (aspirin)
Answer: PCI

248- ECG pic (PEAKED T WAVE-hyperkalemia)

249- Hernia palated lateral and inferior to pubic tubercle is:


A- inguinal
B- femoral
Answer: B

33
251- A study was done about the effects of green tea vs black tea on a certain disease risk
ratio of the effect of green tea in comparison to black tea is 0.85 whats your interpretation?
A- green tea is better than black tea
B- black tea is better than green tea
C- there’s no difference
D- the result happened by chance
Answer: A

252- Post RTA patient complex femoral fracture and tension pneumothorax, chest tube
inserted and transporting to higher center, in transit the patient is desatting and
tachypnoec-cardic what to do?
A. Intubation
B. Nothing
C. Check for bleeding form fracture site
D. Confirm placement of chest tube
Answer: D

254- most degenerative ovarian cyst?


A.Red
B.cystic
C.hyaline
D.another option I don’t recall
Answer: A

255- Too many question about ectopic pregnancy

256- pictures ( bacterial vaginitis, malaria, Down syndrome features, open fracture , ECG
inferior MI)

257- too many questions about Neck injury zones management

259- Try to conceive + hx of chlamydia infection:


A.pelvic Imaging
B.Ovaries US
C.Hystersalpingiocope
Answer: C

260- 24 y.o Patient high FSH, LH, normal GTH, Amenorrhea for 9 months. What is
diagnosis?
A. Ovarian tumor
B. Ovarian insufficiency
C. PCOS
D. I forgot

34
Answer: B

261- Old pt present to ER with massive lower gi bleeding what the cause( believe me this
only what they said)
A.angiodysplasia
B.diverticulosis
C.colon cancer
D.forget
Answer: B

263. Women with 42 weeks gestation No contraction , cervical dilation 2, -3 station, normal
CTG, what u want to do:
A- reassurance
B- Cs
C- induction of labor
D- augmentation labor
Answer: C

264. Treatment of impetigo:


A.Antibitoic
Answer: A

267. 71 y/o patient post subtotal cholectomy, after days his urine output become 0.5mL/hr.
What is the most appropriate drug to be given intravenously?
A- Antibiotics
B- Diuretics
C- 500 ml NS challenge test
Answer: C

268. Case of Child has Intussusception asking what the most common site ( there no
Ileocecal in the options)
A.Ileoileal
B.Jejunojeno something like this
C.Ileocolic
Answer: C

269. case of brucellosis with low back pain and Hx with unpasteurized milk ،what is
treatment:
A.Doxycycline and streptomycin
Answer: A, but 1st line is Doxycycline +rifampin for 12 weeks

270. women came to you with US showed molar pregnancy what is managment?
A- D/C

35
B- methotrexate
C- suction & evaction
D- hysterectomy
Answer: C

271. Molar pregnancy follow up of HCG:


A- weekly till 3 reading negative then 6 months
B- weekly then monthly for 6 months
C-Monthly then weekly
D- monthly till negative
Answer: A

272 . Pt diagnosed with HNT only which drug to start with


A-ACEI
B.amlodipine
C.thiazide
Answer: A

273 . SLE nephritis what to add


A. ACEI lisinopril
Answer: A

274. 2 colon cancer screening questions (repeated)

276. a lot of trauma questions (around 10)

277. RDS/TTN/meconium aspiration Qs

278. A patient presents with a peaked T-wave on ECG. Labs show high potassium. How will
you manage?
A. Insulin
B. Salbutamol inhaler
C. Glucagon
D. Calcium gluconate
I got this Q without an ECG
Answer: D

279. Long scenario about patient with dx of necrotizing pancreas with sepsis and treated,
now on mechanical ventilation and inotropes. What is the physiological process?
A) Increase lipolysis
B) Increase insulin resistance
C) Hypoglycemia
D) Decrease Gluconeogenesis
Answer: A

36
280. 25 yo female Case about multi nodular large goiter needed total thyroidectomy , 12
hours post-op came with SOB and Difficult breathing on examination: there was severe
respiratory distress and very large anterior neck mass , how will you mange initially ?
A- bedside thoracotomy
B - bedside local wound incision
C - transfer to OR
Answer: B

281. Dm sugar is Controlled for the past year but it was not before what’s the treatment for
diabetic neuropathy (diagnosis was mentioned)
A. amitriptyline
B. vit b 12
Answer: A

282. Child came with high liver enzymes and fever I think started within 2 weeks— how to
diagnose?
A. Hepatitis A IgG
B. Hepatitis A igM
C. Hepatitis B
D. Hepatitis C
Answer: B

283. 35 female with sacroiliac stiffness for more than an hour and bilateral knee arthritis and
achilles tendonitis ESR high 200
A. RA
B. AS
C. SLE
D. crystal disease
Answer: A

284. Sle Sx and lab values of low complements high ESR how to confirm diagnosis:
A. ANA
B. Anti Ds-DNA
C. Anti CCP
Answer: B

285. Malaria prophylaxis in Southern ksa?


A-Mefloquine
B- atovaquone proguanil
Answer: B

37
286. scenario about physiological Gerd

287. celiac and asthma questions

288- Male 50 y/o no Hx for alcoholic or using drugs or anything come with hematemesis
fresh bright blood for 5h On PE no tenderness no guarding Dx?
A. ?
B. ?
C. duodenal ulcer
D. mallory Weiss
Answer:

Another Recall:
An male patient who is a known alcoholic for many years presents with abdominal pain,
vomiting and hematemesis. He was recently managed as a case of perforated peptic ulcer.
On exam, there is epigastric tenderness. Liver enzymes are normal. What is the most likely
diagnosis?
a. Acute pancreatitis
b. Chronic pancreatitis
c. Varices
d. Mallory-weiss
Answer: B VS D?

189- Hematemesis palpable spleen. proper therapy?


A. ?
B. ?
C. Osterotide
D. vasopressin
Answer: C, case of Esophageal varices

290- Neonate 2 days old (i think) presented with cyanosis and egg on a string on x-ray ,
Dx?
A- TOF
B- Transpositioning of the great vessels
Answer: B

291- Women with schizophrenia (on tow medication, i forgot name) had orofacial abnormal
movement for 1 month:
A- catatrophic movement
B- Tardive dyskinesia
C- schizophrenia presentation
Answer: B

38
292- Features of down syndrome child asking abt cardiovascular abnormality:
A.cushin defect (or AVSD the same thing)
Answer: A

294- A child starts to develop an awareness for strangers and separation anxiety. How old is
the child in months?
a. 6 months
b. 7 months
c. 12 months
d. 24 months
Answer: C

295- Long long case with ECG pic but in hx they mentioned yellow vision so its: A.digoxin
toxicity.
Answer: A

296- opioid antidote?


A.Naloxone
Answer: A
(Another Recall: patient with pin point pupils and want antidote what you will give ?
Naloxone)

297- -MI patient was given streptokinase then he developed hematemesis , how can we
reverse streptokinase adverse effect?
A-aminocaproic acid
B-Vit K
C-Factor VIII
Answer: A

298- Most common intrabdominal tumor in children?


A. Wilm tumor
B. Neuroblastoma
Answer: B

299- Patient presented with right metatarsal Swelling and redness also he has fever , what
are the possible causes
A: Staphylococcus aureus
B: salmonella bacteria
C: Urate crystals
D: calcium peryaphosphate

39
Answer: C

300- Researcher while analyzing data omitted data to change the outcome results?
A- Falsification
B- Fibrication
Answer: A

302-Screening for pregnant for asymptomatic Bacteriuria


A. 12 weeks
B. 20 weeks
C. 26 weeks
D. 32 weeks
Answer: A

304- Pt, k/c of SLE, she is on prednisolone, hydroxuchlorcqune , MMF , she want to
pregnant, what to do?
A) Stop MMF start MTX
B) Stop MMF start azathioprine
Answer: B

305- female with vaginal discharge and on examination there is red strawberry cevix . what
Dx?
A- Chlamydi
B- nisseria gonorrhea
C- Trichomonas vaginalis
D- Candidiasis
Answer: C

306-patient treated with antibiotics for salpingitis but not effective what is the most causative
organism?
A) N. gonorrhea
B. C. Trachomatis
Answer: B

307- baby has discoloration of his tooth and molars, he sleeps with the milk bottle in his
mouth, what could be the cause?
A- nursing bottle caries
B- Gingivostomatitis
Answer: A

40
308- Orpganophosphate Antidote:
A.Atropine
Answer: A

30- Child with suspension of ADHD, how to diagnose?


A- Ask more information from teachers and parents
B- Other information
Answer: A

311- Female with postpartum depression for 5 days, the baby is breastfed well, but she is
sad and cry, what is appropriate initial step?
A- discharge follow up after 1 week
B- SSRI
C- benzodiazepines
D- multivitamin
Answer is A; based on duration she is having postpartum blues, the first line therapy is
supportive and close observation.

312- A 23-year-old primigravida presented at 32-weeks of gestation with seizure


(see lab results),
Blood pressure 160/110 mmHg
Heart rate 78 /min
Respiratory rate 18 /min
Temperature 36.6°C
Test: Protein +++'
Which of the following is the most appropriate next step in management?
A) steroids
B) diuretics
C) hydralazine
D) magnesium sulfate
Answer: D

313- GERD after gastric sleeve percentage


A- 5%
B- 10%
C- 20%
D- 30%
Answer: D

41
314- 24y/o male presented with *diarrhea and fatigue* after chemotherapy what electrolytes
abnormality would you suspect?
- A- hypokalemia
- B- hypocalcemia
- C- hyponatremia
- D- hypomagnesemia
Answer: B

315- Newborn with history of physiological jaundice. Now presented after 2 weeks with
jaundice and pale stool, dx?
A. ABO incompatibility
B. RH incompatibility
C. Biliary atresia
Answer is: C

316-HCV most common genotype


A-1
B-2
C-3
D-4
Answer: D

317- Child with recurrent URTIS, eczema and thrombocytopenia both brother and uncle
have the same condition:
A Wiskott Aldrich
B Hypogammaglobulinemia
C HIV
D Agammaglobulinemia
Answer: A

318- At which age we screen for IDA in pediatric


A) At birth
B) 2 months
C) 4 months
D) 12 months
Answer: D

- Pregnant in labor and signs of meconium stain how to manage baby? ICQ
A - Suction Oropharynx before deliver the body
B-NICU
C-TOCOLYTIC
D-Intratracheal suctioning
Answer: B

42
-Child with bone age greater than chronological age, what is the diagnosis?
A- Congenital Adrenal hyperplasia
B- Chronic renal disease
C- Hypothyroidism
D- Familial short stature
Answer: A

-Baby cyanosis while feeding stridor and improved in prone position?


A-It will get better by his first birthday
B-Flexible nasopharyngeal scope
C-X ray
Answer: A

-Couple came to preconception clinic the father 2 of their brother have sickle cell
disease and he has sickle cell trait and what is the risk of SCD ?
A. Risk is very high
B. No risk at all
C. Minimal risk if the mother is negative
Answer: C

After herniotomy surgery for 5 years boy came with fever and pus discharge and part
of mesh seen?
A. Give IV antibiotics
B. Draining of pus
C. Draining of pus and remove mesh
D. Observation
Answer: C

Patient diagnosed with CKD now he is coming in outpatient clinic and you decided to
start him on hemodialysis, his crt is 10, what best method of access to start
hemodialysis?
A. Arteriovenous graft
B. Arteriovenous fistula
C. Tunneled venous catheter
D. Non tunneled venous catheter
Answer:

-9 wks pregnant on exam fundus was palpable just at the symphses pubis, what to
order next ?
A-Transvagina us
B-Ct chest
Answer: A

43
-A pregnant lady tells the nurse not to transfuse blood, during surgery she had sever
bleeding and become hypotensive. What to do?
A-Get blood and Transfuse
B-Inform the ethics
C-Take consent from her before she lose conciosness
D-Take consent from husband
Answer: C?

Child with muscular dystrophy. He is in RDS the parents refuse intubation because
they had a child and he died. What to do?
A-Intubate
B-Call ethics
C-Call child rights
D-Do nothing
Answer: A

-How to diagnose a 12yrs girl with HTN?


A-Above 90th percentile BP for age and gender
B-Above 95th percentile BP for age and gender
C-Above 140/90 mmHg
Answer: B

-Child with jaundice and cataract what is the diagnosis?


A. galactosemia
Answer: A

-Senario of Pneumothorax after chest trauma what will you do initially :


A-Needle decompression
B-Chest tube
Answer: A

-Female with decrease sensation in left calf and lateral side of the left foot, what is the level
of spinal stenosis
A- L3-L4
B- L5-L5
C- L5-S1
D- S1-S2
Answer: C

-16 years old with headache, unilateral, preceded with nausea, she has similar attacks 5
times per week, each lasting 3-4 hrs, CT of head is normal, what will you give her?
A. Sumatriptan
Answer: A

44
-Father in ICU who has pain and family ask for painkillers , if you give him he could
die faster, what do you call this?
A-Double effect
Answer: A

-Pt with history of pre eclampsia and IUGR, how to reduce the risk of them in future
pregnancies?
A. Aspirin
Answer: A

-How can we diagnose the pt as cervical insufficiency based on transvaginal US?


A. < 5 mm
B. < 10 mm
C. < 25 mm
D. < 30 mm
Answer: C

-Dr ask you to prepare the RT kidney for op but u are sure it was the LT one what
should u do:
A- tell the surgeon
B- tell chief intern
C- review imaging scan
D- ethical comm
Answer: A

Lady with bruises because her husband beat her, what to do?
A- reassurance
B- inform authorities
C- talk to the husband
Answer: B

-A case of MI — lateral wall

-Child with foreign body aspiration ( peanut ) and doctor prepare him to do
endoscopy, where did you expect its location?
A-larynx
B-trachea
C-right main bronchus
D-left main bronchus
Answer: C

-75-year-old type 2 diabetic undergoes a bowel resection for colon cancer.


He is well priar to the operation with well-controlled diabetes and no other
underlying condition. The operation is successful and the patient is given postoperative
insulin and IV dextrose. 2 days after the operation he becomes very agitated. Which of the
following is the most likely cause?

45
A.Water overload
B. Addison's disease
C. Diabetic nephropathy
D. SIADH
Answer: A

-7 years old girl having nausea and vomiting whenever on airplane flight for mor than 1 hour
what to give .
A-nothing
B-ondansetron
C-granisetron
D-diphenhydramine
Answer: D

-Left hemiparesis for 10 days and started physiotherapy, management?


A-Aspirin
B- heparin
C- thrombolysis
D- warfarin
Answer: A

-Child with barking cough (croup presentation) what else will you find in the auscultation in
the chest:
A- wheezing
B- stridor
Answer: B

10-year-old child came with fever and vesicular rash on the palms associated with painful
mouth ulcer. The majority of his classmates have the same issue. What additional findings
you’ll see in this patient?
A- Enlarged congested tonsils
B- Rash on the soles
Answer: B

-Pt known case of rheumatic heart disease with prosthetic valve 3 years ago. Going for
dental procedure, which of the following is the best prophylaxis?
A- Doxycycline
B- Amoxicillin or ampicillin
C- Ciprofloxacin
Answer: B

Young asthma pt has symptoms after exertion but now present at the clinic
asymptomatic. Spirometry was done and was inconclusive. What’s the next step?

46
A- repeat spirometry
B- methacholine challenge test
Answer: B

- morbid obese GERD grad 3 which procedure:


A. Roux y surgery

-Patient had cancer pancreas with lt lower limb edematous tender


A-Becker cyst rupture
B-Cellulitis
C-Dvt
Answer: Incomplete.

-Breast erythema US 2*3 collection suspect cyst or abscess strat ABX what next?
A- observation
B- incision and drainage.
Answer: B

-Pt after 10 days follow gastric sleeve develop sob and diminish popliteal pulse what next
step do for this pt??
A.TPA
B.heparin
C.IVC filter
D.aspirin
Answer: B?

-46 years old female complaining of joint pain for 6 month, with significantly stiffness in the
morning. Pain now localized on MCP joint & PP joint, associated with bilateral severe knee
pain started 3 weeks ago. What is your management? (Complete case)
A- Methotrexate
B- Methotrexate & Hydroxychloroquine
C- Methotrexate & steroids
D- Methotrexate & Hydroxy & steroids
Answer: C

-Strongest risk factor of Endometrial carcinoma:


A.PCOS
Answer: A

-Which drug you will give for acute management of headache?


A. Aspirin
Answer: INCOMPELET

-TB pt with high liver enzymes what will you do

47
A. stop all
B. stop pyrazenamide
C. stop rifambicin
D. stop INH
Answer:
stop if:
5 fold even if asymptomatic
3 fold + symptomatic

Patient with 2 stage chronic renal disease presented with fatique and pallor
His lab was:
Hgb Low
MCV Low
What is the most appropriate test you will order for him?
A. Iron study
B. Erythropoietin level
C. Hgb electrophoresis
Answer: A

post 7 cycle chemotherapy fever with high neutrophils 70% next


A- give him NSIAD and antibiotic
B- wait for cuture sensitivity
C- give empiric Abx
Answer: C

Most common organism of bronchiolitis


A) RSV
B) parainfluenza virus
Answer: A

69 man and asking for appropriate vaccination:


A-Pneumococcal
Answer: A

-Patient obese smoker with co morbidity and concerned about cancer did
colonoscopy normal Now what can he do to decrease risk of cancer:
A.Life style reduces wt and stop smoking
Answer: A

-7 years female with eyelid erythema and thickening of skin over metacarpophalyngeal
joints + proximal muscle weakness ??
A- Juvenile Dermatomyositis

48
B- Scleroderma
C- SLE
Answer: A
-patient immune compromised what to avoid?
A. Varicell
B. Pertussis I
C. PV
D. Influenza
Answer: A

- pediatric patient with RA on methotrexate and adalimimab came with a dirrheal infection
with fever (seems bacterial) what do we do?
A-Stop adalimumab..
B-stop methotrexate..
C-stop both drugs and give abx
D-continue current rx and give Abx
Answer: C

Pediatric pt has referral from the village with tipical scenario of noonan syndrome (unusual
facial characteristics, short stature, heart defects present at birth, bleeding problems,
developmental delays, and malformations of the bones of the rib cage)
A. Sotos syndrome
B. noonan syndrome
C. Marfan syndrome
Answer: B

-Status epilepticus treated with lorazepam and doesn’t improve what will u give next??
A- Benzo
B- phenytoin
C- Phenobarb
Answer: B

Inferior MI was vitaly stable preparing for coronary perfusion suddenly be hypotensive what
is the cause Bp 80/40 PR 100?? BMJ, UpToDate
A- RV infarction
B- Heart block
Answer: A

Patient diagnosed with major depression complaining of excessive thirst and urination + she
has cancer with lung mets. investigations show hyponatremia and low urine osmolality
what's the dx: BMJ
A- DI
B- psychogenic polydipsia
C- SIADH
Answer: B
(The SIADH should be suspected in any patient with hyponatremia,
hypoosmolality, and a urine osmolality above 100 mosmol/kg.)

49
- Patient presented with Hematemesis medical free, no hx of alcoholic use , no
hx of medications, Endoscopy done show dilated vessels something on
esophagus ?
A. Acute pancreatitis
B. Chronic pancreatitis
C. Esophageal varices
D. Pancreatic psodou
Answer: C

- 3 years has intermittent loss stool only during day, examination all normal,
Dx?
A. Rotavirus diarrhea
B. Salmonella diarrhea
C. toddler diarrhea
D. Food allergy
Answer: C

- 17 year old female, medically free, (athlete) gymnast in her class, breasts later
and never menstruated, on developed examination she is tanner stage 5, but no
menstruation, diagnosis?
A.Hypothalamic hypogonadism
B.Transverse vaginal septum
C.Gonadal agenesis
D.Testicular feminization
Answer: A

- A patient coming from an endemic area for malaria. On Quinine(or


Quinidine?!). Later on, he was admitted to the hospital with Heart Failure.
Doctor did not ask the patient about travel or medication hx. Thus, he ordered
the nurse to give digoxin. The patient went into deterioration. How could it be
prevented?
A.Computerized drug prescription system.
B Nurse failed to write drug formularies.
C. Pharmacist who failed to check prescription.
D.Patient who did not bring his medicine pack/ report his medicines.
Answer: A

- Treatment of chron’s, controlled symptoms on steroid and another drug ,


came with multi-fistula ? What to give ? No abx in options
A Infliximab
B Azathioprine
C Methotrexate
D Budesonide
Answer: A

- Long scenario of acute cholangitis, then after pt was resuscitated with fluid
and antibiotic what should be done next ?(VITALS WAS NOT MENTIINED ) US

50
stone in gallbladder and dilated CBD ?
A. ECRP
B. MRCP
C. Cholecystectomy
Answer: A

- Patient K\C of low lying placenta previa and stable on follow up, came to
the ER today on week 33 or 32 with minimal bleeding and stop at home, what
is initial step?
A CTG
B US
C Biophysical profile
D Emergency delivery
Answer: B

-Pediatric case with UTI clear symptoms , fever 39 , mentioned that child look
unwell , asking about treatment :
A) oral antibiotic
B) iv antibiotic
Answer: B

-Case of COPD , absent gag reflex (nothing about consciousness ) on sever


respiratory distress Ph:7.25
High pCO2 and low O
O sat: 83
What is your next best step:
A. Intubation and ventilation
B. Non invasive
Answer: A

16- Female pt complaining of amenorrhea for 4 month


T4 low
Prolactin 200
What the dx
A- primary hypothyroidism
B- hyperprolactinemia
Answer: B

-Patient with Parkinson and visual hallucination


A.Low body dementia
Answer: A

-Postop 6 days with nausea vomiting and abdominal distention and can't pass stool Labs
show hypokalemia What investigation you need to order
A. Urine analysis

51
B.ECG
C. Urine K
D. Stool k
Answer: B?
To be honest none of these tests are indicated since the cause of hypokalemia is clear in
this case (the persistent nausea or vomiting), although if we are working up Hypokalemia
anyway the main test that we do is a urinary potassium level (either a spot urine potassium
and creatinine or ideally a 24 hour urine potassium level) in order to ensure that there is no
additional inappropriate renal potassium excretion, another helpful test would be a
Magnesium level and a VBG/ABG as the acid base status is important in the diagnosis
And approach Stool potassium can be a part of the workup but isn't as important as urine
potassium and this patient is not passing stool anyway
An ECG win;t change much since we'll just replace the Potassium anyway (this isn't like
hyperK where there are special measures like giving Calcium)
https://www.uptodate.com/contents/evaluation-of-the-adult-patientwithhypokalemia?
search=hypokalemia&source=search_result&selectedTitle=3~150&usage_ty
pe=default&display_rank=3#H6814660

- Female GBS swab 100000, 2 cont in 10 minutes, cervix 7 cm , wt to do :


a. Oxytocin
B. Ampicillin
Answer: B

- A patient with renal failure presents with low calcium, high alkaline phosphatase,
And high PTH. What is the best form of vitamin D to give this patient?
A- Calcitriol
B- Vitamin D2
Answer: A

- Neonate with seizure with normal glucose level , high Hb and RBC ,and the seizure
stopped spontaneously ) ‫متأكده انه الجلكوز كان نورمال ( يعني وقفت من نفسها بدون‬
‫تدخل‬
What is the management?
A- oxygen
B- fluid
C - dextrose
D- lorazepam
Answer: B, case of neonatal polycythemia

- Pt had RTA 6 months ago and have quadriplegia, came today with 2 hrs MI sx what is the
most appropriate management?
A- streptokinase, bb, aspirin,heparin
B-bb aspirin heparin
Answer: A?

52
- What is the minor effect of DTaP vaccine?
A-fever
B-site reaction
Answer: B

- Old male with a history of chronic retrosternal chest pain and exertional dyspnea, after 3
weeks he started to complain about the same problem but now he is still sitting on chairs or
reading a book at night.
A- Unstable
B- prinzmetal
Answer: A

- Pedia case of a child drooling , unstable ?


A. Ct chest
B. X ray
C. Laryngoscape in or
Answer: C

3 years child had drug composed of (na) but convulsions not controlled. What drug
must take in hospital:
A- Diazepam
B- Lorazepam
C- Phenityon
D- Phenoparbiton
Answer: A?

- Pregnant 15 GA kco epilepsy on phenytoin last attack 6 years ago came antenatal visit:
A. Continue same
B. Stop phenytoin
C. Refer to neurologist to stop medication
Answer: A?

- Female exclusively breast feeding want contraception for 2 years


A. Depo provera injection
B. Pop
C. Ocp
Answer: A

- Case about osteoarthritis then asking about.. How to confirm osteoarthritis


A- no need
Forgot the others
Answer: A, x ray?

- Minimal time to get pregnant after hernial repair female did hernia repair, ask
what is the minimum time (not best time) tel she can get pregnant?
A-3m
B-6m

53
C-12m
D-24m
Answer: B, Best=> C

- Pt known he asthmatic he is on SABA only came to ER with sob and now dx by


(persistent moderate asthma) what you will add on his medication
A LABA
B ICS
C ICS and LABA
D forget
Answer: C

- patient came to the hospital, after history and examination you suspected his
symptoms are similar to MRSA Cov, the patient feared loss of his job if the doctor
reported the case in hospital. What should you do:
A- confirm the diagnosis then report
B- report within 24 h
Answer: B

- 4 yo boy presented with fever and low abdominal pain. Mother complains that his urine
has a foul smell. UA: gram +ive bacteria >100,000. What of the following could be the
cause?
A- E.coli
B- Klebsiella pneumoniae
C- Proteus mirabilis
Answer: A

- Women 33 year old undergo renal surgery and during the surgery they found
incidental polyp 5*5 in size , what’s moat appropriate action ?
A- myomectomy
B- leave it
C- abdominal CT
Answer: B

- Antibiotic contraindicated in pregnancy ?


A-ciprofloxacin
B- ceftriaxone
Answer: A

- Pediatric case, came with conjunctivitis, staccato cough, no diarrhea cxr shows
lung infiltration, labs: esinophilia
-adenovirus
-chlamydia
Answer: B

54
- Mother strictly vegan and she is breastfeeding her baby asking what deficiency
baby will have?
-Vit b12
-Vit D
Answer: A

Cause of electrolyte disturbance in glioblastoma?


A-Salt wasting nephropathy
B-Excessive consumption of water
C-SIADH
Answer: Cerebral salt wasting syndrome

14 years old boy came for routine clinic. Does not have any major medical issues. On
examination you find irregular pulse (ECG picture provided with irregular rhythm but normal
P wave), Vital signs all normal including HR
Height >95th percentile
Weight 25th percentile
Mother is asking you about the prognosis?
A- Multiple syncopal episodes
B- Normal development
C- Will need pacemaker
D- myocardial dysfunction
Answer: C

Pt had prosthetic valve disease since 10 days. Underwent dental procedure and developed
infective endocarditis. What’s the organism caused infective endocarditis?
A- Staph aureus
B- Staph epidermis
C- Strept viridians
Answer: A, acc to UTD

(another recall )
Most common organism in infective endocarditis? (Direct Q )

55
Pt with high cholesterol, triglycerides and TSH. T4 was normal, What is the best medication
for her Dyslipidemia?
A- Statin
B- Thyroxin
C- Niacin
Answer: B

most Common cause of death in kidney failure patient ?


A.cardiovascular disease
Answer: A

Pt have Rt testes swelling for 6 months, in the last 4 months it increases in size, in
examination, it was seperable from the testes and doesn’t reach the inguinal, DX?
A - spermatocele
B- epidydmal cyst
C - indirect hernia
Answer: A

40 something pt come for post employee check up found to have cardiomegaly in xray,
Asymptomatic, ecg normal, Echo left EF 40%, What to do next?
A.ACEI
B.Digoxin
C.Niacin
D.Echo after 3 or 6 months not sure
Answer: A

56
An elderly man on NSAIDs developed dyspepsia. Endoscopy showed gastritis. Labs
showed iron deficiency anemia with Hb= 9. What is the best method of treatment?
A- Iv iron
B- Blood transfusion
C- Oral iron
D- Erythropoietin
Answer: A

Pt with head trauma and coma since 5 days. What is the proper way for early feeding?
A. NGT
B. gastrostomy tube
C. central line
D.peripheral line
Answer: A

Child have diarrhea without vomiting ( no other sx), his mother concerned about
dehydration. What is the appropriate intervention?
A. oral rehydration
B. anti diarrhea
C. iv saline infusion
D. Change milk
Answer: A

Done:
SARA

֎13 weeks of gestation + during pelvic examination found cervical lesion, Next step?
1) pap
2) colposcopy
3) cone biopsy
4) endocervical currretage

֎ Pt recently diagnosed with congestive heart failure, what is most important initial
management?
1) bb and diuretics
2) ACEI and diuretic
3) nitrate and bb

֎Male patient recieve 4 liter blood transfusion, what is the most important complications?
1) hypokalemia
2) hypocalcemia
3) citrate toxicity
4) hyper albunemia

֎20 years old lady, recently delivered, presents with symptoms of epigastric pain
whichradiates to the back and nausea and vomiting (not sure about vomiting), on
examination there’s 8x8cm mass in the epigastric area. What is the most likely diagnosis?
A-Gallbladder polyp
B-Pancreatic pseudocysts
C-Gastrointestinal stromal tumor

57
֎ Child has splenomegaly and abdominal pain, signs of anemia, hx of cholecystectomy
Labs were as the following:
Low Hgb - High reticulocytes - NORMAL MCV - Peripheral blood smear was done : Showed
microspherocytes Anisocytosis , Most likely diagnosis?
hereditary spherocytosis OR SCD

֎ 35 female pt with urgency and frequency for 1 month, suspected overactive bladder
syndrome, what is most important next step?
1) urine culture
2) uridynamic
3) cystescopy
4) uroflowmetry
(Another recall)

֎ Patent with active bladder syndrome how you diagnose it ?


A) urodynamic
B) uroflowmetry
C) us
Initial c , definitive A

֎ 24 male pr complaining of tenderness and redness in perianal area, he has history of


recurrent presentation end with spontaneous discharge, abd examenation normal, perianal
examination shows one fistula with redness , Appropriate management?
1) pelvic ct
2) oral Abx with outpt followup
3) needle aspiration
4) examinations under anesthesia
Answer : Although this patient would need to be worked up for Crohn’s, this acute
presentation is very suggestive of abscess. EUA, followed by incision and drainage would
be the way to go.

֎ Male patient with bilateral large breast, make him embarrassed, what is the most
important
next step?
1) LFT and TFT
2) bilateral mastectomy
3) core biopsy in each breast

֎ Pt known case of heart disease take warfarin,lisnopril, amiodarone , diagnosed with tb


and start the intial 4 anti tb medication, what is most important percaution?
1) increase dose of warfarin
2) stop refampin
3) decrease dose of lisinopril

֎ Pt has endometriosis, which one of the following is acceptable diagnostic test for
endometriosis?
A. Endometrial biopsy
B. Ultrasound
Answer:

58
• Transvaginal ultrasound (best initial test) • Laparoscopy (confirmatory test)

֎ 30 years old female, pregnant , GA 33 , presented with fever, rigor , and loin pain , she
has hx of UTI twice in this pregnancy. What is the most appropriate management?
A.MRI pelvis
B. Uretroscopy
C. Septic screening
D. Ultrasound urinary tract

֎ 20 yrs old, has persistent moderate asthma for 6 months, on SABA LABA ICS, he has
STRIDOR, symptoms are especially present at basketball in college?
Next best step
Add leuktrine
Modify ICS to medium dose
refer for otolaryngology evaluation

֎ Pregnant has RUQ, US was was done and was deemed to be CHOLELITHIASIS, Next
step?
Lap chole
Open chole
Conservative management and reassess after pregnancy
NB: Single attack of biliary colic. No signs of acute chole. Conservative for now is
appropriate

֎ 28 GA, has hard breast lump, not sure painful or not. Best next step
bilateral US
bilateral mammogram
Reassure and evaluate postpartum

֎ Right side weekness “equal” in face arm and legs, what is artery affected ?
A.ACA
B.Vertebral artery
C.penetrating branch, MCA (together in same option)

֎ Low BP+ warm + bradycardia, cvp is 2 (no baseline) management? (neurogenic shock)
A. Iv crystalloid
B. Normal saline
C. corticosteroids

֎ Contraindications for vaccine?


Abx
Moderate or severe illness

֎ Child with fever and vomiting and rash on 2nd day rash become over All body?
A. Meningococcemia.
B. Rocky mountain fever
C. Kawasaki.
D. Measles

59
֎ Patient with intestinal obstruction and perforated and the doctor decided to do EX LAP
what is the contraindications?!
A) nitric oxid
B) proprafol

֎ Patent with reduced femoral pulse, Ct angio (show aortoiliac insufficiency) what is the
mangment?!
A) aortofeomral bypass
B) something ballon
C) other thing

֎ Female asymptotic came with varicose veins what is the appropriate investigation??
A) CT angio
B) duplex US

֎ What is the cuses of high fever in first 24 h in abdominal surgery patient??


A) wound infection
B) Pneumonia atelectasis

֎ IBS patient what diet should be avoid?


A) lentils * ‫العدس‬

֎ Patient with medullary thyroid cancer what is the management?


A) total thyroidectomy
B) subtotal thyroidectomy
C) hemithyroidectomy

֎ Patient has high calcium what should you give him?


A) calcitol
B) methramycin ( something like that )
C) Start with IV fluids Nacl then clacitonin

֎ 46 female with polymenorrhea (just like this ) . How to establish the diagnosis ?!
A) endometrial sample
B) US
C)CT
D )MRI

֎ Female came with symptoms of vulval candidiasis , what could be the cause ?
A) sarcoidosis
B) Tb
C) DM

֎ GDM patient on poorly controlled diet what should you give her ??
A) metformin
B) insulin

֎ Child didn’t take chicken pox vaccine what is true regarding it’s vaccine??
A) 2 doses with 4 months apart
B) 3 doses

60
֎ Patient with mid systolic murmer at RT sternal border with S4 but no symptoms of heart
failure or displaced apex of heart asking about dignosis ?
A) aortic stenosis
B) aortic regurgitation
C) mitral regurgitation
D) aortic stenosis

֎ SLE patient on methotrexate and azthro came paraplegia and urinary inconstanc and
hyperreflexes what should you do?
A) LP with ct brain
B) LP with MRi brain
C) LP with mri spine

֎ DVT then became PE what is the high dignostic test ?


A) spiral CT
B) ECHO

֎ Patient came with neck stiffness and symptoms of meningitis , Low glocose and WBC
(70%polymorphic ) ?
Bacterial
Tb
Viral

֎ Child came with ploy arthritis small joint and big joint with hepatomegaly and spleenmegly
And salmon pink rash what is the dignosis ?!
A) oligo JIA
B) systems JIA
C) Poly JIA

֎ PIC of open fracture He is already on antibiotics :


A.Surgical debridement with intramedullary nail
B. Closed reduction with external fexation
C antibiotics

֎ Patient with jaundice splenomegaly and history of cholecystectomy for stone. Most
diagnostic test ?
A. Peripheral blood smear
B. Hemoglobin electrophoresis
C. Bone marrow biopsy
(No EMA or osmotic fragility btw.)
Answer: B or A

֎ Splenomegaly + gallstone(most likely due to hemolysis and pigmented stone ) + jaundice


due to hemolysis à SCA

֎ Splenomegaly + pigmented stones + hemolysis à Hereditary Spherocytosis


֎ Postpartum by one week came with shortness of breath and displaced apex beat.
A. Postpartum cardiomyopathy
B. Pulmonary embolism

61
֎ Women trying to get pregnant but failed. After work up it shows that she has bilateral
tubal
duct occlusion. How to induce fertility?
A. Induction with clomiphene
B. Induction with gonadotropins
C. IVF
D. Intrauterine insemination

֎ Pulmonary edema due to HF. Management?


furosemide
Spironolactone
ACEI
BB

֎ Old man says he sometimes forgets his friends’ names or celebrities in his community
and phone numbers. His wife is worried he has Alzheimer’s. Labs normal. He has tender
knees (not something amazing just cause he’s old). No mention how it affects his daily life.
A. Alzheimer’s
B. Benign forgetfulness

֎ YOUNG AGE PATIENT PRESENTS WITH HISTORY OF DYSMENORRHEA AND


DYSPAREUNIA, SHE REPORTED LEFT ENDOMETRIOMA AND UNDERWENT L.
ENDOMETRIOMECTOMY, NOW SHE HAS RIGHT ENDOMETRIOMA ( 6*7 CM) MX??
1) WAIT TILL BECOME 10 CM
2) HYSTERECTOMY
3) OOPHORECTOMY WITH SALPINGECTOMY
4) ENDOMETRIOMECTOMY WITH ABLATION OF

֎ I got this same question. She’s in her 30s but she completed her family.
History of endometrioma and she removed it came again with dysparunea
and abdominal pain she had another endometrioma

Patient with pancreatitis (given) and splenomegaly. On endoscopy normal portal vein and
gastric varices. Diagnosis?
A. Angiodysplasia
B. Left portal hypertension
C. Aortovenous shunt
D. Splenic artery aneurysm

֎ Cyclical Severe dyschezia similar picture Diagnosis


A.Endometriosis
B. Pid

62
֎ 12 YO patient with recurrent epigastric pain and vomiting and his
mother the same. Stool showed blood and mucus. Best for diagnosis:
A. Endoscopy
B. CT
C. Amylase and lipase
D. US (not sure)

֎ 6months baby milestone :


Sits without support
֎ Case of infantile colic (rule of 3) the same repeated question:
A- behavioral adaptaion
B- change formula
First is A as initial
Most effective =b

֎ Woman came with symptoms of pyelonephritis. Fever, pain, vomiting chills. High fever.
She has history of multiple UTIs.
A. Septic screen
B. Urinary tract US

֎ A patient with sudden loin pain and vomiting. She also has hematuria (from labs). She’s
rolling around from the pain.
A. Ureteric stone
B. Acute pyelonephritis
C. Acute cystitis

֎ Kid with cervical lymphadenopathy and sore throat (now). Has Coca Cola red urine.
1. IGA Nephropathy
2. Acute glomerulonephritis
3. Acute cystitis
IgA nephropathy 1-3 days (now)
If after 2 weeks then post-streptococcal glomrulonephritis

֎ Patient with long term history of DM type 1 in 12 weeks of gestation. HbA1C 12. Which of
the following complication is most likely to happen?
A. Preeclampsia
B. Polyhydroamnios
C. Congenital malformation
D. IUG

֎ Normal vaginal delivery, Baby weight 4.2kg, Laceration reaching rectal mucosa, which
degree:
A-First
B-Second
C-Third
D-Forth

֎ Pregnant at 39 weeks now in labor during the delivery you noticed the amniotic fluid is
mixed with dark black-green what is the cause of this color?
A- Meconium aspiration syndrome

63
B- Fetal distress
C- Placenta abruptio
D- Preterm labor

֎ During vaginal delivery, PPH with failed manual compression and oxytocin what you
should do next?
A. B-lynch
B. Bakri balloon
C. Artery ligation
D. Hysterectomy

֎ A lady with hematuria and swelling of lower limbs (very swollen), Labs showed HTN, low
albumin, high creatinine and BUN, and urinalysis was 5+ protein and, 20 RBCs (normal
2-4). No mention of previous URTI, or skin infections or any chronic illnesses or any drugs
used.
1.Acute interstitial nephritis
2. Acute glomerulonephritis
3. Post renal obstruction
4. Acute tubular necrosis

֎ Child came with uncontrolled seizure that manged in ER, on examination the child looks
dysmorphic, cant sit without support, what regard his vaccines.
A. Change OPV to IPV
B. defer all vaccines
C. Defer all live vaccines
D. Defer DTAP

֎ The lady was G2 P1, her labor was 2 hours long (baby came out before she barely made
it to the hospital). Baby’s weight was 3000. She started bleeding after delivery of the
placenta. Question asking about the cause of PPH.
1. Baby’s weight
2. Prolonged labor
3. Precipitous labor
4. Large multigravida

֎ Chronic AF will increase risk of?


A-MI
B-Sudden cardiac death
C- CVA (stroke)

֎ known of a sickle cell anemia (trial) you want to give her folic acid, what is the right
prescription for her?
A) 5 mg folic acid till 12 week
B) 5 mg folic acid till she gave birth
C) 5 μg folic acid till 12 week folic acid till 12 week
D) 5 μg folic acid till 12 week folic acid till
400 ‫ جاني السؤال ذا بس كان في اإلختيارات‬ug

֎ Mother just gave birth and decided to be given AntiRoGAM (anti-D), what is the indication
to give ?

64
- Rh negative Mother and Rh Positive Child
- Rh negative Father and Rh Positive Child
- Rh Positive Mother and Rh Negative Child
- Rh Positive Father and Rh Negative Child

֎ 25 YO pregnant lady at 30 weeks presenting contractions no bleeding no pooling. CTG


shows assuring status and good contractions.How to diagnose preterm labor:
1. Digital exam to see cervical dilatation
2. From the history
3. CTG showing assuring status
4. If there is no leaking then it is not preterm labor

֎ Male patient diabetic and smoker presented with ulcer on 2nd , 3rd and 4th left foot toes.
He gave a history of superficial thrombophlebitis. Upon examination: he has absent dorsalis
pedis and posterior tibialis pulse in both limbs , while popliteal pulse was intact. How would
you manage the patient?
- Amputation of toes
- Longterm anticoagulant
- Surgical intervention?
-Ask patient to stop smoking with lifestyle modifications

֎ GERD taking antacid mild relief. When laying down after eating feels
heartburn, what to do?
A. Add PPI
B. life style modification

֎ 2 questions same senario, pretem cyanosis SOB


>> What is the dx? Patent ductus arteriosis
>> What is the tx? IV indomethacin
>> Management : Indomethacin
֎ Mother recently delivered her baby, the doctor encouraged her for breast feeding
What is the most significant response?
1) decrease centrocranial infection
2) reduce primary immunity disorder
3) increase bounding between mother and baby

4) reduce the hemorrhagic disease for newborns


֎ Nephrotic syndrome which indicates complete resolution
A) normal serum albumin
B) resolution of edema
C) 3 days negative consecutive samples
D) 5 days negative consecutive samples

֎ 34wks present with pprom, management ?


A-Steroid
B-Tocolytics

֎ 66 YO, man with diastolic murmur best heard at the left sternal edge. And the apex is
displaced outwards. & pistol shot on femoral , What condition typically present with his
findings?

65
A) aortic coarctation
B) aortic regurgitation
C) mitral regurgitation
D) aortic stenosis

֎ Pregnant female + cottage cheese vaginal discharge, Dx and TTT.


Diagnosis > candida albicans.
Treatment > topical antifungal –azole.

֎ Investigation for renal scaring?


A- DMSA
B- MCUG

֎ croup pt given treatment with no improve after 1 hour what to give?


A-racemic epinephrine
B-dexamesathnie

֎ SLE patient on methotrexate with deranged LFTs what’s important to do?


A-Stop methotrexate
Answer: A, UTD: after obtaining an abnormal value, the clinician should adjust the weekly
dose of MTX downward (usually by 2.5 to 5 mg). The result should be confirmed with
retesting if this is the first time an elevation has been observed in the patient. The blood
testing should then be repeated after an interval of two to four weeks following dose
reduction to ascertain that the abnormal values have returned to the normal range.
MTX should be held, rather than just reduced, for patients with a value ≥3 to 5 times the
upper limit of normal.

֎ varicella which one will it affect the most??


A-eye
B-ear
C-liver
D-lung

֎ Hepatitis B vaccine?
A. Live attenuated
B. Recombinant
C. Toxoid
D. Killed

֎ Vaccines for 12 months? OPV, MMR, PCV, MCV

֎ Patient diagnosed with Kawasaki disease 2 months ago and receiving IVIG. Which
vaccine you can give?
A-Varicella
B-measles
C-opv
D- Dtap

֎ Unvaccinated boy and the mother said the vaccine not good. What to do!
A. Explain to the mother about the importance of vaccine

66
B. Ignore and treat
C. Inform child protection

֎ pt 30 wk present with PROM. has fever what give?


A-antibiotic
B-steriod

֎ Pt with paradoxical pulse what is causes:


A. Myocarditis
B. Pericarditis
C. Endocarditis

֎ A pregnant woman at 32 weeks gestation presents with severe


abdominal pain. she denies any abdominal bleeding. She has
CRL of 34 weeks. Examination reveals a tender and tense uterus.
what is the most
appropriate next step?
A. Perform an US
B. Cesarean section
C. Check CTG
Answer: A?

֎ HEELP syndrome, management.


(Mg sulfate , labetalol , delivery )

֎ SAAG > 1 , Dx
More than 1.1 > portal hypertension

֎ Patient with migraine headaches what medication is used for acute attacks?
A- Triptans
B- NSAIDs
C- SSRIs

֎ BABY after circumference bleeding. next step:


A-Apply pressure dressing

֎ Baby cyanosed murmure pic of heart like boot


A- TOF

֎ Vaginosis case ask about tx


A- Metronidazole

֎ child with bilateral knee swelling, (they gave the diagnosis as juvenile idiopathic arthritis)
asking about which type?
A. Systemic
B. Poly
C. oligo

֎ 35 yo female sexually active when to do pap


A-Every 1 year

67
B-Every 5 year
C-No need after 3 negative
After 30 yrs either pap/3y or pap+ HPV/ 5yrs

֎ Patient came to ER then collapsed, abdominal examination showed plusetil abdominal


mass (undiagnosed of AAA before) , what to do? Investigation test .
A- CT
B- US
C- laparotomy
Answer: B then C

֎ Adult male Pt is complaining of fever chills otherwise he’s literally normal, vitally stable,
lab findings are normal, what’s the most likely dx?
A. Sepsis
B. Bacteremia
C. SIRS
D. Sever sepsis

֎ Screening time for asymptomatic bacteruria in pregnancy?


Answer: 12-16 weeks

֎ Hypothyroid pt. For elective surgery what to do?


A. Give thyroxin and do surgery
B. Do surgery then give thyroxine
14. Elective thyroid surgery when to manage?
A. Manage when thyroid hormones are treated
֎ Pregnant with down syndrome what do you expect ?
( High BCG, high inhibin A, low AFP, low estriol)

֎ Child going for hernia repair 12 kg calculate maintenance fluids


A-1000
B-1100
C-1200
D-1300

֎ A patient with SLE and antiphospholipid syndrome, which type of


prophylactic anticoagulation you will give?
A - lifelong warfarin
B- Short term heparin
C- lifelong heparin

֎ Asking for chaperone when you examining patient.. this is represented which ethical
principle ?
1-privacy
2-non-malfence
3-4- Other choices are unrelated
Answer is Not sure

֎ Pregnant at 34 ga with sever preclamc what will be significantly decreased


A-Serum urice acid

68
B-Serum bun
C-Plasma volume < maybe?
D-Serum creatinine

֎Target INR in Mitral stenosis? 2 to 3


֎ 8 days old newborn for diabetic mother came with SOB and cyanosis, looks irritable and
ruddy, no flu like symptoms ( RR 40-49 / HR 169 / Hb 24.1 (normal: 165-175) / Hct 0.75 / Plt
120 ) What is the most appropriate next management?
A. Echocardiogram
B. Partial exchange transfusion
C. Hydration, oxygenation, suction

֎ Epistaxis best position. sit & forward

֎A lump in the neck move with tongue protrusion. Dx= thyroglossal cyst

֎ 14 months old (I am sure he is younger than 2 YO) his perants concerned that his right
testis is not palpable. On examination, right testis was NOT palpable in the scrotum or groin
(or pelvic, I forgot). What is the appropriate management?
A. LEFT orchioplexey
B. Diagnosetic laparoscopy
C. Give him testosterone
D. Observe until he bacomes 3 YO

֎ 10 year old boy presented with bleeding per rectum, and I think that is it no other
symptoms. On examination, there is blood coming out. Colonoscopy showed bleeding in the
iliocecal valve.What is the most likely diagnosis?
A. Angiodysplasia
B. Cronhs
C. Pud
D. Meckales devirticulum

֎ Female has headache, mastalgia, and i think behavioral changes 10 days before her
periods, and she is asymptotic for the rest of the cycle. What is the Dx?
A. Pms
B. pelvic congestion syndrome
ANSWER is PMDD

֎ Child with effusion from eardrum. No fever no pain. How to manage? (This is a
case of otitis media with effusion)
A-observe and follow up after 48 hrs
B-antiobiotics

֎ Pt on sigmoidoscopy found a mass turns out it was adenoma what will you do ?
A- Ct scan
B- Sigmoidectomy
C- Colonoscopy

֎ Patient had a sigmoid polyp removed. They found on histopathology well differentiated
adenocarcinoma and margins are free from cancer what is the best next step ?

69
A) Observation
B) sigmoidectomy
C) Segmental colectomy
D) Fulguration of the poly site

֎ Patient did sigmoidoscopy and found multiple masses in the distal sigmoid,
histopathology report was (adenocarcinoma) what will you do next?
A. Colonoscopy
B. Ct abdomen
C. SigmoidEctomy
D. Mri (not sure)
NB: as next step, to rule out synchronous cancer.

֎ patient underwent sigmoidoscopy found sigmoid lesion, histology report: Sigmoid


Adenocarcinoma with free margin with no invasion to mucosa. What is the next step?
A- Abdominal CT scan à for staging
B- Sigmoidectomy
C- Pelvic MRI
D- PET scan

֎ Which of the following is the most considerable risk factor for MI ?


A) HTN
B) Smoking
C) Age

֎ Female with urine incontinence (during coughing and laughing) , What will you do next ?
A- Bonny test
B- retrograde urethrogram
C- voiding cystourethrogram

Answer : Cough stress test


If not there choose A

֎Type of inheritance of Cystic fibrosis. AR

֎ The MOH is organizing campaigns, lectures, and health education in order to teach the
public about the dangers of obesity and its associated complications. What kind of
prevention is this?
A. Tertiary
B. Secondary
C. Primary
D. Primordial

֎ Long scenario then says: CT show asymmetric kidneys. Dx:


A. Renal artety stenosis
B. renal infract
C. primary hyperaldosteronism
( Other recall )
Pt has resistany HTN on 4 medication and found on US to have one small kidney, dx?
Renal artery stenosis

70
polycystic kidney disease
Answer is A

֎ child hypoglycemia & seizure & metabolic acidosis & characteristics smell & positive
ketone Diagnosis? marple syrup

COPD on pulmonary function test?


A- Increased TLC, Decreased FEV1/FEVC<0.7, Decreased VC
B- Decreased TLC, Increased FEV1/FEVC<0.7, Decreased VC
C- Decreased TLC, Decreased FEV1/FEVC<0.7, Decreased VC

Answer: A

Which type of the angina present when go to bed?


A- Stable
B- Unstable
C- Prinzmetal (variant) angina

Answer: C

Absent of femoral pulses, is associated with ?


A- Coarctation of aorta

Answer: A

What is the type of malaria in the pic blood smear?


A- Plasmodium falciparum
B- P. vivax,
C- P. ovale,
D- P. malariae.

Answer: A

A mother presented to you with her 2 years child who was yelling and

throwing himself on the floor, how would you deal with her
A- positive reinforcement counselling

71
B- Ignorance counseling
C- Strict and firm counseling

Answer: A

Patient came with clear history exam and imaging finding of flail chest

What’s the most important step to do:

Bp:low

Hr; high

Os:82

A-Ventilation

B-Chest tube

C-Thoracotomy

Answer: A

70 year old female came for check up asymptomatic found to have AS he is diabetic HTN.
Echo done showrr normal EF with concentric LV hypertrophy and severely stenosed Valve (
didnt mention how much cm) how would u manage her? ‫‏‬
A- Acei
B- Follow up and observe
C- aortic valve replacement
D- Diuretics

Scenario at the end there’s hemangioma in liver 4cmx4cm, What advice to give.

However, in case this is hepatic adenoma I did not find any relation of smoking

with it, but I found it is related to obesity and diet modification is considered a

general measure for all patients

Social smile at in weeks:

A.2

B.6

72
C.12

D.16

Answer: 2 months (nearest is B)

Child can tell story and run, resemble his father and draw head, body
A. 4y

Patient got rubella vaccine( doctor told her to avoid getting pregnant at least for 2 months).

But she did get pregnant after 1 month. What is the most common possible complication?

A- abortion in the first trimester

B- fetus dead in the third trimester

C- malformations

D- not affected.

Ansewr: D

Clear case of ARDS s/s + given Po2/Fio2 less than 200 and bi lung infiltrate

asking about diagnosis:

Dx = ARDS

see picture below

73
What’s the best management for (clear scenario of SBP) PT confused and

unstable ?

A-Diuretic and iv metro

B-Lactose Enema and iv cephalosporin

Other unlikely options

Answer: B

Pt with sever epigastric pain and vomiting blood , organomegaly , finding of gastric
varicocele on endoscopy, portal vein normal splenic or liver congestion I can’t remember

He doesn’t use med or drink alcohol and he has normal liver

A-Splenic artery Aneurism

B-Pancreatitis

C-Mallory weis syndrome

D-Esophageal variceal

Answer: B

Because it lead to splenic vein thrombosis then gastric varices, tx: splenectomy

Highly suspicion of malaria in person who came from Sudan, his blood film

was negative, what to do?

A-Repeat thin film

B-Repeat thick film

C-Repeat every 8h for 48h

Answer: C

Patient with 2 weeks history of watery diarrhea, vitals were stable What is the

74
expected acid-base abnormality?

A Metabolic alkalosis

B.Metabolic acidosis

C Compensated metabolic, acidosis

D. Compensated metabolic alkalosis

Answer: C

Patient with family hx of type 2 DM came with polyurea labs shows FBS of

7.0, HbA1c of 7.1. Dx?

A-prediabetes

B-DM type 2

C-Impaired glucose intolerance

Answer: B

34Y.O female diagnosed with hyperprolactinemia for one year on

cabergoline. Which of the following is an indication for brain MRI in this

patient?

A-Blurry vision.

B-Bilateral milky breast discharge.

C- Doubling of serum prolactin level.

D- Amenorrhea.

Answer: A

64 y.o had pervious stroke, DM, HTN, basal lung crepitation ,S3

and lower limb edema calculate the chadsvascs score?

A-1

B-2

75
C- 4

D-5

Answer: D

(Stroke hx: 2, DM:1, HTN:1, HF:1)

Old after blood transfusion has leukocytes without fever, dx ?!

A-Hemolytic disorder

B-Non hemolytic

C-Bacterial infection

Answer: incomplete scenario!

A mother didn’t introduce solid food to her child before, he’s 9 months

now, she begin to feed him some little amount, most important advice?!

A-Increase amount & be faster

B-Iron supplements

Answer: B

A child with asthma best advice for parents?!

A-influenza vaccine

B-keep house humidity on 50%

C-avoid playing outdoors

D-something about carpet change

Answer: A

Fetus with breach presentation flexing hip and extended knees, what is

the position?

A-Frank breech.

B-Complete breech

76
C-Incomplete breech

Answer: A

#Medications used in PPH Management:

1) Oxytocin 10-40 units in 500/1000ml NS/RL or 10 units IM (1st line)

2) Methylergonovine 0.2 mg IM (2nd line, Contraindicated in preeclampsia and HTN)

3) Carboprost (hemabate) PG F2a 0.25mg IM(3rd line, Contraindicated in Asthma)

4)Misoprostol 600-1000 micrograms PO,or rectal (4th line)

Female primigravida with irregular cycle and she is infertility for 3 years and with vaginal
spotting and tender abdomen and tender in cervix motion what to do?

A -Confirm pregnant status

B -US

C- Ask about cause of Infertility

Answer: A

1st pregnancy test to rule out ectopic pregnancy

Risk hx of infertility

Neoborn delivered at hospital, healthy, but mother reported that the

previous baby died due to immunodeficiency. What is the best action

regarding Saudi immunization program?

A-Don’t give BCG

B- Give BCG with close observation

C- Refer baby

Answer: A

child with wheezing and foreign body ingestion best investigation ?

A. Rigid bronchoscope

B. Flexible bronchoscope

77
Answer: A

Age of peanut introduction?

A-10 m

Answer: A

Mother who is Rh negative delivered a baby who is Rh + she was given

Anti-D Ig 300 microg what does it cover ?

A- 15 ml of the whole fetal blood

B- 30 ml of the whole fetal blood

C- 10 ml of the whole fetal blood

D- 15 ml of Rh(D) positive foetal RBCs

Answer: B

most common HCV genotype in saudi arabia?


A- 1
B- 2
C- 3
D- 4

Answer: D

30 y/o female has fibroid 10cm With heavy bleeding and doctor want to do hysterectomy
but her refuse ..what the definitive management:
A- Artery embolismation
B- OCP
C- Observe

Answer: A?

56 old male smoker with hx of hypertension what is the best advice?

A- low salt

B-Low fat

no stop smoking in option

78
Answer: A

treated breast cancer when she can get pregnant after treatment?
A- 3M
B- 9m
C- 2 year
D- 5year

Answer: C

50 year old lady had hx of miscarriage, now she asks if her age has any relationship to
miscarriage?
A- 3%
B- not related to age
C- more than 80%

Answer: C

post menopausal lady want Hormonal replacement, the doctor said HRT has no significant
effect for her. pt insest to have HRT:
A- refer to other doctor
B- refuse prescription
C- call the ethics commission
D- prescribe HRT

Answer: B

Pt cant swallow liquids more than solid food

What is the FIRST investigation?

A-Upper endoscope

B-Barium swallow

C-Chest CT

No manometry in options

Answer: A

79
PIC of stepping reflexes asking about what age it disappears at:

Answer: 2 m

influenza vaccine allergy with which food ?

A- egg

Answer: A

Important preschool vaccine?


A. Opv
B. Hib
C. Rota
D. Meningococcal

Answer: A

Child with High riding testes and very painful, absent cremasteric reflex, Dx?
A. Testicular torsion

Answer: A

Definitive treatment for endometriosis for a female who completed her family
A- hysterectomy
B- NSAID

80
Answer: A

Highest risk factor for esophageal ca:


A- Barrett’s esophagus
B- smoking age

Answer:

28 y/o male MVC victim. Vitals are stable and he complains of abdominal

pain. FAST done and shows peritoneal fluid behind the liver and sleep what is

the next management:


A- CT scan
B- Laparotomy
C- Laparoscopy
D- Observation

Answer: A

Female 18 year old came to Gyne doctor asking for pap smear, what to

tell her about pap smear:


A- when she is sexually active
B- At age of 21

Answer: B

81
28 years old patient came with severe perianal pain and swelling. On examination, there is
1x1 cm perianal swelling with tenderness. Vitals: normal, no fever. Labs: WBC 8 (normal).
Which of the following is the most likely diagnosis ?

A. Anal fistula

B. Anal fissure

C. Perianal abscess

D. Perianal hematoma.

Non-Hodgkin lymphoma on Tx developed tumor lysis syndrome:

A-Hypokalemia hypocalcemia

B-Hypokalemia hypercalcemia

C-Hyperkalemia hypocalcemia

D-Hyperkalemia hypercalcemia

Answer:C

1y.o child brought by his mom complaining if constipation since birth, his mom started high
fiber food and stool softeners 1 year ago. He has one motion every week. No diarrhea or
any symptoms of soiling. Doing PR examination - empty rectum with good anal tone - on
withdrawing the finger there was gush of stool. How would you manage this case?
(Hirschberg disease)

A-Increase stool softener dose

B-Refer to surgery

C-Refer to oncology

D-Increase water and high fiber diet

Answer:B

82
Pt came from Asian with typical symptoms of TB, hemoptysis, and X ray shows upper lobe
cavity what is the next step:

A. Start isoniazid (INH), rifampin (RIF), ethambutol (EMB), pyrazinamide (PZA)

B. Send for AFB test

C- IV ceftriaxone

D- isolation in negative room pressure

Answer:D

Pt has severe worst headache ever for several hours associated with

neck pain, what next to diagnose her?


A- mri brain -
B- Lumbar puncture
C- LUBNBAR PUCTUER AND MRI SPIN
D- LUMBER PUNCTURE AND MRA MRV

Answer: CT + LP

Pt presented with dysuria , dyspareunia , frequency , most likely dx:

A. Urethral diverticula

B. Urinary incontinence

C. Overflow

Answer: A

Patient with jaundice 5 days ago :


A- Hepatitis A IgM
B- Hepatitis A IgG
C- Hepatitis c
D- Hepatitis b

Answer: A

Initial mx of massive pulmonary embolism:


A- LMW heparin
B- Thrombolysis

Answer: B

83
Most common organism for meningitis in adult:
A- Strept pneumonia
B- Staph auoru
C- Listeria
D- Nisseria meningitis

Answer: A

Patient with facial and tongue swelling on BB , ACEI , diuretic

What is the cause ?


A- ACEI

Answer: A

GERD do endoscope was normal what is the most appropriate next

step ?
A- PPI
B- Abx
C- 24hr ph monitoring
D- Manometry

Answer: c

49- Patient want to do bariatric surgery what test do to before the surgery ?
A- Endoscope

Answer: A

A case of HUS, Pt come with fever headache high creatinine and hx of GI infection 1 week.
What’s the diagnostic culture test?!
A- Blood
B- Urine
C- MRI
D- Stool

Answer: D

84
Pregnant with herpes treated with Acyclovir, Rationals of acyclovir ?
A- Decrease Duration of infection
B- Increase immunity of mother
C- Decrease shedding of the virus

Answer: A vs C?

Q about best test to confirm h.pylori eradication :

Answer: Urea breath test

pediatric pt with sign and symptoms of dehydration, Management?


A- fluid replacement over 6 h
B- fluid deficit replacement over 12 h
C- fluid deficit replacement over 24 h
D- fluid deficit replacement over 48 h

Answer: C

3 years old boy barky cough no inspiratory stridor, positive monophasic wheeze in
auscultation, dx?
A- bronchiolitis
B- Tracheomalacia
C- Laryngomalacia
D- Bronchial asthma

Answer: B

Pediatric pt fail 2 days ago ( Fx of radius and ulna ) her parents said she doesn’t move her
hand after that but they think it was normal they didn’t see any bruises, Ask about most
identifying child abuse in this case ?
A- Delay presentation 2 days later
B- same parent story
C- type of the Fx radius and ulna

Answer: A

When does infantile colic end or go away?


A- 3 weeks of life
B- 6 week of life
C- 3 month of life
D- 6 month of life

85
Answer: C

Milestone , 3 years old:


A- clamp stairs
B- know day week
C- drow triangle
D- catch balls

Answer: A

Saudi’s vaccination program from birth to primary school aim to prevent:


A- HBv
B- Haemophilus influenza

Answer: B

when you detect pregnant women infected with HBV, type of prevention ;
A- -primary
B- -primordial
C- -secondary
D- -tertiary

Answer: C
MOH reported the infant mortality rate in 2020 which was 4.81 . Which of the following help
in calculating this ratio
A-Knowing the childbearing women in the middle of 2020
B- Population percentage in Saudi Arabia
C-Knowing percent of those died before the first year of their life.

2Q about intoxication

64- One q pt is euphoric, tachycardia and have visual hallucinations


A- amphetamine intoxication
B- cannabinoid intoxication
C- something withdrawal

Answer: A

31- Patient presented to the ER with diarrhea, nausea, vomiting, salivation, lacrimation and
abdominal cramps. What do you suspect?
A. Organophosphate poisoning
B. Paracetamol poisoning
C. Aspirin
D. Penicillin
Answer: A

86
Pt came to ER after MCA have multiple injuries, lost of 25% of blood , most expected
decreased is ?
A- UoP
B- pulse pressure
C- RR
D- GCS

Answer: B

Pt in ER after gun shoot in the chest 1 or 2 hours ago, chest thorax tube was done for LT
hemothorax , after 15 min there is 2 liters of blood in the seal , most appropriate next step ?
A- another chest tube
B- diagnostic scopy
C- Thoracotomy ?

Answer: C

Pt with stab wound in the neck 2 cm anterior to the ear , close to the angel of mandible
?most appropriate step ? (All the options were investegation)

Answer: Angio CT of the neck

Pt with face laceration underwent emergency repair procedure with therapeutic dose of
Lidocaine What is the Side effect?
A- Nystagmus
B- Droswness

Answer: A

pt with femur fx , before reduction most imp is ?


A- Give analgesic
B- give sedation

Answer: A?

Female pregnant, what of the following true regarding Elevated BhCG?


A- High BhCg indicator of ectopic pregnancy
B- High BhCg in second Trimester indicator of molar pregnancy
C- High BhCg in second Trimester is the most sensitive marker of Down syndrome
D- High Bhcg can cause depression of TSH

Answer: D, if not in the options then c. (according to dr.WAFA)

87
Pregnant in 34 week of GA came with labor pain admitted, she had PROM 4 weeks ago ,
O/E cervix is 3 cm, there is clear liquor, Ask about next step?
A- Emergency CS
B- call Anastasia and deliver pt
C- Wait until 37 weeks
D- give her corticosteroids

Answer: A?

lady c/o Amenorrhea.. she did D&C before because menorrhagia, Dx is:
A- PCoS
B- Asherman

Answer: B

Pt isolated then start the course of Abx , the Q was when to discharge the pt from isolation
room after the ABx ?
A- 24 h
B- 48h
C- 12 h

Answer: A

Pt dx with acute pancreatitis came with Hematemesis, hepatosplenomegaly ,varices

Investigators done show splenic vein thrombosis, spleen laceration , portal vein N !

What is the next step appropriate management?


A- splenectomy
B- splenorrhaphy

Answer: A

SLE pt with neuropsychatric symptoms ask about the management:


A- Steroid + Cyclophosphamide
B- Iv Cyclophosphamide-
C- Steroid

Answer: A

k/c of SLE came c/O unilateral knee pain only O/E tenderness when internal rotation of
knee is done

Ask about dx ?
A- Active lupus arthritis

Answer: A

Replacement in pt with this pic below:

88
A- Iron
B- Folic acid

Long scenario ask about What medication is used to reduces recurrent variceal bleeding?
A- BB

Answer: A

Pt with ovarian mass ask about the tumor marker ?


A- CA125
B- CEA

Answer: A

250- HIV patient had a PPD test done and it showed an induration of 6mm. What’s the
management?
A.INH and vit B6 for 9 months
B.INH and vit B6 for 3 months
C.nothing
D.INH + pyrazinamide + ethambutol + rifampin
Answer: A, if no clinical symptoms and radiographic changes

1-An old patient with symptoms of fever and productive cough admitted to isolation as a
case of T.B, then next day results came out showing him having HIV. What shall you do
now?
A. Start anti-HIV immediately

B. Start anti T.B for 3 months then anti-HIV
C. Consider Pneumocystis Carinii
D. Start both ant T.B and anti-HIV
Answers is B

89
According to uptodate
ART initiation should never be delayed until the completion of TB treatment; For
HIV-infected patients with pulmonary TB and CD4 cell count <50 cells/microL, we
recommend initiation of ART within two weeks after starting TB treatment (Grade 1A).
•For HIV-infected patients with pulmonary TB and CD4 count ≥50 cells/microL, we
recommend initiation of ART within eight weeks after starting TB treatment (Grade 1A).

3- Pt (mostly female i forget) came to er with abdominal pain central radiate to back within
24 or 12 and has history of cholelithiasis more than one attack, what is the dx?
• acute cholecystitis
• acute pancreatitis
• maybe peptic ulcer
• forgot

4-4- Septic shock case indicates adequate systemic perfusion?


What indicate adequate systemic perfusion?
A. Cardiac index
B. Mixed venous O2 saturation
C. Central venous pressure

D. pulmonary capillary wedge pressure

ℹ️ Other recall
the most indicative of systemic perfusion?

* central line✅
* periphral line

* cardiac index
* pcwc

6- dengue fever is more common in ksa in:


• east
• west
• South

• North
Other recall from earth
In order to eliminate dengue fever from saudi arabia with witch you should start
- central
- East
- West ✅
7- Female pt bleeding and cervical os is SEMI-open and patient denies passage, What
does this indicate?
• Complete
• Inevitable
• Threatened
• Missed

Old female constipation with difficult urination tx ?


A- Sacro colpopexy
B- Colporrhaphy ✅
C- Sacro hysteropexy

90
Note:
A - For vaginal prolapse
B - For cystocele or rectocele
C - For uterine prolapse

Case of a patient with multiple vomiting and ECG showed flat T wave what you will find?
A. Alkali urine
B. High urine K+
C. Aciduria
D. High urine Na+
Answer is c

11- Patient wants to travel, All his LFTS high. What to give?
• No prophylaxis needed
• Fluoroquinolone

13-•Female convinced she is allergic to gluten she said (wheat) and did gluten free diet by
her own and there was relief of symptoms:
•A. refer to gastroenterologist
•B. introduce gluten and then serolgy for celiac
•C. then no need to eat gluten if feel good
Answer: continue in gluten free diet and do serology test

5 month old boy with a history of flu like symptoms 3 days ago Came to
the ED with respiratory distress, subcostal retractions..etc His oxygen
saturation is 80% despite giving 100% O2 on mask What is the most
important next step in managment?
-IV steroids
-Intubation and mechanical ventilation
-Oral antibiotics
- I forgot

Child with recurrent chest infections and sinusitis bronchoscope and sputum culture showed


pseudomonas and burkholderia cepacia, what is the most likely diagnosis?
A. cystic fibrosis
B. primary ciliary

16- CF case, pt with wheezing and shortness of breath, taking Albuterol respond partially
and FFT, what’s of these symptoms that suspected to be due CF?
• FFT (failure to thrive)

1. Elderly was completely healthy except for elevated BP for the first time
A. Ambulatory BP measurement
B. Start anti HTN
C. Measure the BP two times later on in the clinic
D. Measure BP two times in home

91
2. Mother find her baby dead in his bed Exam post death within normal She is smoker, likely
cause of death
A. Sudden cardiac arrest
B. maternal smoking
C. Infant abuse
D. Congenital diseases

3. There was a question about 20 yrs female c/o intermiTTent crampy abdominal pain and
altered bowel habits pain relieved by defecation which can be cons9pa9ons and other 9mes
loose and watery content, and aPer defecation s9ll feel not empty her gut, what is the
appropriate management

A. h.pylori test
B. abd ct
C. I don’t recall other op9ons

4. u/s about endometriosis “ground glass appearance” asking about further complication
“infertility”

A. Aspirin✅
6. Infant diagnosed Kawasaki syndrome Which treatment

B. Cyclosporine
C. Prednisolone
D. Immunoglobulin

8. 5 month old infant with coughing, sneezing and circumoral cyanosis for 3 days. Recurrent
frothy sputum around the mouth. On examina9on there are diffuse? rhonchi all over the
chest and basal crackles but good air entry bilaterally. Chest x-ray shows bilateral infiltrates.
What is the most likely diagnosis?
A. Heart failure
B. Pneumonia
C. Bronchiolitis
D. Bronchiolitis Obliterans

9. patient diagnosed with acromegaly what you do for him in future?


A. transesophageal echo
B. colonoscopy

10. Pedia 4 years old come with concern mother about wetting he bed almost every night

A. Reassure ✅
he has a 8 years old sister didn't wet the bed when she was 3 how would you manage him

B. Give him a drug and restrict drink aPer 6pm


C. urine culture and treat any infection

Mother brought her child complaining of perianal itching and rice like coming out ?
A- stronglyloides infection

92
B- enterobius vermicularis

13.77-Right side hot nodular goiter 2*3 management with oxthalmus (eye protrusion)?
A-near total thyrodectomy
B-right hemithyrodectomy
C-radio active ablation

Answer: A
“In patients with coexistent thyroid cancer and those who refused RAI therapy or had severe
ophthalmopathy or life-threatening reactions to antithyroid medications (vasculitis,
agranulocytosis, or liver failure), total or near-total thyroidectomy was recommended.”
– Schwartz's Principles of Surgery, 11e
“total or near-total thyroidectomy as the procedure of choice for the surgical management of
Graves’ disease”
– American Thyroid Association

15. Dysmenorrhea doesn't relieve wit NSAIDs:


A. Endometriosis
B. not remember

16. Female in 20s has diabetic present with confusion. The pa9ent has a long history of
type 1 diabetes. She also complains of periorbital swelling, rhinorrhea, and black necrotic
spot over the face. labs show glucose 600 mg/ dl and ketones. Ct scan shows obliteration of
all the sinuses. Which of the following is the causative organism?

A. Rhizopus oryzae
B. Candida albicans

C. Moraxella catarrhalis
D. Staph. Aureus

A 45- year old man presents to the emergency department with a 3 day history of fever,
productive cough of yellowish sputum of yellowish sputum Medical history significant for
type 2 diabetes melitis. He is a smoker. His chest examination reveals decreased breath
sounds throughout and crackles at the right lung base . Other a systemic examination is
unremarkable. Chest radiograph shows -Right lower lobe infiltrate Which of the following is
the most appropriate management?
A. admit him start Ceftriaxone & Azithromycin
B. Admit him, start intravenous amoxicillin
C.Outpatient treatment with azithromycin
D. Outpatient treatment with CEFUROXIME & AZITHROMYCIN

19. Pt admitted to ICU given 15 units blood , developed bleeding from incision , iv site
A. Von willebrand
B. Hypocalcemia
C. Thrombocytopenia ✅
93
D. Transfusions reaction
30. How to monitor the response for pt taking H. Pylori medication:
A. Urea breath test ?
B. Stool
C. Blood test
D- Clinical manifestations

21. 60 y/o female did Pap smear and showed ( ASC-US ), her treating physician prescribed
her topical vaginal estrogen for 1 month, she came back aPer that and Pap smear repeated

A. Colposcopy
B. Punch biopsy

and it was also ( ASC -US ). What you will do for her

C. No further investigation
-Ascus twice is colposcopy Ascus once if older than 24 do hpv Ascus once and younger
than 24 repeat pap

22. A case of adrenal carcinoma is planned for surgery. What to do before the surgery?
A. PET Scan
B. 24-hour urine metanephrines
C. Serum cortisol level

D. Urinary cortisol level

A. no need to tx done
B. Antifungal tx

23. asymptomatic UTI with fungal infection and patient have indwelling catheter

-Candida can only cause uti when there is an indwelling catheter, and when removed
candida cannot adhere to urethra

24. A newly married woman has been on her honeymoon for 1 week and is now
complaining of dysuria and frequency. How will you manage?
A. Ampicillin
B. Nitrofurantoin
C. Ciprofloxacin

D. Cephtriaxone

A. Bb
B. Ccb

26. Pt post mi developed R HF he is on ACEI what should give him?

C. Arb
D. Heparin

27. A child with a recent history of fever now presents with bilateral lower limb paralysis and
tingling . The face, trunk, and upper limbs are not involved. On neurological examination,
both lower limbs have a power of 3/5 and absent reflexes. What is the cause of this
condi9on?
A. Poliomyelitis
B. Transverse myelitis


C. Mul9ple sclerosis
D. Guillain-Barre

94
28-105-A toddler presents with cough and wheeze. When he cries, it looks like he is having
breath-holding spells. What is the most likely diagnosis?
A. Asthma
B. Pneumonia
C. Croup
D. Bronchiolitis@ (RSV)
Answer is D, by exclusion

31-428--year old male with prosthetic valve developed infective endocarditis Blood cultures
done and were positive for MRSA What is the Abx regimen for this case: ??
A- Vancomycin & Gentamicin
B-Vancomycin & Rifampin
C-Vancomycin & Ceftriaxone

32-44 years women is conplaing of severe secondary dysmenorrhea and menorrhagia ,
pelvic examination reveals the uterus is symmetrically enlarged and tender . Endometrial
biopsy is normal

A.adenomyosis
B.leiomyomas

Which of following dx?

C.endometriosis
D.sarcoma

Tonsillitis and fever maculopapular rash after antibiotic >> EBV


140-Long scenario about a pediatric patient febrile with cough, and rash started in the face
and behind the ears and neck, minimal on the trunk, with cevical lymphadenopathy, what’s
the cause?
⁃ measles
⁃ Rubella
⁃ Chickenpox
⁃ Mumps

rash develop after fever subside. > > In Roseola Infantum .


– hand, foot and mouth rash >> Coxsackie

35-125-12 year old child complain of recurrent epigastric pain abdominal pain lasting for a
year , sometime associate with vomiting and restrosternal pain , his stool analysis was
positive for occult blood , what diagnose
A- IBD
B- IBS
C- Abdominal Migraine
D- peptic ulcer disease

answer : D
excluding
A >> bloody diarrhea , weight loss , crampy abdominal pain
B : negative for occult blood mucse in stool , blotting , constipation or diarrhea
C : + positive family history of headache , - negative for occult blood

95
37. internal hemroid 3 degree
Answer: rubber band ligation

38 lady is pregnant with twins. What is the most common risk factor associated with
increased mortality in multiple gestation?

A. Birth defects
B. Birth trauma
C. Prematurity
D. Placental insufficiency

The answer is C, according to UTD: "The most serious risk is preterm delivery, which
accounts for most of the increased perinatal mortality, neonatal morbidity, and long-term
morbidity of twins".

39 years old male patient present to ER with productive cough and fever.
Tempreture 39
O2 96
What is the appropriate management?
A-Moxifloxacin IV (dose written)
B-one of 4th generation cephalosporin IV (dose written)
C-meropen IV (dose written)
D-Pepracillin tazobactam 4.5 g every 6 hours
Answer: A

42 YO mother presented with paraumbilical mass, On exam you found


the mass tender, irreducible, and negative cough impulse. What is the
appropriate next step ?

- Abdominal Us
- Open repair

- Diagnostic Laparoscopy

- Biopsy

42. Child can ride tricycle , say his name , feed himself and separate easily from parents?
3years ?

43. Child has immature pencil grasp , knows colors , jump on one leg and say 6-10 words ?
?

564-Patient with PPH and she is known case of ITP , treatment?


A. Cryoprecipitates
B. FFP
Summary for ITP management (BMJ+UTD):
• Acute critical bleeding \ hemodynamic instability = triple therapy!!!
• Acute Maj(or) bleeding = steroid (OR) IVIG
• <30,000 – Steroids \ ivig
• >30,000 + bleeding – steroids \ ivig

96
**Platelet transfusion is the fastest way to increase the platelet count in critical bleeding
** IVIG will raise the platelet count within 1 to 3 days
** Glucocorticoids with raise the platelet count within 2 to 14 days.

49-8- (I think new question) Female comes with her new husband to the clinic. Her son from
previous marriage has sickle cell anemia. Now she asks if she has children from her new
marriage. How to know if they will have sickle cell? –

Chromosomal testing for the mother


– chromosomal testing for the father
– Hemoglobin electrophoresis for the mother
– Hemoglobin electrophoresis for the father ✅
a female came with bruises on face from her husband. What to do next?
a. reassurance
b. talking to husband
c. inform authorities
d. educate about violence

answer : c

a newborn was born and left abandoned by his mother . he was found to have inguinal
hernia. What to do ?
a. perform the surgery without consent
b. something from police
c. something from social worker
d. ask ethical committee

answer d

Pt. With hx pulmonary embolism 3 years want contraceptive drug


Other Q cardiac on anticoagulant drug=
A-Dermal patch
B-OCP
C-IUD
Answer: C

finding biopsy in celiac disease ?


subtotal villous atrophy

adult female smoker and obese did colonoscopy , found polyp in sigmoid colon which
removed and was beign adenoma , what is your advice ?
stop smoking and reduce wt

97
36 yo male k/c of crohns for 10 years presented to ER c/o abdominal pain , fever , vomiting
, and diarrhea , p/e there is abdominal tenderness , ct show 12x 15 collection and
ileo-jejunal fistula how to manage ?
a. laparoscopic drainage
b. percutaneous drainage
c. open drainage
d. open drainage with fistula resection

child with non bilious vomiting and olive shaped mass in the epigastrium , next step ?
a. npo
b. ngt
c. iv fluid
answer : c

57. Child came from Egypt and now presenting with headache , nick s9ffness,vomiting .
Positive Kernig sign.( No Lymphadenopathy ! ) LP showed Lymphocytosis but negative
bacterial culture. What is the most likely cause?
A. Polio virus
B. Corona virus
C. CMV ?
D. EBV

58. Pregnant came to the clinic and was found to have vulvar mass confined to the vulva ,
biopsy report should SCC. What is the most appropriate Management?
A.Chemoradiation
B. Vulvectomy
C. Wide local Excision

68-Question asking about the most common organism causing AOM in peds :
A-viral
B- Bacterial
C- Fungal

Answer: B

I personally chose analgesia ✅


69-Trauma pt with fracture of 3rd, 4th, 5th and 6th ribs. How to initially manage?

Flail chest treated with analgesia


But if there’s low o2 answer will change
Other recall:
1/ Patient with blunt chest injury resulted in fracture in 3rd, 4th and 5th ribs in more than one
site. What is the initial treatment ?
A. Intubation
B. Assisted ventilation
C. IV fluid.

No analgesia in the choices.

A- g a streptococcus
B- g b streptococcus

71-Qustion asking about the organism causing rheumatic fever:

98
Answer: A

Tic syndrome
Other recall

74-Case pt pedia recurrent blankish ?!

-An 8-year-old boy’s parents complain that he has episodes where he blinks multiple times
and becomes okay after that he is conscious and responsive during those episodes. The

A. Tics✅
most likely diagnosis?

B. Tourette syndrome.
C. Blinking disorder
Answer : A
87-Female with discharge and positive motion cervix adnexal what is Dx
A- CERVICITIS
B- CANDIDA

Answer: if A, the cervix would he erythematous, edematous, friable, possible visible


discharge, NO fever.

___

Another previous recall:


Lady with abdominal pain and I think vaginal bleeding. Exam she is febrile, with a normal
looking vagina and cervix no adnexal masses, and motion excitation. Dx?
A. PID.
B. Acute appendicitis.
C. Cervicitis.

Answer: A
PID will present with FEVER, lower abdominal pain, cervical motion tenderness (elicited by
examination), purulent or bloody cervical/vaginal discharge
Cervicitis -----> discharge only, no pelvic tenderness
Acute salpingo-oopheritis (PID) -----> discharge + cervical motion tenderness
Candida -----> doesn't cause PID

102-Women with palpitation, pulse rate 174 and vitally stable, what you will give?
A. Amiodarone
B. Adenosine
C. DC shock
Answer: Depends on type of arrhythmia …

if
Regular supraventricular tachycardia (SVT) of unknown mechanism
• vagal maneuvers &/or IV adenosine, if ineffective or unfeasible ⟶
• Hemodynamically unstable patients⟶synchronized (DC) cardioversion.
• Hemodynamically stable patients ⟶
• IV diltiazem or verapamil can be effective
• IV beta blockers are reasonable
• unresponsive to pharmacological therapy, synchronized (DC) cardioversion

99
Sustained ventricular tachycardia (SMVT):
• Hemodynamically unstable patients⟶ Urgent cardioversion.
• Hemodynamically stable patients ⟶
• lidocaine, may be more effective in the setting of acute MI.
• IV procainamide
• IV amiodarone
103-Pt with resp symptoms and labs have interferon >> answer is TB

104-Patient with psychiatric symptoms


and you decide to start neuroleptic medication. What is the side effect that can develop?
‫‏‬A-Hyperthermia
‫‏‬B-Myocardial infarction

✅ ‫‏‬C-Seizure
‫‏‬ D-Akathisia
105-Male patient came from India RUQ pain .. on and off fever for 3 weeks . raised LFT ,
high WBC (Neurtrophol 70% Lymphocytes 20%) . image showed homogenous hypoechoic
mass in the liver.
a) hydatid cyst
b) TB abscess
c) amebic abscess✅
d) pyogens abscess
Since he came from endemic area .. and the fever duration

107-Malaria prophylaxis in Southern ksa?


A-Mefloquine
B-chloroquine * (could be correct cut lately we reported resistance cases )

*best answer is Atovaquone 250 mg–Proguanil 100 mg (Malarone) one tablet daily (for the
one who will travel to south
note :
First line treatment of uncomplicated malaria: (ARTESUNATE + SP); alternative
(ARTESUNATE + MEFLOQUINE)
Second Line Treatment of uncomplicated malaria:: (ARTEMETHER + LUMEFANTRINE)
Third Line Treatment of uncomplicated malaria: (oral QUININE + DOXYCYCLINE)

110-What does variable deceleration mean? Cord compression


How to treat? Left lateral

111-Question asking the diagnoses >> Placenta abruption ( because of painful bleeding)

112-RTA with hypotension. Asking what is the cause of hypotension. The man can shrug his
shoulders but can't flex hands or move lower limbs.HR 70.>> spinal cord injury

116-How to know fetal weight intrapartum in 37 w?


A- femur length
B- head circumflex
C-Biparietal diameter
D- abdominal circumference
Answer= D

100
117-Post sleeve abdominal pain what to do?
Answer : CT

118-Pediatric patient brought by his parents; X ray showed widening of the ends of his
bones (growth plates): Calcium (low) PTH (mildly high) Alkaline phosphatase (very high)
What does the patient have?
A. Hypophosphatemia
B. Primary hyperparathyroidism

D. Renal osteodystrophy
Answer is: D

C. Vitamin D deficiency rickets

120-Dm pt on metformin with good glycemic control and with high liver enzymes, what to
do?
Stop metformin A couple of other options about changing it to some other drug I cant
remember

121-70 y/o male on MV in ICU due to intracranial hemorrhage, 7 days later he developed
ground coffee vomitus. What is the Dx?
A) Stress gastritis
B) H.pylori gastritis
C) Dyspepsia
Answer is A (coffee ground vomitus)

122-Female with breast mass and axillary lymph node birade 4 what to do
A-follow up
B-FNA
C-core biobsy ✅
D-excional biopsy
Answer C (BIRAD 4)
Stroke question where lower limb affected more >> ACA

159-t I think he known case of Alzheimer came to ER with severe agitation the doctor give
him a big dose of Haloperidol and he develops side effects I don’t remember what r they
exactly, what u should give him now?
They mean what is the antidote of haloperidol?
A-Naloxone
B-bromocriptine
C-Glycogen

158-The minimal time for exercise in child ?
30 min
60 min
100 min

120 min

157-Patient with pyloric stenosis and olive shaped mass, resasitation done, what’s the
definitve management?
Pyloromyotomy

101
154-Patient with erythema and joint pain in the morning, reynaud’s, malar rash,
photosensitivity and high RA factor, how to treat

⁃ methotrexate ⁃ Hydroxycholoquine

Seziure pregnant > review medications


Diabatic and hypertensive pregnant > review medication

155-Patient came with typical history of TIA, what to do? ⁃ mri

156-Pediatric with abdomen distention, what’s the intial imgaing?


⁃ US
⁃ Plain x ray
⁃ Ct

153-3 hours baby, with sudden hypertonia and seizure, resolved on its own, labs show
slightly decreased glucose 2.4 (normal 2.8 and above, all other blood tests was high (hb,
mcv… etc), what to do next?
⁃ bolus glucose
⁃ Iv fluid
⁃ Diazepam
⁃ Antibiotic

150-Pt with history of vomiting for 24 hours and before 3 hours it become hemoptysis =
A-Mallery wiess
B-Esophageal varecis

151-elderly Patient came for prevention clinic, what vaccine you’ll recommend? ⁃
pneumococcal ⁃ Meningococcal

152-Elderly asymptotic came for check up found o\e found murmur, valve gradient 40,
what’s your next step?
⁃ F/u
⁃ Surgery valve replacement
⁃ Valvuloplasty
147-patient with renal failure presents with low calcium, high alkaline phosphatase, and high
PTH. What is the best form of vitamin D to give this patient?

A- Calcitriol B- Vitamin D2 C- Vitamin D3

148-When to diagnose pedia as hypertension? A- bp > 120/80 B- bp > 150 C- >90%


depends age and sex D- >95% depends age and sex

149-Patient long scenario with bloating and loose stool and weight loss, biopsy shows
vellous atrophy, management?

⁃ gluten free diet


126-Old Pt with dysphagia everything is normal all labs are good but he only has weight
loss.
Urgent egd

102
Refer to gastro
Semi urgent Something

127-14yo vaginal bleeding every 3 weeks or 2 months with no symptoms of menstruation.


Everything is normal on exam she looks like a normal pubescent girl. (Question was hinting
that its itramenstrual bleeding i think) q didnt mention weight.
FSH , prolactin
US
Reassure

128-Down syndrome with constipation and weight gain investigation?


A-Tsh

129-Sever flank pain with hematuria and painful micturation


A- Ct with oral and iv contrast
B- Ct with out contrast
C- Xray
D- Us

131-Radiologist suggests something for the patient case. Who should take the patient
consent for the radiologist opinion?
-radiologist
-head nurse
-internal resident
-anyone

135-14 years old female came with painless spotting prior her period : OCP Reassurance

136-Manegment of postpartum bleeding :


Asking for the best step after giving 20 unit of oxytocin and RL boules :

137-Na+ level in dehydration patient : Low , high , not affected

138-Patient MVA with pelvic fracture managed with pelvic binder, what Fluid you’ll manage
her with:
⁃ albumin
⁃ RL
⁃ Hypertonic saline
⁃ Dextrose

141-Vaccination of 2 months

102- Pt with hepatic benign tumor dx is hemangioma (mentioned), decided to be


managed conservatively, but we should advise the pt to ?
A. Avoid contact sport and trauma
B. Loss weight
C. Smoking cessation

103
Answer: A

-Ortho case what type of dVt prophylaxis


A-ENOXAPARIN 40 mg SC
B-IM ASPIRIN
C-Heparin 10,000 U IV Fondaparinux 20 mg
Answer: A

144–HSP picture twice, one what dx and one what’s mx

145-Pregnant in labor given tocolytics, side effects? ⁃ palpitations

Pregnant came with a routine visit , US showed oligohydramnios , what is associated with it
?
- DM
- Deudenal atresia
- Placental insuff

Management of dka ?
-Reach 18 mmol
-Reducing 3mmol per1hour
-Reducing6mmolper1hour
-Aim to be above 14mmol

After complete h.pylori eradication how many weeks follow up?


-4week
-2week
-1week

Case painful soft plat and posterior pharynx and sparing of gingiva?
-Congenital something
-Aphthous ulcer
-Gingival cyst
-Herpangina

Case ‫ عن‬cushing ‫ويسال وش اسوي بال‬vaacine

104
primigravida with pain post-defecation pain and blood stool after
defecation, describe it as sharp and.. What is the Dx?
-Anal fissure
-Perianal abscess

Pregnant presented in 2nd trimester with signs of anemia + had severe vomiting in 1st
trimester. Labsshowed mcv 112, hgb 9. Dx:
- physiological anemia
-
- ✅
folate deficiency
b12 deficiency
- iron deficiency

Asthma exacerbation not responding to salbutamol, o2 and steroid.


1. Magnesium sulfate
2. Aminophylline
3. Albuterol
4. Not sure maybe intubation?

18 month Child came with shortness of breath.decrease air entry in right side ( forgot the
rest sorry) + they was a finding in chest x eay(forgot) parents tell you there is no prior
infections or hx of fever orchoking, past medical hx unremarkable:
1. Foreign body aspiration
2. Bronchiolitis obliterans
3. Asthma
4. Pneumonia

8 y.o patient immunization schedule unknown, came with fever and neuro symptoms (not
sure what they were) examination reveals bilateral tender and enlarged parotid glands and
pain with neck flexion:
1. EBV
2. CMV
3. Mumps

4. Measles

Duct ectasia birad 2


A- wle
B- reassure and f/u
C- mastectomy
Answer: B

Morbid obesity which is high diagnosis for porceder?

105
A-Barium sallow
B-Endoscopy
C-Ct
D-Us
Answer: D

Esonophilc esophagitis with?


A-Over weight
B-Snoring
C-Eating exsseive
D-Pain at night
Answer: C

Old patient with Left lower quadrant pain. Found large collection 11*_ and diverticulosis.
What todo:
A- sigmoidoscopy
B- laparotomy
C- percutaneous drainage
D- resection and anastomosis
Answer: C

6 days post abdominal surgery patient had obstruction. X-ray: multiple air fluid levels. Dx:
A- paralytic ileus
B- volvulus
C- adhesion
Answer: A

Lady post cholecystectomy presented with right upper quadrant pain. Found to have
collection in rightupper quadrant:
A- us guided percutaneous drainage
Answer: A

Gcs 2 questions. One to calculate the other calculate + which degree of impairment:
minimal, mild,moderate, severe

Patient after rectal surgery presented with sob. Sinus tachycardia in ecg. What to do?
A- d-dimer
B- ct
Answer: B

Patient with unilateral lower limb non-pitting edema. What will you give:
A- aspirin
B- heparin
C- warfarin
Answer: B

106
Female dm uti what’s contraindication:
A-Nitrofuntoin
B-Cipropfloxacin
C-Trimethoprium
D-Tazoco or meropenum not sure
Answer: A

Pt 60 new onset of ssptting minmal amount of bleeding from genitalia, What the source of
bleeding?
A-Tubal
B-Ovary
C-Uterus
D-Lower Genitalia
Answer: C

‫جا انو واحد كان يتابع مع دكتور وبعدين نقل مستش ىف ثانيه وزغللن يبا يروح لدكتوره اخ نتمعطيه كو ين من الميديكال‬
‫ريكورد‬

‫وجابو سؤال انو دكتور ن ش ميديكال ريكورد لبيشنت فاش الم نر‬
A) Reportable disease
B) Hospital want to share information in social media
C) family member want the records or ask for
Answer: A

Child has recurrent otitis media and pneumonia and chronic diarrhea, What to order
A-Immunoglobulin screen
B-Forget other choices
Answer: A

Pregnant iugr what appropriate next step


A- Nst
B- Serial us
Answer: B
Copd has minimal edema reach to ankle Pco2 7.1
Po2 8.6

Pt not child alert with shallow breathing


A-Tracheostomy
B-Cricothyroidotomy
C-Orotracheal intubation
D-Nasotracheal intubation

107
Answer: D

Pt after MVA has severe pain in legs compartment pressure 35


Pt with severe pain and numbness no posterior tibial or dorsalis pedias pulse Not sure but i
think has fracture
Faciotomy

Pregnant with placental abruption what indicate ?


A-Urine output
B-Hypotension

Apoendicular absscess what’s the pathophysiolgy


A- Redisribute blood supply
B- vasoconstriction
Answer: A

High tsh and normal t4 and high cholesterol, What treatment for dyslipedmia
A-Statin
B-thyroxine
Answer: B
Microcytic anemia with target cell and inclusion bodies what’s the Dx?

No mentaly issue but tall and thin limbs?


A-Kkinfinter
B-Ehler
C-Marfan
Answer: C

Psychiatric sympotoms and pt has weight gain,So what’s the appropriate treatment
-Olanzipine but wrong
-and other medication forget them

Pt with MR ( written severe rhumatic mitral regurge) and sx of HF, Pt on lazix ,


spironolactone,ACEI, BB, what’s appropriate for managment?
A-Digoxin
B-Mitral valve replacement
Answer: B

108
Patient diagnosed with cancer ,doctor prepare to break bad news.who should the patient
bring withher?
A-Parents and closed friends
B-husband and her kids
C-no body
D-any one she want
Answer: D

Athletic male gain a 20kg in 4 week, good musculature he came for screening clinic what
screening testwould you do to him?
A-Anabolic steroid
B-TFT
Answer: A

Patient developed SOB and chest pain after long flight, Ex:hyperresonance chest with
decreasebreathing sound ,medical and surgical Hx clear, he is TALL
VS:hypotensive
What is your Next Mx?
A-Needle decompresion
B-chest tube
C-I can’t remember the others choices
Answer: A

Dr treat famous football player and he put his data in nurse station,then someone come and
take a picture of the player data and post it in social media something like this, The court is
suing or the mistakeon:
A-the dr who forgot to sign out as he plied to maintain confidentiality
B-nurse who didn’t checked after dr
C-no one because the patient is famous
D-the one who take a picture
Answer:A

Pregnant came with Colicky abdominal pain and bleeding with some tissue pass through
cervix
A-incomplete
B-Complete
C-inventible
D-Threatend
Answer:A

109
50 y/o (I think) post surgery (I think it’s gastric sleeve) by 5 days developed obstruction Sx
(vomiting, bloating and abdominal pain) when they were going to push the Pt for CT they
noticed in the NG tube coffee ground vomitus and fresh blood, what will you do?
A- CT abdomen
B- Upper endoscopy
C- Colonoscopy
Answer: B

Pt known to have MR from Rheumatic heart disease came to the ER with dyspnea,
orthopnea and PND, she was given medication (I think ACEi and diuretics) and her Sx
improved.
ECHO:
Severe MR, EF=45 What will you do? A- follow up
B- MV replacement/repair
Answer:B

BETHESDA IV/4 ?
A- Left thyroidectomy/Hemithyroidectom
Answer: A

Female wants to conceive and it’s winter, what vaccine will you give at preconception visit ?
A- Influenza
B- Rubella
Answer: A

2.5 y/o boy keeps yelling “no” (and some other similar things) what will you advise the
parents?
A- Ignorance counseling
B- Positive reinforcement
Answer: B

Which of the following is the most important medication regarding asthma exacerbation?
A- inhaled SABA
B- Systemic steroids
Answer: A

‫اذكر جا ى ين سؤال‬
‫ يب ىغ‬granuloma caseating with LN Lung diagnosis ‫ لكن ما اذكر وش الخيارات ال يل اذكره ان فيه‬TB

110
MVA patient with decreased air entry in left hemithorax& was tympanic on percussion (+
wasunstable) what’s next:
A- chest tube
B- needle decompression
C- intubation and ventilation
D- x-ray
Answer: B

Patient was treated conservatively for appendicitis then presented with appendiceal mass.
Next?
- appendectomy after 12weeks
- colonoscopy after 6 weeks
Answer: B if Old
A patient presented with an infection. Which one of the following diseases the doctor need
to report it to ministry of health?
A-Chlamydia trichomatis
B-Bacterial vaginosis
C-Lobar pneumonia
D-Infectious mononucleosis
Answer: A

‫مرض اختفى بسبب التطعيمات بالسعودية؟‬


A- influenzaHِ B- fever Q

Pt underwent rt parathyroidectomy due to hyperparathyroid came with muscle and bone


ache and headache (signs of hypercalcemia)
Labs showing:
PTH high
Ca slightly increased What the cause of this?
A-missed Adenoma
B-new Adenom
C-parathyroid metastasis D-parathyroid ca
Answer: A

- ‫ كبير بالسن كان عنده‬A.fib ‫ وجاء ب‬pain abdominal sever ‫ايش نطلب له؟‬
A-CT
B-MRI
C -US

111
Answer: A

Reassuring biophysical profile ?


A-7/10
B-8/10
C-9/10
D-10/10
Answer: B

Child w/ low grade fever and inspiratory strider?


A-Acute trachitis
B-Acute epiglottis
C-Larngeotracheobronchitis
D-Acute asthma
Answer: C

PID treated by cefutaxim but not response what is the couse?


A-hsv
B-adenovirus C-chlamydia D-nessiria
Answer: C

Child brought to the primary health care clinic for regular assessment, he laughs and when
his mother put him on the bed and the doctor went to examine him he reached for his
mother, how old is the child in months ?
A- 2
B- 4
C- 6
D- 8
Answer: C

70 Y.O male complain of lower abdominal pain and desire for urination and has a history of
progressive urinary tract obstruction most likely diagnosis
A- prostatic cancer
B- BPH
C- UTI
Answer: B

Vaccine that can be given throughout pregnancy (influenza)

Mediator that is responsible for hot water burn edema


A- prostaglandin
B- bradykainin
C- serotonin
Answer: A

112
uterotonic drug contraindicated in asthmatic patient?

Lung LN with caseating granuloma otherwise normal,, management;


A- observe?
B- steroid
C- ABx
Answer: C
A question about a young girl (7-9 years) her mother brought her because she was worried
about her developing genital hair , but no muscles enlargement, no breast development or
axillary hair and thegirl is normal socially and interact with people
On examination there is increase height, Dx:
A- Turner syndrome
B- congenital adrenal hyperplasia C- premature adenarche
Answer: C

Most important initial investigation for PE? A- D-diamer


B- VP scan
C- Doppler US
Answer: A

5 months child her mother has concerns about her development reassure her by saying?
A- child can reach and object B- sit without support
C- hand grasp
Answer: A

Drug of diabetes
1st thing to give >1st biguanide metformin If obese? > metformin
How to diagnose by HgA1C

7 months old boy presented with history of interrupted feeds associated with difficulty in
breathing and sweating for the last 4 months. Physical examination revealed normal
peripheral pulses, hyperactive precordium, normal S1, loud S2 and Pansystolic murmur
grade 3/6 with maximum intensity at the 3rdleft intercostal space parasternally. The MOST
likely diagnosis is:
a) Small PDA (Patent ductus arteriosus).
b) Large ASD (Atrial septal defect).
c) Aortic regurgitation
d) Mitral regurgitation.
e) Large VSD (Ventricular septal defect).
Answer: E

113
10- old female e back pain, relieved e leaning forward on walker and walking uphill,
Peripheral pulsesare intact
Answer: spinal stenosis

How to diagnose celiac


A-Anti-endomysial antibody
B-tissue transglutaminase antibody
Answer: B

Prognosis of schizophrenia ?
A-5% remission
B-33% reduction of symptoms
C-70% satisfied with their life
Answer: B

Long case child complain itching his eyes and ..nasal congestion on examination there is
periorbital swelling and enlrged mucous turbinate
A . allergic rhinitis
B. rhinitis medicamentosa
Answer: A

what congenital anomaly could be associated with oligohydramnios?


Answer: renal agenesis

Female with fishy-smell discharge, analysis: PH: 5.1 What type of cells will you see?
A- Multinucleated giant cells
B- Overproduction of lactobacillus
C- Single nucleated cells Review
D- Granulated epithelial cells
Answer: D

Mother came for antenatal care and US shows week 32 reversed end diastolic blood flow:
A. Follow up 2 week and reassess
B. Immediate delivery now
C. Administer steroids 1 week and delivery
D. NST
Answer: B

post roux-en-y developed shoulder pain whats the initial step of management ? A-CT with
Contrast
Answer: A

114
Patients with bleeding peptic ulcer and history of long use of aspirin. What is the managent?
A-High dose oral PPI BID
B-IV PPI for 24hr followed by oral PPI
C-IV PPI for 48hr followed by oral PPI
D-IV PPI for 72 hours
Answer: D

Calculate the GCS score of someone who opens his eyes to painful stimuli, says
incomprehensiblesounds, and flex his arm due to pain
A- 8
Answer: A

Medial ulcer, what is rhe risk factor?


A-DM
B-Atherosclerosis
C-Venous HTN
Answer: C
‫ اذكر مر ع يل بس‬0.3 ‫ او‬0.6 ‫ىي كم التارقت مفروض ي ى ىنل السكر بالجسم ى يفالخيارات‬ ‫جا ى ين سؤال عن االنسول‬
‫مالقيته يوم بحث عنه‬

7 girl , what the Antibiotics for uncomplicated acute cystitis ?


Answer:oral amoxicillin

Female come with multiple green discharge no mass , and do us + mammogram There is
dilatation in duct and unspecious leaion What to do ?
A-Glactogram
B-F/u 6m with multiple images
C-Mastectomy
ABnswer:

Copd and cor pulmonale , Which of the following Improve mortality


A-Multaknin
B-Inhaler steroud C-LABA
Answer: O2 therapy

-Pt has fever and didn’t eat for 1 day BP : 64/42 ‫متأكد‬
A- discharge the pt
B- admtion for observation fever and discharge if become afabril
C-advice the mother to come back if happen again
D- admission for Iv antibiotic and culture
Answer: D

115
female with suprapubic pain with purulent discharge . Vaginal Ex tenderness in fornix
A-Acute cervicitis
B-Acute salpingitis
C-Acute appendicitis
D-chronic appendicitis
Answer: B

Open eye to the pain . Incomprehensible sound . Flexion to pain


A-5
B-6
C-7
D-8
Answer: D

Sep 27 – Part 1

-64 pt admitted to hospital because of pneumonia, she K/C of end stage renal disease , “ no
thing about surgery “ vital stable , what prophylaxis will you give pt ?
A- enzoparin
B-UFH
C- founoparix

Pt with absent distal pulses, palpable popliteal pulses, diminished sensation, altered motor
response, diagnosed with afib
A. Upper knee amputation
B. Thrombolytic therapy
C. Femoral artery embolectomy
D. Heparinization and observe
Answer: C

Trauma Pt with extra peritoneal bladder injury?


A) A-Suprapubic catheter
B) B-Urgent exploration and repair
C) C-Catheter repair and assess after 2 weeks
D) D-Catheter us after 2 weeks
Answer: C

Sandpaper rash what is the organism?


A- streptococcus varidans
B- streptococcus epidirmidis
C- streptococcus pyogenis
D- staphylococcus
Answer: C

116
Child with chlamydia diplococci (conjuctivitis etc.) Asking about the causative organism

Why is acei CI in pregnancy


A-Drop BP very low
B-Cause congenital malformation
Answer: B

How to know fetal length intrapartum


A-Femur length
B-Head circumflex
C-Biparietal diameter
D-Abdominal circumference
Answer: A

250-200 hr narrow complex irregular whats the arrhythmia


Answer: AF

Perkland formula
Fluids in a case of both legs burn

Q about baby with jaundice and fever, in the end they said he has mideterranian fever, what
is the drug ( Amoxacillin).

Q about eldrly with Afib came with severe abdominal pain, what to order?
A-CT
B-MRI
C-US
Answer: A

child come to well baby clinic , the Dr decided to examine him in proper position : faceing
the wall , bending forward , drooling hand with uncontrolled , what is the Dr looking for ?
A- scoliosis
B- CF
C- nutritional assess
Answer: A

middel age female come with heavy menusturation and abdominal fullness for 2 years (bulk
symproms) on examination upper pelvic tenderness , US showes Fibroid. what is thy
subtype of fibroid ?
A- submucosal
B- subserosa
C- cervical fibroid

117
Answer: A

women (forgotten age) have a history of genital warts and now complaining of bright red
scanty
bleeding also moderate itching, what is the most common site of this bleeding ?
A- vaginal
B- uterus
C- cervical
D- vulvar
Answer: A

Patient 28 y had MVA with no obvious external bleeding presented with decreased air entry
in let side. And hypotensive tachycardia tachypnea and tachycardia most appropiate
management ?
Thoracotomy
Neddle thoracostomy
Thoracic tube insertion
Pericardiocenthesis

c since (3yrs I guess ) her father has colon cancer Her mother has breast cancer What she
should do regarding screening?
A -Mammo at age of 40
B - Colonoscopy at age of50 and -mamo at age of 40
C- Should do colonscopy after 8or 10 yrs after her dx of U.C

Pt with hepatitis C + liver cirrhosis treated , - ve HCV RNA, what to do next?


reassurance,F/U
B- FU with US
C- liver biopsy

case with ecg finding : Dx Anterior MI


ECG : heart block - mobtiz 2 Tx;
ECG : third degree heart block

24- A 45 yrs old female P6. Presented with Chronic pain and heavy menstruation. Imaging
shows:
Thickened myometrium. What’s the most definitive management?
A. Hysterectomy
B. Combined OCP
C. D&C

37-An or 25 y/o female worried about cervical cancer. She took her first dose of HPV
vaccine 3 months ago. What the best thing to do at this visit today?
A-Schedule app after 3 months
B-No need to do anything at this visit
C-Give 2nd dose at this visit

118
D-Repeat 1st dose

Most common cause of interstitial pneumonitis?


- Lobar pneumonia
- Bronchopneumonia
- Viral pneumonia
- Secondary TB

Hb screening in pediatrics
- 6
- 10
- 12
- 14

12 old female diagnosed as Beckwith-Wiedemann syndrome , Dr told the parents that


is highly
associated with other neoplasms such as liver cancer , whats will you order to
screen for her ?
- PSA
- AFP
- CEA
- B-HCG

63 years old male medically free complaining of right knees swelling And pain
Examination revealed right knee pain erythema , on examination there is right knee
effusion and redness , on x ray (left) knee shows osteophyts and narrowing space,
no labs , whats your diagnosis ?
- gout
- septic arthritis
- osteoarthritis

known case of systemic sclerosis and daibetic came with fatigue and abdomial
discomfort , labs shows AKI (very high Cr) , whats your tratment ?
- lisinopril
- amliodipine
- hydrochlorothiazide
- bisoprolol

child diagnosed with pertusess and admitted to hospital to start treatment , he has 2
siblings at home one ate age of 3 y and other at 5 y , that is the best regarding
asymptomat sibling prophylaxis ?
– booster vaccine now
- booster vaccine in they are high risk children
- start macroliods
- close observation

119
COPD acute exacerbation not responding to bronchodilator (written like this), no any
vitals or investigation provided, whats next?
- IV steroid
- IV thiotropium
– CPAP

Female with bone pain that goes away when she drinks milk High ca Low phosphate
High PTH
A- 1ry hyperparathyroid
B- 2ry hyperparathyroid
C- milk alkali

Known case of something ( iforgot) came with peptic ulcer perforation and need
urgent surgery Inr :2 Plt 90 Hgb 90 What to do first ?
- Cryopreciptate
- FfP
- Plt transfusion
- PRBC

Question about DM incidents and prevalence after insulin


- Decrease prevalence
- Increase prevalence
- Decrease incidence
- Increase incidence

Pediatric 18 m introduced cow milk at 9m after stopping exclusive brest milk came
with soft stool
- cow milk allergy.
- The rest irrelevant

The rest irrelevant No repeat epinephrine and no laryngeoscope in choices


Which age infant more dependent on his mom ?
A)4-6 week
B)6 month
C)12 months
D)9 month

50 years old c/p of sever bleeding failed medical mangemnt UC show fibrod 7*3 What
is the definitive treatment ?
- Hystrectomy
- Utrine artry embolization

Couple try to convce 6m and didnot work women has reguler period 4 day What is
the next?
- Serum TSh
- Prolactin
- Urinary Lh/Fsh 21
- Progesterone

120
6 days old infant , had sob , dyspnea poor feeding , his mother was diabetic . Dx?
A- ARD
B- Bronchitis

50s female with 2nd degree vaginal prolapse, urgency , incontenance. Tx 4- pregnant
with polynephritis symptoms . High urine wbc . Tx?
A- oral abx
B- admission for iv abx
C- renal us

45 women , smoker came for cancer screening . medical, surgical , family hx : -ive
- mammogram
- colonoscopy

female with 5 depressive symptoms for 2 months:


A) major depression
B) minor depression

Scenario about a 60 year old male with DM controlled , had 3 readings of 149/90
A- add ACE
B-no need to add medications
C- add BB

Anal fissure surgery:


- lateral internal sphincterotomy
- lateral external sphincterotomy

Lung nodule 7 mm what to do? Follow up after 6-8

23 y/o come with fresh bleeding per rectum in imiage found the lesion near to
iliocecal vavle ,Dx?
- Crohns
- PU
- meckel diverticulum

Dm pt come after 4 h with LT side paralysis (seems like stroke) what best next step?
-Mri
-Ct
-Aspirin
-Clopidogrel

Psychiatric case about a patient with echolalia, echopraxia and bad hygiene for some
months. Patient is oriented. How would you treat?
A Lithium
B Ox something zine
C Other drug D Other drug

121
Patient with UC exacerbation on medications 6 bloody diarrhea per day and
abdominal pain. Culture showed C. Difficile How would you manage this patient?
A IV Cefuroxime
B IV Metronidazole
C Oral Vancomycin

patient underwent utrine surgery and in the report endometrial was entered, rate of
placenta accreta is:
A-increased
B-decreased
C-unknown
D-not affected

12 year old child presented with vesicular rash over the trunk and body
A - Mumps virus IgM
B - HSV-1 IgM
C - HSV- IgM
D - VZV IgM

2 year old with diarrhea, distension, growth delay, pallor and buttocks wasting. What
is the most
approximate investigation?
- Dudodunal biopsy
- Endomyseal Ab
-Colonoscopy
-CBC

4 year old, known case of febrile seizures. Contracted febrile viral illness 4 days ago,
now presenting with lethargy, vomiting but no change in bowel motion. Mother is
anxious about the seizures so she gives her child paracetamol every 4 hours. On
examination, jaundice with palpable liver. What is the likely diagnosis?
- Paracetamol induced hepatotoxicity
- Acute viral hepatitis
- HUS
- Viral autoimmune hemolysis

2 day old infant came to hospital with complaints of seizure and decreased feeding
since yesterday. Inactive child with generalized increased muscle tone. CSF analysis
is normal. What is the most likely diagnosis?
-Hypoxic ischemic encephalopathy
-Neonatal sepsis
-Neonatal tetanus
-Pyogenic meningitis

Pt who have recent travel history to india c/o Right upper quadrants pain US show
hypoechoic mass?
-amebiasis
-hydatid cyst

122
PT with Crohn which of the following increases risk of malignancy?
a. Cancers located within 15 cm from the anal verge are
B.Primary sclerosing cholangitis
c. duration of 3 years

Pt k/c IHD comw After lap chole feel chest pain + lab show hypotension, what inx ?
-Ecg
-Cta
- Cxr

Fetus 180 HR , early decerelation And other information, and ask about which one
indicate severity I think:
-HR
-early decerilation

Baby born with microcephaly (forget 2ed one but looks like CMV ) ask about next ?
I choose congenital infections screening

A 60-year-old man is admitted to the Coronary Care Unit with an acute myocardial
Infraction. His hemodynamic parameters 2 hours later are: Blood pressure :80/50
mmHg Heart rate 40 /min Oxygen saturation °C 95% on room air Which of the
following would be the appropriate management?
A. IV0.6 mg atropine sulphate
B. Normal saline infusion
C. IV isoproterenol
D. IV dobutamine

Pt came from jeddah with fever and rash and headache what is the Mx:
A-Antibiotic
B- Steroid
C- Supportive management

Patient came to the ER with signs and symptoms of myocardial infarction he was
going for PCI, when the cardiologist was assessing his condition he notice the
patient was depressed with low mood, the patient refused the PCI he demonstrate
good understanding of his problem to the doctor what to do ?
A. Treat the patient regardless of the consent
B. Refer the patient for psychiatric assessment then take the consent
C. Respect the patient choice after discussing the reasons of refusing
D. Take the consent from the patient relative

year-old child came with fever and vesicular rash on the palms associated with
painful mouth ulcer. The majority of his classmates have the same issue. What
additional findings you’ll see in this patient?
A. Enlarged congested tonsils
B. Rash on the soles

123
Pt with painfull skin lesion in chest and Crusted how to dx :
A- swap from ulcer
B- blood culture C-skin biopsy

Pt diapetic with pneumoonia came with parapneumonic effusion what will made poor
prognosis:
A-Dm
B-high wbcs
C-od pleural effusion

Baby 29ws born with distress , most appropriate management:


- CPAP
- Surfactan

ECG : third degree heart block

Scenario of Inferior IM (Given the dx) then showed ECG like this and asked
what’s the dx?
-Heart block 1
-Heart block 2
-Heart block 3
-Heart block 4

Milestone: 5 years

Vaccines: varicella how many vaccine recommended for pt with no


previous Hx of infection with herpes zoster?
2 dose depend on the age
> 7 4wk apart
< 7 3 M apart

Another recall
14 yo did all vaccinations except variella, he never had chickenpox what
method of vaccination should be used ?
A-Don’t give vacc
B-1 dose vac
C-2 doses 4weeks

A patient coming from an endemic area for malaria. On Quinine(or


Quinidine?!). Later on, he was admitted to the hospital with Heart Failure.
Doctor did not ask the patient about travel or medication hx. Thus, he
ordered the nurse to give digoxin. The patient went into deterioration. How
could it be prevented?
A. Computerized drug prescription system.
B. Nurse failed to write drug formularies.
C. Pharmacist who failed to check prescription.
D. Patient who did not bring his medicine pack/ report his medicines.

124
6 days old infant , had sob , dyspnea poor feeding , his mother was
diabetic , What’s the Dx?
A- ARD
B- bronchitis

80 years old bed ridden has pelvic organ prolapse appropriate management?
A.Pessary
B.Le forte colpcelesis
C.Others

pregnant with polynephritis symptoms . High urine wbc . Tx?


A- oral abx
B- admission for iv abx
C- renal us

pt for surgery didn’t stop warfarine and nurse fine out , so surgeon ?
A- near miss

surgeon manipulation data :


A- falsification

Child , Abx for uncomplicate cystitis

child with typical symptoms of DM( polyuria, thirst, ..) on lab: the random
blood glucose was normal. ask about the cause of symptoms
- Decrease renin
- decrease insulin
- increase insulin
- absent of glycogen

Patient with MALT lymphoma and have urea breath test positive what to do?
A- ABx to eradicat H.pylori
B-no need

Pt with neck swelling move with swalloing . In US the mass was solid and
2.5 cm What is the most appropriate investigation
A- thyriod scan
B- FNA
C- Ct of chest
D- TSH
hours associated with neck pain,

70 Y.O male complain of lower abdominal pain and desire for urination
and has a history of progressive urinary tract obstruction . High psa. most
likely diagnosis
1- prostatic cancer
2- BPH
3- UTI

125
Pt with hepatitis C + liver cirrhosis treated , - ve HCV RNA, what to do
next?
A- reassurance,F/U
B- FU with US
C- liver biopsy

Pediatric with abnormal movements + tea color urine+ piruritis + lower


limb edema + slightly high blood pressure
A- HUS
B- ITP
C- PSGN
D- Forget
No TTP in options

Which age infant more dependent on his mom ?


A)4-6 week
B)6 month
C)12 months
D)9 month

Croup s&s was giving epinephrine Whats next ?


I chose steroid .
The rest irrelevant
No repeat epinephrine and no laryngeoscope in choices

Known case of CODP came with HF s&s What to do next


Echo
Spirometer
I forgot the rest but irrelevant

Question about DM incidents and prevalence after insulin


Decreaseprevalence
Increase prevalence
Decrease incidence
Increase incidence

50 years old c/p of sever bleeding failed medical mangemnt UC show


fibrod 7*3What is the definative treatment ?
A-Hystrectomy
B-Utrine artry embolization

48 YO Male with colicky abdominal pain in his left flank with blood in
urine. IVP: filling defect in renal pelvis
US: acoustic shadow
Most likely diagnosis:
A-Uric acid stone
B-Sloughed renal papilla
C-Renal tumor
D-Blood clot

126
Patient presented to ER with history of 3 day fever and cough and rash
all over the body. There is consolidation and crepitation.
What is the treatment?
⁃ Antibiotic ⁃ Acyclover ⁃ Antipyretic

There is alot of Q about atypical pneumonia and it’s treatment

A neonate was found to have a generalized petechial rash and intracranial


hemorrhage immediately after delivery. His Hb is low, Platelet is 4, and all
other labs are normal. The mother has a normal platelet count. What is the
most likely diagnosis?
a. ITP
b. Alloimmune thrombocytopenia (NAIT)
c. Congenital thrombocytopenia d. Congenital CMV Infection

Child knows 4 colors and say 6-10 words ? 4 years.

Child Climb stairs walks well and points 1-2 body parts? 3 yrs

Male with gonorrhea , what is the BEST procedure to diagnose him:


Urethral swap

Pt with abdominal pain patient stable, imaging shows: increase thickness


of sigmoid and thumb print due to edema. What is the next step?
A/ Colonoscopy
B/ Diagnostic laparoscopy
C/ Exploratory laparotomy

12 years old found to have klebsiella with in 100,000 colony from mid
stream urine, no sign or symptoms
1- no need for treatment
2- treat as acute UTI
3- give ABx as prophylaxis 4-repeats culture

Newborn vaccination? Bcg hbv (no hbv alone!) Others were wrong

91 yrs old in a residential home for geriatrics no drug history was given
came with 24 hrs of severe abdominal pain on examination there was
generalized nonspecific tenderness no bledding no vomiting most
appropriate initial thing to do?
Colonoscopy
Sigmoidoscopy
Digital rectal examination

Case picture open fracture of hand and whats the most immediate thing
to do?
Irrigation analgesia blah blah

Symptoms of bone abnormalities and bowing of legs what vitamin


deficiency? D

127
Case Bethesda 4 what to do? Hemithroiectomy

Case birad 0 what to do? Additional Imaging

What drug cause patent ductus arteriosus?

Mother with face bruises came to the ER with her child multiple bruises all
over the body what will you do :
A - call the father to take consent
B - Ask the mother to call the police
C - admit the child for treatment and call child protection services

Long case about uncontrolled DM


Bp in sitting 168/90 I didn’t remember exactly but was very high then another
reading on standing 115/60 what’s the must likely complication
*Autonomic neuropathy I think

- A Case of endometriosis (dx given) and you are asked what is


considered an acceptable diagnostic method?
- Us
- Laparoscopy
Both are given and the questioner didn’t specify what answer is wanted , an
initial or confirmatory modality, just asked about an acceptable one!)

A young man fell on an outstretched hand and there was pain at the
anatomic snuff box:
A.Hamate fracture
B.Scaphoid fracture
C.Colle’s fracture

A case of dysphagia and other related s/s, there is a barium swallow


report that would lead to you to the answer:
Report: ( dilatation of the esophagus with loss of peristalsis and the lower
sphincter won’t relax) No picture was provided.
- GERD
- achalasia cardia

history of intermittent claudication absent pulse in the right limb

A patient came with lower limb pain and no pulses in right limb while the
pulses in left limb are intact How to differentiate between acute and chronic
of this case ?

Child with widening of wrist in x ray


-hypophosphatemia
-vit d deficiency
-renal osteodystrophy

128
Case female with sudden unilateral blindness then resolve ? TIA

1- 4 year old Child with recurrent chest infection, inspiratory crepitation


and expiration weezing, poor growth and fatigue, finger clubbing, on further
examination he has offensive greasy stool, what will you find on further
examination?
nasal polyp
inguinal hernia

Child presented to the ER with his parents as they were saying that their
child and just said a potentially toxic Medications what is the next step in
management ?
A- Active Charcoal
B- gastric lavage
C- Ipecac Syrup

Postop fever and other signs and symptoms, surgery were before 4 days?
1. Atelectasis
2. Pneumonia
3. UTI
4. SSI

type of the estrogen in pregnancy ?


A-Estriol
B- estradiol
CD-
Esstron

old age has sever abdominal pain just above umbilicus just say like that
+ Vital sign i think he is mentioned it and there is no significant
What is the test needed to be done?
-Amylase
-ABG -

Q about nurse has Positive HCV but, Negative AntiHepatitis C DNA and
there is no risk factor mentioned and free from symptoms and he didn’t take
any medication. So What to do?
A.Repeat HCV serology “something like that”
B.Reassure nothing

Child with dark urine and pain , in lab there is positive nitrate.. What first
investigation to order?
-Urine analysis
-I think US

Obstructive jaundice and septic cholangitis , Which will be detected?


-Increase cardiac output

129
Febrile neutropenia >> Tx ?

Old age with bleeding . What next investigation?


Biopsy
US

Intrabroncheal biopsy for 52 y/o , last menstrual 12 month ago.What is the


responsible for this condition?
-FSH
-Estradiol
-progesterone
-testosterone

Vaccination at 6 year

Milestones well trained towlet and can tie his shoes? 5 years

Most associated with weight gain


A- Olanzapine
B- colanzapine

Pediatric asthmatic, good control but not improve, his mother complain
about baby weight is low compared to other children at his age
Examination :chest normal
Weight and height below centile:
A- screen for other dx mimking asthma Others are irrelevant

Which type of heart block associated with inferior wall MI?


A. 1st degree
B. . 2nd degree
C. . 3rd degree
D. . 4th degree

Answer: Inferior MI — Conduction disturbance in inferior MI may occur upon


presentation or after hours or days. Sinus bradycardia, Mobitz type I (Wenckebach), and
complete heart block inferior mi ST segment elevation in the inferior
leads (II, III, and aV)

man obese with somelence and fatigue + BMI high What is the investigation?
A ECG .
B- Echo,
C- Chest X-RAY
D- sleep study

Female patient with ovarian tumor and during remove it the nurse notice
thick secretion coming out from the ovary, what’s the type of this tumor ?
A- Thecoma
B- Teratoma
C. Fibroma
D. Granulosa

130
how to remove the placenta if it takes more 30 minutes after vaginal
delivery ?.
A) Manual removal
B) Wait
C) CS

How to differentiate between crohns and UC?


A. Extra intestinal manifestations
B. Non caseating granuloma
C. Apthous ulcer

Long case about pcos with hormon profile and ask about further inx
1-testosterone level
2-glucose intolerance and lipid profile
Answer is 2

Female pregnant GA 20 something present with complain of uterine


contractions CTG done showed acceleration and 4 contractions in 10
minutes, what is the appropriate management?
A. Reassure and discharge
B. IOL
C. Check cervical dilation
D. Tocolytics (not sure if this was an option)

55 years old female menopause present with heavy vaginal bleeding


associated with weight gain BMI 35. TSH done was normal
Endometrial biopsy showed adenomatous hyperplasia
What is the pathophysiology?
A. Adrenal hyperplasia
B. Peripheral conversion of precursors to estrogen
C. Genetic mutation of endometrium

6 Years old child with nausea, vomiting, headache & visual disturbance
worsening over the past 6 weeks.Symptoms provoked at early morning &
late night, and improved by walking.
What is the diagnostic test?
A- Brain MRI.
B- Abdominal US.
C- Muscle biopsy.
D- CT (i forgot where).

Long scenario about a female with celiac and iron deficiency on iron
ferrous 325mg but still complains of symptoms what would you do?
A. Mix in oral solution
B. IV iron
C. Divide dose three times

131
Most associated organizm with vesicular rash on trunk and upper limb -
HSV1
-HSV2
-VZV

17 years old female present with jaundice and upper abdominal pain, she
is single, does not take any medication, not smoker, does not drink alcohol or
a drug user, her liver function is elevated, what is the most appropriate test
should be ordered first?
A. HAV IgG
B. HAVIgM
C. Hepatitis
B surface antigen
D. anti-HCV antibodies

Child came with mass on right quadrant (not reach the medline )not
mentioned?
nephroblastoma

what is earliest manifestation in fructure something like that ?


-pain
-cold
-parasthesia
-hottness or redness

Case of hydated cyst ask about management ?

30 year old female came to the ER with abdominal pain LMP was 6
weeks ago. US shows abundant fluid in pouch of douglas. What to do ?
A. Laproscopy
B. Terminate pregnancy
C. Drain fluid

Elderly male with epigastric pain that started after a meal and became
“tearing”, with diaphoresis ( vitally stable) . Next step in managment
A. Diagnostic laparotomy
B. US
C. CT

Child with signs of epiglotitis. Whats the organism ?


A. Hib
B. Streptococcus

Newborn with respiratory distress and pulses of upper and lower limb are
different. What to do?
A. Surgery
B. Prostaglandin infusion
C. Epinephrin infusion

132
Pediatric had fever now comatose CSF shows high wbc, high protein, normal
glucose
A. TB meningitis
B. Viral meningitis
C. Diabetic coma

RTA what would you most likely see ?


A. Hollow organ perforation
B. Fluid in peritonium

4 year old child with soft spongy 10 cm left thyroid mass. FNA showed lymphatic
fluid?
A. Surgery
B. Sclerotherapy
C. Chemo

50 year old with family history of breast cancer came with right upper
outer mass, atypical ductal hyperplasia. What to do?
A. Simple mastectomy
B. Wire localization excision

Patient DM HTN with lower limb edema What is causative drugs


-Ramipril
-Amilodarone
-Verpamil

Girl referred to immunology to investigate if she may have an


immunodeficiency. Which vaccine to defer ?
A. Hep b
B. Dtap
C. Hib D. Bcg

PPH questions asking what to give


A. Vit k
B. Platelets

32 weeks pregnant 7 cm dilated sudden loss of 700 ml of blood. What


type of hemorrhage
A. Intrapartum
B. Antepartum
C. Postpartum

30 year old endometriosis on nsaids with no symptomatic relief. Next


most appropriate
A. Combined ocp
B. Laprasopic fulguration

133
Case about child with fever and sore throat with tonsilitis what is the
diagnosis
-kawasaki
-scarlet fever

Long scenarion Elderly complaining of lower limb weaknes and urinary


incontinance What is diagnostic method?
-mri spine
-lumpar puncture
-mri brain

What is chemical mediators in hot burn within 24 hours


Bradykini
Prostaglandin 2
Hyperreflexia

65 years man presents to your clinic and looks weak , dehydrated, pale ,
thin and emacitaed. he complains of anal itching , discomfort from the pas
few months. On examination, you find an anal mass that is 2 cm away from
the anal verge , cauliflower like and friable. What is your most likely
diagnosis?
A- Anal Cancer
B- Rectal Cancer
C- condyloma accuminatae

Sickle cell anemia perepheral smear ask what is the management ?

Brucellosis management ? Doxycycline and clindamycin

Man who is working in factory and he has Solid lung nodule how you will
treated?
-FNA
-True cut
-observation

SLE patient who have sign of infection how will you treat him ?
-start antibiotic with held adanilimab and hydroxycoline
-continue same treatment and add antibiotic
-stop one of them and give antibiotic

Case about mild spinal stenosis What’s the mx:


1-physiotherapy
2-epidural corticosteroids
3-sx

134
Patient with MVA came with rips fracture, x-ray on admission show 3-7
rips fracture x-ray repeated after 1 day showing lung infiltration at the site of
fractured rips. What is the diagnosis?
A-Flail chest
B-Pulmonary contusion

What cause false positive BNP?


Copd
Obesity

50 years male came for screening, found polyps and then has done
endoscopic polypectomy, what type of cells have the highest predator to
transform to malignancy?
A) Tubal
B) Villous
C) Tubulovillous
D) hyperplasia
Villous

68 y/o male presented to ER 8 (days or hours) post esophageal stricture


dilatation c/o sever chest pain and change in voice, dx ?
Esophageal perforation

2 y/o pt post on chemo for ALL , contacted a chicken pox pt, what to do ?
A- Acyclovir 7 days
B- Give vaccine now
C- Stop chemo
D- Don’t remember

7 years old boy complaining of purulent ear discharge for the past 3 days, he is
known to have tympanostomy tube placements because of purulent otitis media for
12 months, he like to swim. On examination he is afebrile, and purulent discharge is
noted coming out from tympanostomy tube. Which of the following organism is most
likely cause of the patient’s presentation:
A. Streptococcus pneumonia
B. pseudomonas aeruginosa

Female with behaviour and pain symptoms before menses?


Pelvic congestion syndrome
Pms
No pmdd in options

What the organism that cause rheumatic fever?


group A streptococcus

135
Fatigue pale SOB no spleenomegaly , Labs only showing low MCV and
normal MCHC what’s next ?
Periphral blood smear
No need for further test

Indication of thoracentesis
A-loculated
B-ph < 7.2
C-Glucose > 60

Chest crusting vesicles whats next ?


Swap from lesion
No need (clinical diagnosis )

Pt with jaundice uqp fever Cbd and extra hepatic duct are dilated mx?
⁃ mrcp
⁃ Ercp

2 day old infant came to hospital with complaints of seizure and decreased feeding
since yesterday. Inactive child with generalized increased muscle tone. CSF analysis
is normal. What is the most likely diagnosis?
Hypoxic ischemic encephalopathy
Neonatal sepsis
Neonatal tetanus
Pyogenic meningitis

Tx of hcv type 4
A.Sofobuvir ledipavir
B.No sofo-vel

RTA patient came to ER and goes throught exploratory laprotomy and


you find multiple laceration of liver, patient is unstabe
What will you do :
A-right hepatectomy
B-prehepatic packings
C-right hepatic artery ligation
D-indvidual hepatic artery ligation

Treated HCV case with negative RNA. How do u FU?


A.Liver US every 6 months
B.AFP every 3 months

DM with cardiac disease your goal in management?


A-triglyceride less than 150
B-BP less than 140/90
C- LDL less than 70
D - HDL more than 70

136
Pt with crohn which of the following increases risk of malignancy
a. Lesion located within 15 cm from the anal verge are
B.Primary sclerosing cholangitis
c. duration of 3 years

headache, unilateral, preceded with nausea, she has similar attacks 5 times per week,
each lasting 3-4 hrs, CT of head is normal, what will you give her?
A. Sumatriptan

Highest risk factor for BPH ? Age

Most considerable risk factor for MI?


A.Smoke
B.HTN

After accident found to have air under the diaphragm and spleen
laceration and thoracic aortic perforation [ patient hypotensive], + free
abdominal fluid by us what is the first thing to do?
• Thoracotomy
• B. Exploratory laparotom

An 8 years old healthy male, presented with a history of 2 months of


abdominal pain with 2 episodes of bloody stool. Proctoscope was done and
showed numerous polyps covered the linings and multiple biopsies taken.
What is the most likely diagnosis?
A-Familial polypoid
B-Ulcerative Colitis
C- Diverticulosis coli
D- Human papillomavirus polyp

Obese female concerning about GERD as a complication of gastric sleeve


Percentage:
A-5%
B-10%
C-20%
D-30%

Pt who have recent travel history to india c/o Right upper quadrants pain
US show hypoechoic mass?
A.amebiasis
B.hydatid cyst
Dx is given tx metronidazole

Preg with abdominal pain tenderness and bleeding uterus is firm


(placental abruption) Mx
Rapid response and multidisciplinary team

137
Natural history stages when best to do screening ?
A. pre-disease stage
B. Pre-clinical stage
C. Pre-clinical disease stage
D. Pre-recovery, disability, death stage

child with *pica*,pale, low socioeconomic state , low hemoglobin hand


lead was 2 (in tha labs resolute) threm, iron profile was normal(in tha labs
resolute also), x-ray report :dense metaphyseal bands , what is the
treatment?
A. vit-K
B. D-penicillamine
C. Iron

Pt treated with streptokynase due to MI came with massive hemoptysis tx?


Aminocaproic acid

30 something Female had her 1st menarche at 15 came with irregular cycle FSH LH
were high dx?
Ovaria insufficiency
Pcos

Pt with had black neck pigmentation with irregular cycle now hairstism
cause of the pigmentation ? Acanthosis nigricans

Pt post mcv abd destintion and tender jvp was raised and hypotensive
distant heart sound Type of shock?
Cardiogenic
Hypovolumiv
Neorogenic Septic

Infected bedsore mx? Depridment with vac closure

4 year old, known case of febrile seizures. Contacted febrile viral illness
4 days ago, now presenting with lethargy, vomiting but no change in bowel
motion. Mother is anxious about the seizures so she gives her child
paracetamol every 4 hours. On examination, jaundice with palpable liver.
What is the likely diagnosis?
Paracetamol induced hepatotoxicity Acute viral hepatitis
HUS
Viral autoimmune hemolysis

Patient has MI 6 weeks ago now he presented with dyspnea cough and
frothy sputum with harsh systolic murmur heard over the left parasternal area
what is the diagnosis?
A- Pulmonary embolism
B- Tricuspid regurgitation
C- papillary muscle rupture
D- aortic root rupture something similar

138
Patient persented with Hematemsis medical free, no hx of alcholic use ,bno hx of
medications :
A- Acut pancreatit
B-Chronic pancreatitis
C-Esophageal varicsis
D- Pancreatitc psodou

2 scenarois pt taken chemotherapy with neutrpenia and he is febrile tx?


Abx

Post thyroid surgery nurse notes spasm of the arm what to check ? Ca

Postop 6 days with nausea vomiting and abdominal distention and cant pass
stool Labs show hypokalemia ,What investigation you need to order?
A) urine analysis
B)ECG
C)urin K
D) Stool k

46 Y/O G3P1 at 34 weeks’ gestation presented to antenatal clinic for


regular check-up, she has unremarkable medical hx and uncomplicated
pregnancy Braxton Hicks and non-pruritic cervical discharge. Her prepregnancy
weight was 54.4 on examination cervical length was 33 mm.
VS were given & I believe they were normal.
Current weight: 52
Rubella AB: -ve
HBsAg: -ve
Blood type: O+
Which of the following is the most appropriate next step?
A. F/U after 2weeks
B. OGGT test
C. Do rubella Ab test / Repeat rubella screen
D. Give anti-D Ab

Man whats to travel and seeking travelers diarrhea treatment labs were
given showed abnormal KFT what treatment to give ?
A. Fluco....
B. Bismuth
C. No need
D. Probiotics

SLE how to confirm renal involvement? Renal biopsy

Long scenario about patient with dx of necrotizing pancreas with sepsis and treated,
now on mechanical ventilation and inotropes. What is the physiological process?
A) Increase lipolysis
B) Increase insulin resistance
C) Hypoglycemia
D) Decrease Gluconeogenesis

139
Definitive treatment of olive mass epigastric ( pyloric stenosis)

Pain in neck radiate to shoulder and arm , feels like electrical?


1- cervical disc prolapse
2- polymalacia romatica

Case of positive combs test


1- autoimmune hemolytic anemi
2- g6pd

Long case After hysterectomy urine out of the vagina?


1- uretrovaginal fistula
2- urethrovaginal fistula
3- vesiclovaginal fistula

drug that showed improvent in survival in a case of heart failure?


A.ACE
B.DIJOXIN

Chest pain that is progressive with ST depression and also normal cardiac enzymes
and asking about the diagnosis ?
A. Unstable angina

Q about a patient who is bleeding from the mouth and nose with severe
mandibular fracture and asking about the route of ventilation the answerds
were:(no mention of the his conciseness)
- orotracheal,
- crirothyrodotmy

Which of the following congenital heart disease associated with th LOWEST risk of
infective endocarditis?
A- ASD (secudnom)
B- VSD
C- PDA
D- sub valvular aortic stenosis(discrete)

27 years old male presented 4 weeks ago with severe necrotizing pancreatitis. He
underwent several interventions for sepsis control and drainage. He was put on
mechanical ventilation and inotropes. What is he expected to have?
1. Decreased gluconeogenesis
2. Increased lipolysis
3. Hypoglycemia
4. Decreased insulin resistance

140
A 50 years old women is complaining of amenorrhea for 6 and hot
flushes.. which of the following is going to help reach a diagnosis?
A) FSH
B) LH
C) progesterone
D) estrogen

A 2 months visit well baby clinic. The mother is counseling about when
she can start giving solid food to her baby, which one of the following is
generally recommended?
A) 3 months
B) 4 months
C) 5 months
D) 6 months

A 6 years old was broyght by his parents with hx of sudden abrupt loss of
environmental awarness with lip smacking followed by sudden return to normal
baseline. Brain Imaging= Normal, EEG= 3-Hz spikes Which of the following is the
most appropriate diagnosis?
A) complicated febrile seizure
B) typical abscence seisure
C) infantile spasm
D) cerebral palsy

patient did screening for DM : fasting was 7.4 , HbA1c was 6.3 what to do next
A- repeat fasting after 3 months
B- repeat a1c after 6 weeks
C- random glucose
D- 2hr 75 g

gestational diabetes is ?
A. Normal pregnancy associated with diabetogenic condition due to progesterone
B. Normal pregnancy associated with diabetogenic condition due to estrogen
C. Normal pregnancy associated with diabetogenic condition due to FSH
D. C. Normal pregnancy associated with diabetogenic condition due to lactogen

Pt with malar rash, photosensitivity, arthritis, cant brush her hair? Raynaud’s asking
about how to confirm:
A. Anti-ccp
B. Ant-RNA
C. Ani- ds-DNA
D. Anti-smith

A patient with sudden severe epigastric pain diffuse guarding and


sluggish bowl sound high amylase next step?
A. Erect chest X-ray.
B. Abdominal X-ray.
C. CT abdomen.

141
pt diagnosed with acute suppurative appendicitis , hitso show appendicular carcinoid
at base 2 cm ?
- observeation
- hemicolectomy

mother with her child 4 yrs her concern about his development he can jump in 1 feet ,
cry crosss (appropriate with his age ) ?
-reassure
- development delay as he cannot use tens verb

newborn his mother tell you that somebody in her family die from immune diefincy
disease what to do ?
-give BCG now
-all vaccine according to schedule
-delay

Pic of chancer

pic of hemothorax ask about next step

pt with pulmonary HTN , THROMBOEMBOLISM most appropriate investigation ?


-ECG
-ct angio

pt c/o cough that increase when lying flat next step ?


A.PPI

Brucellosis case tx duration with joint involvement

If u have a disease that has high incidence female and equal prevalence in male and
female . How did that happen :
- case fatality is high in female
- male patient die more from this disease
- female carry this diseasefor longer period
- male carry thys disease for shorter period

What has highest risk of malignancy in uc patients?

Criteria for thoracotomy after hemothorax drain

Case of heparin induce thrombocytopenia

pic p.falciparum
- malarial prophylaxis? Atavoq...
- malaria prophylaxis for close contact? Rifampcine

Pt with overian cyst and pleural effusion? Sex cord

Screening for GDM at which week?

142
pregnant with HTN& DM , in 14w come to antenatal care clinic for booking what will
you do?
- Review drug
- forget…

Pregnant come in 14w Known case of epilepsy for 2 years and poor
control on medication , what will you do?
- review the medication
- EEG
- CT

Kid with cervical lymphadenopathy and sore throat 3 weeks ago. Has Coca Cola red
urine?
A. IGA Nephropathy
B. Acute glomerulonephritis
C. Acute cystitis

Baby 29ws born with distress , most appropriate management:


- CPAP
- Surfactant

HTN on hydrochlorothiazid has sun burn after apply sun protect, step
after advicing not to exposed to sun and avoid midday sun:
1- Worm shower two times daily
2- Stop hyrochlorothiazidand monitor blood pressure
Answer: by M.alharbi IM consultant SMLE:
Change to another Anti HTN and decrease exposure to sun

Recurrent abortion percentage: (No age in Q)


A-5
B-10
c-30
d-50
Answer:
First time > B
Recurrent > C

Female with 4 month history of mass , no sx of hypo or hyperthyroidism all was (-)
US: 7 mm solid nodule with no cervical node enlargement Pt labs: TSH is normal,
What’s the management?
•A. Follow up imaging
•B. FNA
D-ct chest
Answer :
A vs B

143
COPD patient developed sudden symptoms of spontaneous pneumothorax, x ray
was done and showed *2 cm* pneumothorax, mx? Pt was vitally stable
A. Needle
B. Chest tube
C. Observation
Answer :
Chest tube

12 year old boy presents w epigastric pain and burning sensation for 1 y sometime
associated w vomiting. His mother have same issue chronic abdominal pain .
Test: blood and mucus-> positive (normal is negative) -> this is what was (written
exactly,Dx)
A-Ulcerative colitis
B-pancreatitis
C-cholecystitis
D- peptic ulcer disease
Answer: D

Female had stress incontinence what you will find on P/E


A-Urethral hypermotility
B-Rectocele
C-Paraurethral defect something like that
Answer: A

Female came to antenatal care During invs ~> high TSH


A Thyroxine now
B Thyroxine during pregnancy
C delay pregnancy one year
Answer: A

pregnant DM what is the complication for her as in normal pregnancy?


A-Cystic fibrosis
B-Precalmpsia
C-Hydramnios
Answer: A

4cm thyroid mas + normal TSH + FNA inadequate what to do?


A- Follow up
B- Repeat FNA
C- Thyroidecromy
Answer: A

Cystic hygroma in first trimester


A. CVS

144
Thumb printing sign in in old pt , Next
A.Colonoscopy
B-diagnostic laprotomy
C-Exploratory laprotomy
Answer:
A if stable
C if unstable

How to prevent neonatorum toxicum


A-Agive vaccine to mother after 72 hour delivery
B-give neonate human toxoid injection
C-give infant antitoxoid tetanus
D- Give mother during pregnancy
Answer: D single dose of DTap b/t 27 -36 weeks

about a patient came with unprovoked seizure for the first time and everything else
was Normal, what investigation to be done
A: EEG
B:Ct of brain
Answer: brain imaging MRI > CT

years old female presented to gynecology clinic complaining of 54 dysuria and


urinary incontinence, she stated that the leak is not related to specific activities and
occasionally associated with cough. Upon investigation urinalysis and cultures were
insignificant, However, urodynamic study showed: contracting bladder even with
small amount of dripping, Which of the following is the most appropriate
management?
A. Kegel exercise
B. Anterior Colporrhaphy
C. Anticholinergic agent
D. Bladder Suspension
Answer : A vs C

Buisness man travelling, with oral candida, normal neutrophil high lymphocyte
diagnosis?
A-HIV
B-Toxoplasmosis
C-Brucellosis
Answer : A

Twin pregnancy, IDA, Placenta previa. What is the indication for IV IDA?
A- Constipation
B- Moderate to severe anemia
C - Placenta previa
D - Twin gestation
Answer :
If Hb < 9 B
If no Hb : C

145
butcher cut his finger by knife and comes after few days ( developed read streaks
from the wound) what’s the causative organism ?
A pseudomonas
B streptococcus pyogen
C brucella
D klebsiella
Answer: B

Clonzapin use for childhood with?


A.Bipolar
B.Schizophrenia
C.Major depression
Answer : B

A 45-year-old male patient complaining of back pain worsen in the night, not
improved by rest, and improved with exercise and in the morning, and the most likely
diagnosis is Ankylosing spondylitis, which of the following is the most appropriate
test in this case?
A HLA b27
B MRI sacroiliac
C CT sacroiliac
D Anti CCP
Answer: B

questions about Jugular foramen fracture?


A-ipsilateral vocal cord paralysis

Pt with A fib on heparin , in labs ( low PLT)


A.stop heparin and start another anticoagulant

How to measure Fetus length in 38 weeks?


A-weeks gestation
B-Femur
C-Abdominal circumference
D-CRL
Answer:
Hight = femur
Wight = abdominal circumference

Loin pain + hematouria + hard lump on examination how to diagnose ?


A.ct with contrast
B-scintigraphy
C. US
D- MRI
Answer : A
Dx : RCC

146
56 y /o appendicitis treated with abx and resolved what next?
A- Colonoscopy after 6 w

Scd which invesx will help u the most?


A-Liver bx
B-Bone marrow
C- Retic. Count
Answer: C

after 2w of lap-chole, patient P/W vague abdominal pain, all vitals is normal including
temperature. CT shows large collection in the subhepatic area, the most appropriate
next step is :
A)ERCP with stenting
B)laparoscopic drainage
C)CT guided drainage
D)operative drainage
Answer: C

What anti diabetic medication can reduce mortality in DM patients


A-metformin
B- Gliflozin
C- Glipizide
D- Acarbose
Answer ; A

RTA, decrease air entry in the right side, tympanic on percussion The patient
was hypotensive 90/60 , Whats next?
A.X ray
B.Chest tube
C.Needle decomposition
Answer: C

IUGR case, which has highest value for diagnosis?


A. Nonstress test
B. Serial US
C. Take kinck chart
Answer: us Doppler umbilical artery

A woman was taking highly androgenic progesterone without knowing she is


pregnant. What complication will her daughter face?
A. Nothing will change
B. Hirsutism
C. Masculinization
D. Feminization
Answer: C

147
year-old lady with secondary amenorrhea (elevated FSH and LH) which of risk or
complication she might probably develop in the future?
A. Risk of endometrial cancer
B. Risk of ovarian cancer
C. Risk of osteoporosis
Answer: C

Patient with pelvic pain and yellow vaginal discharge and flagellated protozoa, what
is the organism?
candida
trichomonas
Answer : trichomanonas vaginalis

Molar pregnancy follow up of HCG


A-weekly till 3 reading negative then 6 months
B- weekly then monthly for 6 months
C-Monthly then weekly
D- monthly till negative
Answer Mostly B

women came to you with US showed molar pregnancy what is the managment
A- D/C
B- methotrexate evaction
C- suction
D- hysterectomy
Answer C

Women with 42 weeks gestation No contraction , cervical dilation 2, -3 station, normal


CTG, what u want to do ?
A. reassurance
b. CS
C.induction of labor
D. augmentation labo
answer : C

Lady presenting with vaginal bleeding with fever 15 days after C/S. Most probable
Dx?

A. Wound infection

B. Retained products of conception

C. Endometritis

D. Mastitis

Answer: C

148
case of a 14 months old baby with hx of abdominal pain and passing red currant jelly
stool, whats is most common site of this pathology?

A. iliojojnal

B. jojnoilial

C. iliocolic

D. ilioilial

Answer : C

Pt with hand injury clean open injury with separation of tendons and median nerve
(injured) what’s best management?

A- Primary repair of injured structures

B- debrid and graft

C- Debrid and secondary closure

Answer ; C

Post sleeve abdominal pain what to do?

A. Us

B. Xray

C. Ct

Answer : CT

27 year old patient 3 days post sleeve gastrectomy, he is presented with mild RUQ
pain,upon examination there was mild tenderness with no signs of peritonitis vitals:
BP:100/80 HR:133 what is the cause?

A- gastric leakage

B- sepsis

C- decreased oral hydration

D- inadequate analgesia

Answer: A

149
25yA patient presents with a scalp laceration which occurred 6 hours ago. How will
you repair this injury?

A. Healing by granulation

B. Debridement with primary closure

C. Vacuum assisted closure (VAC)

D. Debridement with secondary graft

Answer: B

Pt had history of ectopic pregnancy 2 years back treated with methotrexate, now
present to preconception clinic , how to minimize her next

A.Wait for one year more

B.Folic acid

C.Early detection pregnancy

Answer : C

How can we diagnose the pt as cervical insufficiency based on transvaginal US?

A.<5mm

B.<10mm

C.<25mm

D.<30mm.

Answer : C

patient immune compromised what to avoid?

A.Varicell

B.Pertussis

C. PV

D.Influenza

Answar : A , live attenuated vaccines should be avoided

150
10 years old child came with fever and vesicular rash on the palms associated with
painful mouth ulcer. The majority of his classmates have the same issue. What
additional findings you’ll see in this patient?

A.Enlarged congested tonsils

B. Rash on the soles

Answer : B

2 years child suffering for sob 1 day and cough, he came and stable, no history of
contact, asthma rolled out, no fever, diminished air entery in right lower lobe, wheez,
(foreign body) initial :

A. chest x ray lateral and AP

B. CT

C. bronchoscopy

D. direct laryngioscopy

Answer : C

How to diagnose a 12yrs girl with HTN?

A. Above 90th percentile BP for age and gender

B. Above 95th percentile BP for age and gender

C. Above 140/90 mmHg.

Answer : B

parents brought their kid who has worsening nausea and vomiting, plus headache
and visual impairment, these symptoms are worst at early morning and late night,
and improving with walking and activity, SO; what’s true about Dx and Mx of this
condition?

A. EEG

B. US abdomen

C. Brain MRI

D. Muscle biopsy and EMG

answer : D; case of Eaton Lambert)

151
Elderly with massive lower GI bleeding, most common diagnosis?

A. Angiodysplasia

B. Diverticulosis

C. Colon ca

Answer : B

Typical case of gallstones and came with severe RUQ pain after fatty meal for 6 hours
and after 2 hours the pain gradually decrease, ultrasound shows multiple stones with
normal gallbladder, what is the treatment?

A) laparoscopic cholecystectomy

B) observation

C) ursodeoxycholic

Preterm baby (23 weeks) with multiple congenital malformations and in need of
intubation and resuscitation, the mother is one of the hospital physicians and wish to
not resuscitate?

A. Honor her wish

B. Consult the ethical committee

C. Ignore the mother's wish and resuscitate

Answer:A

The World Health Organization (WHO) published the WHO Surgical Safety Checklist
in 2008 in order to increase the safety of patients undergoing surgery. The Checklist
consists of three phases of surgical procedure :

A. Before admission, Before skin incision , at discharge from hospital.

B. Before admission , before induction of anaesthesia , at discharge from hospital.

C. Before induction of anaesthesia , Before skin incision , Before patient leaves operating
room.

D. Before admission , Before induction of anaesthesia ,Before patient leaves operating


room.

Answer:C

152
1year child has HIV, Low CD4 count, which vaccine is contraindicated?

- OPV

- Varicella

- MMR (I think)

- Dtap (I think)

Answer : A

neck injury in zone 3 , cta show’s vessel injury whats your next step?

A-ligation

B-primary suture

C-embolization

Answer : C ( not sure )

contraindications for preeclampsia pt :

A- methyldopa

B- hydrlyzine

C- nifidipin

Post appendectomy female came with LR abdomen mild tenderness Ex Normal By


CT there is 2*2 collection in Retrocecal :

A. Exploring laparotomy

B. percutaneous drainage symptomatic

C. laparoscopic

D. conservative with Anitbiotic

Answer : B

how the surgeon can find the appendix , best way to identify appendix in surgery?

A- From internal ilac artery

B-Plapate ( something)

C-There is word Taeniae coli

D-ileocecal valve

Answer :C

153
Patient in icu on chemoradiotherapy hie family tell the doctor to give pain killer for
the patient and doctor said that may accelerate his death :

A) principle of double effect

Answer: A

Baby when mother put him on table for examination he started to laugh then when
the doctor came to examine him he turned to his mother, how old is he :

A. 4m

B. 6m

C. 8m

Answer:B

Milestone: walk steadily, two body parts , says 6 words and immature pencil grasp :

A. 12 months

B. 14 months

C. 17 months

D. 24 months

Answer:A

20 married women came to clinic want contraceptive , she had amenorrhea and
oligommenorea ?

A )combined oral contraceptive

B)IUD

Pregnant women has vaginal bleeding she has placenta Previa and she had 2 vaginal
episode before :Mx

1) wait until other vaginal bleeding

2) immediate delivery

3) observation and follow up

154
Pregnant women in CS how remove plecenta in appropriate way that decrease risk of
bleeding

1) manual

2) hysterectomy

3) wait ….

Answer : 1

27years old male presented 4 weeks ago with severe necrotizing pancreatitis. He
underwent several interventions for sepsis control and drainage. He was put on
mechanical ventilation and inotropes. What is he expected to have?

1. Decreased gluconeogenesis

2. Increased lipolysis

3. Hypoglycemia

4. Decreased insulin resistance

Child had long history of purulent otitis media with effusion which was treated with
tympanostomy tube came to you today with yellowish discharge from the tube. No
fever. In history he likes to swim (WRITTEN LIKE THIS) what is the organism?

A. STREPTOCOCCUS Pneumonia

B. STAPHYLOCOCCUS AURUS

C. PSEUDOMONAS AERGONISA

D. STAPHYLOCOCCUS EPIDERMIS

Pregnant patient come with vaginal bleeding and abdominal pain ( no vital signs
mentioned) next step ?

1. Vaginal examination

2. Ultrasound.

3. 2 IV line ( something like that ).

- most important initial step to reduce ICP

1- hyperventilation

2- elevate head

3- IV mannitol

155
old patient came with subdural hematoma with signs of lateralization imaging
revealed 13 mm shifting. his GCS 7/15 then was intubated and resuscitated what to
do next?

A- iv mannitol

B- admit ICU and observe

C- craniotomy

D- insert intracranial monitor

Newly married women come to gyne dr for screening and check..which of the
following is highly diagnostic?

A-vaginal inspection

B-general appearance

C-Vaginal US

D-pelvic digital examination

-Case of Parkinson disease, which one is the following factors presents in patients
who are high risk of developing dementia?

A. Forgetting future appointment

B. Word finding when talking

Pregnant with previous hx of DVT What will give her?

- Enoxaparine

- Aspirin

- Warfarin

- No need for any drug

Pt DM& HTN & previous MI come with chest pain relief by rest on enalipril , heparin,
furismoid , statin , what will give?

- bisoprolol

- no need any drug

156
Pelvic Fracture with injury to membranous part of urethra. Blood seen in external
meatus. What is the most appropriate action?

A) Cystoscopy

B) Folly’s Catheter

C) Suprapubic catheter

D) CT pelvis

blunt trauma,left upper abdominal pain , imaging shows 7 cm tear resulting in


subcapsular splenic hematoma , normal portal vein, thrombosed splenic vein. What
to do?

A.Splenectomy

B.Preserve surgery spleen ( conserving surgery )

C.Distal splenorenal anastomosis

D.portocaval something

Chad score for pt 65, dm, HTN , previous stroke

Female 60y with heavy bleeding , have fibroid 5y ago and by medical record indicate
that fibroid same size , what will do next

- Endometrial sample

- Hystrectomy

- Myomactomy

Femal with heavy veginal bleedingHypotensive, Hb 7 . Proper next step?

A. hestroectomy

B. Treat anemia

C. Progesterone ~not sure

D. D&C

157
41YO P5 +3 presented to the clinic complaining of abnormal uterine bleeding her
Menstrual period is regular, associated with blood clots and pain that is not relieved
by analgesic she had previous myomectomy, she is a known case of PCOS& her BMI
is 40?

- A-Adenomyosis

- B-Endometriosis

- C-Uterine fibroid

- D-Endometrial hyperplasia

which tocolytic drugs cause patent ductus arteriosus ? Indomethacin

Dose of iron in pregnancy?!

0.2

0.8

1.4

pleural effusion positive interferon gamma?! TB

Eczema can on scalp and face and spread to the body except the diaper line :

A. Scapes

B. Atopic dermatitis

C. Wiscot aldrech syndrome

Pt came to the clinic, he is talkative jumping from topic to topic without completing
each one. What is this called?

A. Flight of ideas

B. Thought insertion

C. Broadcasting

D. Thought withdrawal

a female pregnant didn’t remember his LMP ?

Crown rump length

158
patient come with medial leg ulcer. The most likely diagnosis is?

A. Diabetic

B. Venous hypertension

C. Atherosclerosis

D. Buerger's disease

surgeon manipulated data to make the results support his new surgical technique:

A. fabrication

B. falsification

A long scenario about an infant who’s at high risk of developing IDA. The question
was, in such patients who have ((high risk)) they’re screened at which age?

A. 12 months

B. 4 months

C. 8 months

Worried about cervical cancer. She took her first dose of HPV vaccine 3 months ago.
What the best thing to do at this visit today?

A. Schedule app after 3 months

B. No need to do anything at this visit

C. Give 2nd dose at this visit

Child with chest pain and can hear loud heart sound HR:268. What’s the most
appropriate invx?

1-12 leads ecg

2-echo

3-cardiac enzymes

159
10 year old boy presented to the pediatric clinic with a history of skin rash. The
parents report that the symptoms had started with a single red spot over a small
scratch and have spread over the last 4 days. There are several lesions on the
cheeks. The trunk and the upper limbs. Physical examination confirms swollen lymph
nodes in the arm pits and groins Blood pressure 100/65 mmHg , HR 80/min ,
Respiratory rate 24/min , Temp 37.4 c , Which of the following is the most appropriate
treatment?

A- steroid

B- acyclovir

C- antibiotic

D- antiseptic

Patient with interstitial pneumonitis (mentioned), he is asking about what is common


with this type of inflammation or common cause of this finding?

A- Lobar pneumonia

B- Bronchoalveolar pneumonia

C- Viral pneumonia

D- Secondary (T.B)

Post CBD surgery complains of chills and rigors. BP 119/? HR 80 RR 28 T38. Normal
WBC ( 11 i think?).

⁃ sirs

⁃ Bacteremia

⁃ Sepsis

⁃ Severe sepsis

child with typical symptoms of DM( polyuria, thirst, ..) on lab: the random blood
glucose was normal . ask about the cause of symptoms

- Decrease renin

- decrease insulin

- increase insulin

- absent of glycogen

160
23 y/o come with fresh bleeding per rectum in image found the lesion near to
ileocecal valve Dx?

- Crohns

- PU

- meckel diverticulum

- another recall

Pediatric patient with recurrent lower GI bleeding. Px looks healthy & well. On
colonoscopy: blood seen coming from ileocecal valve. Dx?

A. Angiodysplasia

B. Crohns

C. meckel diverticulum

Female with fishy-smell discharge, analysis: PH: 5.1 What type of cells will you see?

A- Multinucleated giant cells

B- Overproduction of lactobacillus

C- Single nucleated cells Review

D- Granulated epithelial cells

Child presented to the ER with fever and abdominal pain. After a fall 1 day ago, the
mother noticed abdominal distention mainly on the right side. On examination he is
pale and hypertensive, your dx ?

A. Liver contusion

B. Pyelonephritis

C. Wilm's tumor

D. Neuroblastoma

Baby 29ws born with distress , most appropriate management:

- CPAP

- Surfactant

161
Scenario of Inferior IM (Given the dx) then showed ECG like this and asked what’s the
dx?

-Heart block 1

-Heart block 2

-Heart block 3

-Heart block 4

Open fracture initial management? (Was dirty, )

-Antibiotics

-Debridment

-Internal fixation

-External fixation

4 year old Child with recurrent chest infection, inspiratory crepitation and expiration
weezing, poor growth and fatigue, finger clubbing, on further examination he has
offensive greasy stool, what will you find on further examination?

nasal polyp

inguinal hernia

Elderly with ejection systolic murmur, asymptomatic. His pressure gradient is 40


mmhg. What’s the appropriate management?

- balloon valvuloplasty

- Valve replacement

- Evaluate by echo or exercise test

- follow-up

47 female presented with heavy vaginal bleeding since 7 months and in us there is
blood in cervix, she denies any tenderness or fever US: the endometrium thickness
was 13mm, there is 2.2 x 1 cm mass , what is the best action to reach the dignosis:

a - sonohysterogram

b - ct abdomen and pelvsi ( not sure )

c - mri pelvis

d - complete blood count and coagulation profile

162
high Hb A2 and high Hb F Dx?

-IDA

-Sickel cell

-Thalasemia Alpha

-beta Thalassemia

-male comes with chest pain and dyspnea 2 days after landing, he is tall , what will u
do?

-needle decompression

-thoracocentisis

-chest tube

-conservative

Supra condylar fracture management? k wire

Case of Wilson ask about diagnostic test?

- 24h Copper urin

- Iron liver biopsy

- AFP

Patient with signs and symptoms of IBD, loose of stool etc., (they didn't mention
bleeding), on imaging, there is lead-pipe appearance in the colon (I'm sure), also
during endoscopic examination, there was continuous inflammation of colon, and
there is ileitis (they mentioned it, sure 100%).

A- Crohn's

B- Ulcerative colitis

C- Gastrointestinal T.B

Abdominal pain and loose stool for months associated with weight loss. Biopsy form
ileum showed non-caseating granuloma. What is the diagnosis?

A⁃ Pseudomembranous colitis

B⁃ Intestinal Tuberculosis

C⁃ Crohns

D⁃ Celiac

163
Case of newborn with muffled rash and growth retardation which infection ?

- Rubella

- Toxoplasmosis

- HIV

Seizure case and EEG finding showed temporal spike? rolandic seizure

2 day old infant came to hospital with complaints of seizure and decreased feeding
since yesterday. Inactive child with generalized increased muscle tone. CSF analysis
is normal. What is the most likely diagnosis?

Hypoxic ischemic encephalopathy ✅


Neonatal sepsis

Neonatal tetanus

Pyogenic meningitis

Tx of hcv type 4 ?(No sofo-vel) Sofobuvir ledipavir ✅


A neonate was found to have a generalized petechial rash and intracranial
hemorrhage immediately after delivery. His Hb is low, Platelet is 4, and all other labs
are normal. The mother has a normal platelet count. What is the most likely
diagnosis?

a. ITP

b. Alloimmune thrombocytopenia (NAIT)

c. Congenital thrombocytopenia

d. Congenital CMV Infection

RTA patient came to ER and goes throught exploratory laprotomy and you find
multiple laceration of liver, patient is unstabe What will you do :

A-right hepatectomy

B-prehepatic packings

C-right hepatic artery ligation

D-indvidual hepatic artery ligation

164
Treated HCV case with negative RNA. How do u FU?

- Liver US every 6 months

- AFP every 3 months

Pt with crohn which of the following increases risk of malignancy

a. Lesion located within 15 cm from the anal verge are

B.Primary sclerosing cholangitis ✅


c. duration of 3 years

Highest risk factor for BPH ? Age ✅


Fetus 180 HR , early decerelation And other information, and ask about which one
indicate severity I think:

- HR ✅
- early decerilation

Natural history stages when best to do screening ?

A. pre-disease stage

B. Pre-clinical stage

C. Pre-clinical disease stage ✅


D. Pre-recovery, disability, death stage

Pt treated with streptokynase due to mi came with massive hemoptysis tx?

Aminocaproic acid ✅
Infected bedsore mx:

Debridement with vac closure ✅


4 year old, known case of febrile seizures. Contacted febrile viral illness 4 days ago,
now presenting with lethargy, vomiting but no change in bowel motion. Mother is
anxious about the seizures so she gives her child paracetamol every 4 hours. On
examination, jaundice with palpable liver.What is the likely diagnosis?

Paracetamol induced hepatotoxicity ✅


Acute viral hepatitis

HUS

Viral autoimmune hemolysis

165
Patient has MI 6 weeks ago now he presented with dyspnea cough and frothy sputum
with harsh systolic murmur heard over the left parasternal area what is the
diagnosis?

A- Pulmonary embolism

B- Tricuspid regurgitation

C- papillary muscle rupture

D- aortic root rupture something similar ✅


Patient persented with Hematemsis medical free, no hx of alcholic use , no hx of
medications :

A- Acut pancreatit

B-Chronic pancreatitis

C-Esophageal varicsis ✅
D- Pancreatitc psodou

A 17 - year old man developed FEBRILE NEUTROPENIA and shock POST


CHEMOTHERAPY for lymphoma . Which of the following is the most appropiate initial
antibiotics ?

A- meropenem

B. vancomycin

c. cefuroxime

D. caspofungin

Answer : A. MEROPENEM ✔️
Tumor lysis syndrome finding

Hypocalcemia ✅

166
Postop 6 days with nausea vomiting and abdominal distention and cant pass stool

Labs show hypokalmia

What investigation you need to order

A) urine analysis

B)ECG ✅
C)urin K

D) Stool k

46 Y/O G3P1 at 34 weeks’ gestation presented to antenatal clinic for regular


check-up, she has unremarkable medical hx and uncomplicated pregnancy Braxton
Hicks and non-pruritic cervical discharge. Her pre-pregnancy weight was 54.4 on
examination cervical length was 33 mm. VS were given & I believe they were normal.

Current weight: 52

Rubella AB: -ve

HBsAg: -ve

rta type: O+

Which of the following is the most appropriate next step?

A. F/U after 2 weeks ✅


B. OGGT test

C. Do rubella Ab test / Repeat rubella screen

D. Give anti-D Ab

Male patient had oil dissolvent injury to his index finger. On P/E mild tenderness
when moving the hands and no other abnormalities, what is your next step?

A) Oral antibiotic

B) Oral antibiotics and steroid

C) Elevate the hand and ice packing

D) Surgical management

167
Female in her 40,s complain of headache and irritability 8,10 days before
menstruation and resolves 2-3 after ‫؟‬

- Endometriosis

- Premenstrual syndrome

which of the following decrease efficacy of clopidogrel ??

1- Omeprazol

2- SSRI ✅
patient with RUQ pain and tenderness and mass. Investigation showed Entamoeba
histolytica, and there’s abscess 12X14 cm I think, and ask about first next step in ttt?

A. metronidazole

B. laparotomy

C. percutaneous drainage

D. Percutaneous aspiration (exactly like this)

Pt known know case of diabetes Mellites came with abnormal liver functiontest LFTS
were all high?

A-hemochromatosis

B- viral hepatitis (unlikely if pt Asymptomatic) C- autoimmune liver disease

D- non -alcoholic fatty liver disease

65 years man presents to your clinic and looks weak , dehydrated, pale , thin and
emacitaed. he complains of anal itching , discomfort from the pas few months. On
examination, you find an anal mass that is 2 cm away from the anal verge ,
cauliflower like and friable. What is your most likely diagnosis?

A- Anal Cancer

B- Rectal Cancer

C- condyloma accuminatae

D- Dtap

168
About 28 years old, came to ER with heavy vaginal bleeding, she is nulliparous,
pregnancy test negative, no pain, regular cycle, but always with menorrhagia, how to
stop the bleeding now?

A- Give conjugated estrogen

B- Give progesterone

C- Give GnRH

D- Insert levonorgestrel IUD

Female patient complaining of pain before menses and resolved in the third day of
menses, how do you diagnose it?

A-Hysteroscopy

B-Abdominal US

C-Clinical symptoms

Patient with severe abdominal pain with menstrual cycle (dysmenorrhea), affecting
her work, what can you give?

A-Misoprostol

B-NSAID

C-Progesterone

D-OCP

Intrauterine device implantation patient have brown discoloration and abdominal


pain. Whats the most likely diagnosis?

A. Uterine rupture

B. Pelvic inflammatory disease

Patient with hypertension, heavy bleeding and anemia, what contraceptive can she
use?

A. Tube ligation

B. IUCD mirena

C. Condom

D. OCP

169
A 28 year old female known to have an ovarian cyst, came to your clinic asking about
contraception, which of the following methods is most appropriate for her?

A- OCP

B- Oral progestins only pill

C- Vaginal ring

D- Male condom

Answer: A

23 years old female pregnant in her 28th week, pregnancy test was done at home.
She presented with mild bleeding loss of fetal movement. She denied any passage of
tissue or abdominal pain. Transvaginal US was done and showed pregnancy of 18
weeks and no heart beat. (No other information was provided about the cervix). What
is your diagnosis?

A) Fetal Demise

B) Missed Abortion

C) Incomplete abortion

Which of the following is the indication of cervical cercalge?

cervical length less than 30

Cervical length less than 35

cervical length less than 25 cervical length less than 20

Pregnant lady had seizure and is unconscious, her baby is healthy, what to do?

A- mgSo4

B- Establish airway

C- Fluids

D-Urgent delivery

Pregnant 37 week with BP 160/110 every thing else was normal Her BP during
pregnancy was 120/90?

A. Eclampsia

B. Gestational HTN C. Preeclampsia

170
Pregnant women k/c of epilepsy on valproic acid, which of the following associated with it?

A. Polyhydraminos

B. Oligohydraminos

C. Neural tube defect

-A full-term C-section baby presents with intercostal retractions and cyanosis.

What is the most likely diagnosis?

A- Transient Tachypnea of Newborn

B- Meconium Aspiration Syndrome

C- Hyaline Membrane Disease

D- Bronchopulmonary dysplasia

Answer: A

Patient with pelvic pain and yellow vaginal discharge and flagellated protozoa, what is the
organism?

Trichomonas vaginalis

child came with potential toxic dose of paracetamol. The physician requests an
“Acetaminophen level”. The lab technician calls to report results. He says: “two” stops and
pauses and then says: ”one, three”. The nurse mistakenly writes it as “2.13” while the real
result was “213”. The patient went into irreversible liver damage, What was wrong here?

A-Poor communication

When should we stop metformin according to GFR:

A- gfr 40-60

B- gfr below 40

C- gfr below 30

D- gfr below 15 Answer: C

171
Lactating mother with a 5 cm breast mass with thinning of the skin and tender?

A. Incision and drainage

B. Conservative with antibiotics

Twin pregnancy both vertex how to deliver?

- nvd

- c/s

- forceps

- ventose

Patient presented with Sjogren disease (mentioned), with hypokalemia, asking about type of
renal tubular acidosis?

o Type 1

o Type 2

o Type 3

o Type 4

- systemic sclerosis question

Another recall

systemic sclerosis pt w HTN ?

ACEI

30 year old female with a sclerotic disease. She has LL edema, +3 protein, creatinine was
238, what is the appropriate management?

A- Lisinopril

B- Thiazide

C- Beta blocker

D- Verapamil

172
- Rate control afib ??

Another recall

Patient known case congestive heart failure with atrial fibrillation.

Which of the following could be given for rate control:

A-Digoxin

B-Lidocaine

C-Adenosine

D-Verapmil

- Rhythm control afib amidarone

Pt with CHF and Afib. What will you use to control his cardiac rhythm?

A - Bisoprolol

B – amiodarone ✅
(Rate control : BB, CCB or digoxin

Rhythm control: Amiodarone , sotalol, flacinide)

Case about meningitis and CSF culture results show gram +ve cocci in chain what is the
appropriate

antibiotic ?

A- Ampicillin

B- Vancomycin

C- Ampicillin and Gentamicin

D- Ceftriaxone and Vancomycin

Pregnant 25 weeks GA with asymptomatic UTI what is the treatment ?

A- oral ciprofloxacin

B- oral nitrofurntine

173
C- oral TMP

Pt 25 yo medically free and no history of DM in the family came to the clinic ask for DM
screening

because his friend recently diagnosed has DM

At which age do you screen him:

30

35

40

45

Parents bring a child to the clinic and the doctor tell them to start Iron supplementation next
month,

what is the age of the child:

21. patient c/o 4 months of hearing loss now come with 7 n palsy ]

- A. rumsy hunt

- B. brain tumor

- C. bell's palsy

561-Patient ingested alot of paracetamol 3 hours ago then came to er asymptomatic they
did paracetamol level and it was above the toxic level what to give:

A) oral activated charcoal

B) NAC

174
C) gastric Lavage

Pregnant in labour, she takes heparin,have low platlets post delivery she has heavy
bleeding? What to give?

A) FFP since it’s heavy bleeding

B) Portmine sulphate

C) Vit K

227-35 years old woman known case of DM did cholecystectomy 2 years ago, she has
been using NSAID to relieve RA symptoms, she develop HTN, Na normal, K below
borderline, Cr higher than normal, what do think the cause of HTN?

A. Essential hypertension

B. pheochromocytoma

C. NSAID induced HTN

D. Primary hyperaldosteronism

Risk factor for preeclampsia?

- A) Maternal age

- B) Gestational age

- C) Multiple pregnancy

What is the best to use for follow up prognosis of COPD?

A. FVE1

B. FVE1/FCV

Case of COPD , absent gag reflex (nothing about consciousness ) on sever respiratory

distress Ph:7.25

High pCO2 and low O

175
O sat: 83

What is your next best step:

A. Intubation and ventilation

B. Non invasive

‫ ؟ ايش اهم شيء في التحليل يدل على المرض‬.. UTI 16-‫سؤال عن‬

A-increase ESR

B-nitrate

C-increased WBC

6- Time of ovulation :

A)36 hours after LH surge

B)36 hours before LH surge

C)At time of LH surge

D) something related to estradiol

Pediatric patient with lacrimation , miosis , salivation . What is the

diagnosis?

Organophosphate poisoning

Pt had history of ectopic pregnancy 2 years back treated with methotrexate, now

present to preconception clinic , how to minimize her next pregnancy risk ?

A- Wait for one year more

B- Folic acid

C- Early detection or diagnosis of pregnancy

176
How to know weight of the baby on US?

A. Measure abdominal circumstance

3 months years old present with paroxysmal cough with deep

inspiration between the cough, conjunctivitis, diarrhea, he is

up to date with his vaccination.. What is the causative

organisms?

A. Chlamydia

B. Adenovirus

C. Pertussis.

29 sep (36-70)

172- Blood loss 25% what will most likely be affected first?
A. Urine output
B. Pulse pressure
C. GCS
D. Respiratory Rate

173- A 37 years old male presents with pain in the lateral aspect of right leg with
redness and tenderness and a red streak on the thigh. Bp normal, Temperature is
39, What is the most appropriate management? A. penicillin
B. clexane
C. tetracycline
D. azithromycin

174- Old patient smoker 10 packs


Has respiratory symptoms
didn’t mention weight loss has clubbing:
A-COPD
B-Lung cancer

175- Trauma patient with emphysema


Injury level one (mentioned)
Bp stable
What to do next :
A-Embolization

177
B-CT

176- 50 old DM with 5y hx of unhealed ulcer about 2*2 cm in the dorsal of big toe biopsy
taken and show ( psoudmembrance hyperplasia like that !)
Aproubrate mangment ?
Ambutation of big toe
Surgical depriment

177- Patient in her 40&apos;s she had mamogram and US done showed some findings
that indicate birads 3 (probably benign) how to
manage?
Follow up in 6 months
Core needle biopsy

178- male had MVA accident, unconscious in the ER with open wound in the anterior neck
Vitally unstable Whats the appropriate management ?
Oxygen mask
Cricothyroidotomy
Tracheostomy
Endotracheal intubation

179- with tocolytic can cause pda ‫االدویة من نوع كذا وحاط‬

180- a question about marfan syndrome and its character want the diagnosis

181- 4 days baby brough by his mother with cyanosis his labs : hgb 214 hematocrit 75%
what is your mangment ?
partial exchange transfusion
oxygenation
hydration and monitor
no need for treatment
‫نفس السیناریو مره التشخیص و مره المانجمنت‬

182- -2 months vaccinations


Rota not within choices

199- pregnant female 37 weeks present with moderately sever vaginal bleeding and
placenta covers cervical os (Us GA 32 weeks ) Choices didn’t make sense but included
urgent cesarean delivery

200- case of chorioamnionitis what’s the management

201- Patient had surgery after 1 day he had diminished breath sounds in the left lower lobe
how to manage?
A- Incentive spirometry
B-Chest tube
C-Needle decompression

178
202- 7 years child , systolic murmur heard in lower sternal area with no thrill, ( more
noticeable when supine ? ( 105 - 476)
ASD
VSD
AORTIC STENOSIS
STILL MURMUR

203- Case of child acute pancreatitis ( dx not mentioned) vitality stable , increased WBSc.
what’s the most appropriate next management:(106 , 474)
A. Npo , fluid resuscitation, analgesia
B. Start septic work up.
the rest i think unlikely.

204- Middle aged,Medically surgically free , came with epigastric pain and abd. Distended,
Examination: tympanic abdomin + tenderness. (107–473)
Radiology : pneumobelia ( exactly)
Diagnosis?
A. Small bowel volvulus.
B. gallstone ileus.
the rest i think unlikely.

205- Appendicular abscess ( 10x12 cm ) reaching flank ?


Perctenous drainage (122 , 408)
Open app.
Lap app.
Open drainage

207- Patient presented to ER after 8 hours of esophageal dilitation, He has changes in


voice with chest pain, what&apos;s the cause for his presentation?
A-Aspiration
B-Esophageal perforation

208-5 months old had bilateral inguinal hernia what&apos;s the apropriate procedure?
A- laporosopic repair with mesh
B- herniotomy

209- Child with bilateral undescended testis, absent iris, .. Etc what will you also find? (365)
Nephroblastoma

210- What is the most appropriate alternative treatment for severe depression ? (242 , 436)
Electroencephalography therapy
Electroconvulsive
TCA
psychotherapy

211- Primi 22wk GA cervix is 1.5 cm dilated fetal membranes herniating management fetal
viable (i think there was no contractions I forgot sorry ) Tocolytic
Progesterone
I forgot the rest

179
Weird long scenario about a child i think girl with decreased appetite and fatigue , she had
pallor and splenomegaly , nasal polyp and no mention of any respiratory symptoms , blood
labs showed only IDA , normal retics no signs of
infection Provided this same exact picture ,what should be replaced ? Iron
Folate
VitB12
Erythrocytes

213- Scenario of pt with PUD syx and urea breath test was +ve and started treatment
What is the most appropriate to monitor response to the therapy ?
Endoscopy and biopsy
Urea breath test
Clinical symptoms
Blood serology

214- Scenario of pediatric with GI bleeding initial image was normal and asking what is the
most appropriate to know the source of bleeding? Barium swallow
Barium meal
Radioisotope
Ct or MRI not sure

217- Pt prepared for CS she told the nurse she don’t want to get blood transfusion during
the CS bleed why u should do?
-Try to get consent before unconsciousness
- take consent from housebound

218- Pregnant case of epilepsy for 2 years and poor control despite medication , what will
you do?
- review the medication

219- Most common cause of nephrotic syndrome in paediatrics? (102) Minimal change
disease

220- What is the murmur heard in ASD? Fixed split S2

221- 7 years old female normal breast development and coarse public hair? (99) Complete
androgen insensitivity

222- trauma to the jugular foramen? (99 , 414) Causes ipsilateral vocal cord paralysis

223- non caseating granuloma, how to treat? (99) Corticosteroids

224- Infant with continuous Machinery murmur Came with symptoms of heart failure chest
x- ray shows wet lungs what will you do first ?
A) give diuretics

180
B) prostaglandin inhibitors
What is the initial management?
‫ بین قوسین‬NSAID ‫ جاب نفس ھالخیار و حط‬inhibitors prostaglandin
‫انتبھو الیخدعكم‬

225- Milstone : 1- stranger + anxiety away from parents


6
7
10
12

226- pincer grip


6
9
12

‫ واحد عنده انیوریا وش اول شيء تسویھ؟‬-


227
Iv fluid
Abg
Ct

228-
‫ سم وش تسوي‬٦ ‫واحد جاه السیریشن فراسھ‬
Depridment + 1ry
Secandry
‫نترك الجرح مفتوح اتوقع‬

229
‫ نفس السیناریو‬TTN ‫جات سؤالین نفس بعض عن‬
‫ والثاني فل تیرم ؟‬preterm . ‫بس االول كان‬

230-
? uti ‫سؤال وش افضل شيء تشخص بھ لل‬
Sprapupic aspiration
Mid stream
Cath

231-
‫ نازلھ وش اول شيء تسویھ‬refill capilary ‫ وحدة طاح ع رجلھا شيء جات المستشفى وعندھا ال‬Cta
Diplex
Compartment measurs

232- ‫ حق‬pms ‫ عندھا ایام‬٨ anixity + mastalgia

233-
‫ وش ممكن یصیر ؟‬syndrom shock septic ‫ سؤال واحد بعد عملیة جاتھ‬Hypokalemia

181
Res acidosis
‫ او شيء كذا‬division cell Anearobic

-237
‫ وعنده‬not or def iron ‫ ومو محدد ان عنده‬hemolysis ‫ واحد كأن عنده ؟‬test ‫ وش افضل‬spherocytosis
Prehral blood film
Hb electrophoresis

238
dichoriinic monoamniotic 238- ‫جاء سؤال عن‬
0-3
4-8
9-12
>12

239- Lactating mother , erythema and pain in right outer breast , not fluctuating ,
management?
I&D
Aspiration
Flucloxacillin
mastitis ‫ او ال لكن من الكیس كأنھا ماشیھ مع‬mass ‫نسیت الصراحھ اذا فیھا‬

240- 70 yrs old male, inguinal hernia in scrotum


with mild pain ,
By examination the doctor can reduce it back throw the inguinal canal Management?
A.Herniotomy
B. Herniorhaphy
C. Tension- free with mesh
D. No treatment

241- Elderly patient post abdominal aortic aneurism repair by 8 hr found that urine output
50 cc
What to do
A-urine catheter
B- hydration
C- dialysis
D- ‫ تحسب انك معناه ما في‬urine out put and electrolyte

254- Pt right upper quadrant pain amebiasis (dx given ) imaging report > right hepatic lobe
abcess 10X15 cm poorly defined septations , most appropriate initial tx ?
Percutaneous aspiration
Metronidazole

182
Rifampin (not sure)
Surgical (not sure)
255-
localize ‫ مني متذكرة بس االنفكشن كانت‬diverticulitis ‫و فیھ سوال عن‬
‫مطلوب مانجمنت‬
A-Conservative and antibiotics
B-not remember

256- 25-year-old man presented to the Emergency Department with severe pain during and
after defecation for 3 days associated with passage of a small amount of fresh blood after
defecation. Physical examination confirmed an acute posterior anal fissure. Digital and
proctoscopic examination were not performed due to the anal pain. Which of the following
is the most appropriate management? A. examination under anaesthesia
B. lateral internal anal sphincterotomy
C. chemical sphincterotomy with diltiazem
D. botulinus toxin paralysis of anal sphincter

257- Female 18 year old came to Gyne doctor asking for pap smear, what to tell her about
Pap smear?
A⁃ when she is sexually active.
B⁃ At age of 21.
Answer : B

258- Highest risk factor for esophageal ca?


A⁃ Barrett’s esophagus
B⁃ smoking age
Answer :
For Adenocarcinoma , Barretts
For Squamous ca , smoking

259- Baby 8 weeks old was otherwise fine is coming today with fever, wheezy chest,
tachypnea and cough for 4 days. He was otherwise well apart from pansystolic murmur.
CXR revealed increased bronchovasculalr markings and increased heart size. What’s your
immediate action:
A- IV Abx.
B- Steroids.
C- Refer to surgery.
D- Medical treatment with diuretics
Answer : D

260- Same scenario, what’s your diagnosis:


A- VSD
B- ASD
Answer : A

261- Pregnant lady everything was normal but she complain of general itching and yellow
sclera what the most important investigation?
A- Bile acid and LFT.
B-US.
Answer : A

183
262- Child complain from fever and sore throat, had contact with child has flu-like illness ,
in examination only hyperaemia of throat , Rapid antigen is negative and swab culture was
taken. What you will do
A- supportive treatment
B- antibiotics pending culture results ( written like this exactly) C- Single dose IM
ceftriaxone.
D- Start ampicillin 10 days regardless to culture.
Answer : B

263- Screening for anemia is of significant importance, especially in high risk infants as
they are more prone to iron deficiency.
At which age should most infants be screened for anemia? (106 , 448)
A. At birth
B. 4m
C. 9m
D. 12m
Answer :D
High risk:4
Most :12

264- 26 Years old female came with sudden pain for 12 in right iliac fossa and periumbilical
, there’s tenderness over right iliac fossa and suprapubic no rebound tenderness
In lab there’s High WBCs what you will do?
A- Transvaginal US.
B- CT abdomen
C- emergent appendectomy
Answer :B

265- Pt with ovarian mass ask about the tumor marker ? A-B hcg
B-CA 125
Answer: B

266- Mother bring child preschool for vaccination, most indicated (no other detail):(33 , 423)
A-OPV
B-Hib
C-Rota
D-MCV4
Answer: A

267- Pt presented with palpitations, diagnosed now with AFib ( given). Next most
appropriate investigation?
1. Carotid US
2. Thyroid Function test
3. Exercise tolerance test
4. 24 ECG monitoring
Answer: 2

268- White vaginal discharge with itching for 3 days:(86 , 46)


A- BV

184
B- trichomoniasis
C- candida
D-gonorrhea
Answer: C

269- Pt has ve fever and didn’t eat for 1 day


BP : 64/42 ‫متأكد‬
A- discharge the pt
B- admtion for observation fever and discharge if become afabril C-advice the mother to
come back if happen again
D- admission for Iv antibiotic and culture
Answer: D

270- Open eye to the pain . Incomprehensible sound . Flexion to pain (184)
A-5
B-6
C-7
D-8
Answer: C

271- Copd and cor pulmonale


Which of the following Improve mortality
A-Multaknin
B-Inhaler steroud
C-LABA
Answer is : oxygen therapy

272- Elderly man complains of urinary retention, had back pain in the past months. High
PSA.
Possible diagnosis?
A -Prostate cancer
B -Benign prostatic hyperplasia
Answer: A

273- Went for myomectomy of subserosal fibroadenoma, but opened to the uterus.
Risk for placenta acreta?
A -increased
B -decreased
C -Unknown
D -not affected
Answer: A

274- Placenta is deep in the uterus,


Which one is it?
A- Placenta increta
B- Acreta
C- Percreta
Answer: according to the depth

185
275- Trauma patient, currently well. lucid interval reported by paramedics, now he is
deteriorating. Most likely diagnosis?
A -Epidural hematoma
B -Subdural hematoma
C -Base of skull fracture
Answer: A

276- Hard mass in right thyroid,


Turned out to be Papillary thyroid cancer, management?
A -Hemithyroidectomy
B -Subtotal thyroidectomy
C -Hemithyroidectomy with isthmusectomy
D -Total thyroidectomy
Answer: mostly D with some exception

277- Stab injury to arm, Nerve and tendon cut, Clean wound. Management?
A -Primary repair
B -Debridment and secondary closure
C -Debridment and prinary closure
D -Debridment and secondary graft
Answer: A

278- Open fracture initial management? (Was dirty, ) A -Antibiotics


B- Debridment
C -Internal fixation
D- External fixation
Answer: A

279- Pica, pale, (nothing about socioeconimic status) Investigation?


A -Iron studies
B -Lead level
Needs more information♀

280- Pediatric case came with cardiomegaly and chronic congestive heart failure
Lower limb edema and sign of right side heart failure what to give ?
A⁃ increas the dose of digioxn
B⁃ Start fusomide
C⁃ Reassure it is part of disease
Answer: B, BMJ: Diuretics should be considered for patients who have evidence of, or a
prior history of, fluid retention.

They should generally be combined with an ACE inhibitor and a beta blocker. All patients
with symptoms and signs of congestion should receive diuretics, irrespective of the left
ventricular ejection fraction (LVEF).
281- Features to be confirmed test ?
A⁃ Easy to preform
B⁃ Non invasive
C⁃ Sensitive
D⁃ Specifically

186
Answer:D

282- Pregnant lady came with polyhudromnes what is the casue ? A⁃ trisomy 21
B-IUGR
Answer:A , came w duodenal atresia

283- Injury in the 3 neck zone ?


A-vessel ligation
B-Endovascular intervention
Answer: B

284- Pt came after road traffic accident with thigh wound and bleeding , how to manage?
A⁃ apply direct pressure on the wound
B⁃ Apply pressure above femoral artery
‫ قد شفتھ بالتجمیعات كذا كانت بدایتھ‬....tuo Apply⁃ C
Answer: A

285- 5 weeks baby boy , complining of progress projectial vomiting , ask about fluid
replacement?
A⁃ normal saline
B⁃ Potassium chloride
C⁃ Ringer lactat
Answer:A

286- Pregnant lady 38 weeks gestational age came with PPRM for 24 h no other compline
ctg is reassured and no contraction with no cervical dilation
What is appropriate management ?
A⁃ Induction of labor
B⁃ Emergency cs
C⁃ Observation
D⁃ augmentation of labor
Answer: D

287- Female with suprapubic pain with purulent discharge . Vaginal Ex tenderness in fornix
A-Acute cervicitis
B-Acute salpingitis
C-Acute appendicitis
D-chronic appendicitis
Answer: B

288- Normocytic normochromic + inclusion body


A- lead poisoning
B- SCD
C&D not related
Answer: B

289- Normocytic normochromic + inclusion body (288)


A- lead poisoning

187
B- SCD
C&D not related
Answer: B

290- HCV most common genotype


A-1
B-2
C-3
D-4
Answer: D

291- Hcv risk of transmission


A- 0.3 %
B- 3%
C-30%
Answer: B

292- 14 years old didnt have vericella vaccine what you do for her?
A- no need for vaccine
B- give 2 doses 4 weeks apart
C- give 3 doses 0,1,3
D- give one dose
Answer: B

293- adolescents and adults (>= 13 years) without other evidance of immunity ?

* give 2 dose 4 to 8 week apart


* if it has been more than 8 weeks since the first dose , the second dose may be given
without restarting the schedule

294- Infant with pulging in the umblical region appears when crying and disappear when he
asleep what is the managment?
A- Herniotomy
B- Conservative
Answer: B

296-A- wilms tumor (theres no neuroblastoma in choices)


B- ..
Answer: A

297- 30s? old diabetic pt T1DM on insulin, asymptomatic, came for f/u. Urine analaysis
shows (+ve for glucose and urine)
What to add? (88)
Diuretic
BB
ACEI
cant remember
(Im not sure if the vitals showed HTN)

188
298- Morbid obesity, no relevant past medical hx, consult to do biriatric surgery .. which
investigation should be done to choose appropriated procedure? (89)
-EGD
- CT abdomen
- Abdominal US
- not remembered

299- Pt w T1DM, has recurrent episodes of hypoglycemia, (he do some intensive exercise )
on aspart and galrgine... what to do? (90)
- decrease aspart and continue galrgine
- increase pre exercise protein meal and continue ....
- decrease both
- forgot

300- Coca cola urine + rash + had congestion throat and lymphadenopathy (didn&apos;t
specify the duration)
Vitals (fever 38, Tachy)
Likely dx? (91)
IgA nephropathy
Acute glumornephritis
Acute cholecystitis
Not remembered

301- Alcoholic, smoker, upper abdominal pain radiate to back (pic of abdominal xray shows
(collapsed something)
Dx? (92)
Pancreatitis
Duodenal perforation

302- Flank pain, no other sx or signification in hx , most useful test?(94)


Ct scan (wrote like this)
Xray KUB
Biopsy
Not remembered

303- Tx of Bethesda IV: hemithyroidectomy(93 , 443)


BI-RADS: 0 > further imaging

Case abt Alzheimer dx (easy)

Parkinson dx (easy)

304- Tx of ITP?

305- lithium in bipolar case (18 , 154)

306- side effects of tocolytics (palpitations)

307- started neuroleptic medication at risk of ?


Akathsia

189
Seizures
MI
Hyperthermia

308- Trichomalacia scenario

309- Laryngomalcia scenario

310- 80 years old bed ridden has pelvic organ prolapse appropriate management?
Pessary
Le forte colpcelesis
Others

311- Pakinson case where’s the lesion ?

312- diabetic started life style modification came back with high HbA1C what to add next
(bigunide)

313- Tumor lysis syndrome (hyperk hypoCa) (452)

314- risk developing GERD 30% (521 , 104)

315- hemangioma what to advise

316- liver cases 1 what is prognostic

317- ICU patient receives a lot of fluids and inortrop now has high LFTs (ischemic hepatitis)
(362)

318- SLE patient + joint pain ( HCQ+MTX) she was already on steroids. 319- volume
overload case

320- child with excessive thirst family history of same issue DI


DM

321- Chads2 (DM, HF, HTN, stroke) (390)

322- Chest pain and everything normal except decreased DLCO (PE) 323- Lung cancer
patient had repeated tapping and still has effusion what’s better?
chemical Pleurodiesis

324- Double effect scenario

325- Surrogacy scenario (334, 87)

326- Malaria prophylaxis

327- Primary hyperparathyoroidism

190
328- developmental milestone expected in a 3 year old ?
a. climbs stairs
b. catches ball with foot
c. hoping in one foot
d. tell story

329- 2 or 3 yo with swelling behind the ear , and FNA showed was clear lymphatic fluid ,
what to do ?(3)
a. surgery
b. sclerotherapy
c.radiotherapy
d. chemotherapy

330- pedia pt have tachypnea , runny nose , cough , slightly elevated fever , audible
wheezing sound whats is the devinitive dX?
a. chest x ray
b. nasopharyngeal swab
c. sputum culture
d. CBC

331- They suspect hypovolemic AKI what you’ll see ? A-BUN/Cr >20
B-FENA >2%

332- Elderly with massive lower GI bleeding, most common diagnosis? A. Angiodysplasia
B. Diverticulosis
C. Colon ca

333- 10 year old boy presented to the pediatric clinic with a history of skin rash. The
parents report that the symptoms had started with a single red spot over a small scratch
and have spread over the last 4 days . There are several lesions on the cheeks. The trunk
and the upper limbs. Physical examination confirms swollen lymph nodes in the arm pits
and groins
Blood pressure 100/65 mmHg
HR 80/min
Respiratory rate 24/min
Temp 37.4 c
Which of the following is the Dx?
A)Impetigo
B) candidiasis
C) Herpes simplex
D) Hepatiformis dermatitis
‫جاتني الیوم وحاط صورة معھا‬
‫سوال‬

334- infertility couples, surrogacy option, and not available in the hospital and doctor is not
sure about if it is legal in saudi arabia (87 , 325)
A- tell them it is not available option in this hospital

191
B- refer him to other consultant with connection to surrogacy centers C- tell them it is illegal
in saudi arabia
D- give them appointment and read about the topic

335- Couples coming for preconception visit, the husband has a sickle cell trait and two of
his siblings, the wife has no family history of sickle cell but she has not been tested, they
are asking what is the chance of SCD in their offsprings?
A- Increased risk of sickle cell
B- if the wife tested negative there will be zero chance of SCD in their children

336- Type of RTA in sjogren syndrome (190)

337- which vessel injury in epidural hemaoma

338- Sergen change some of study ? Falsification or fabrication (424)

339- 6 ys Pedia complain of RT lion mass had previous episode of vomiting and ABD pain
and bloody urine

340- Ix Anemiac protenuria and HB


What is Dx ?
Nephrotic syn ,or neuroblastoma
‫ناسي باقي الخیارات‬

341- treatment of neuro SLE (343)

342- Female taking NSAID for her osteoarthritis, in vitals she has high BP, labs provided
and there is low potassium (2), what is the most likely diagnosis? (387 , 95 )
A- Essential HTN
B- NSAID-induced HTN
C- Primary hyperaldosteronism

343- pt with SLE have neurosycatric symptoms what is the initial mx for him ? (341)
A-Steroid+ cyclophosphamide
B-Iv cyclophosphamide

344- 27 year old patient 3 days post sleeve gastrectomy, he is presented with mild RUQ
pain,upon examination there was mild tenderness with no signs of peritonitis vitals:
BP:100/80 HR:133 what is the cause?
A- gastric leakage
B- sepsis
C- decreased oral hydration
D- inadequate analgesia

345- A 30 weeks gestation, she has lower abdominal pain with NO bleeding or leaking, if
we suspect preterm labor, which of the following will confirm preterm labor? (127)
A- History would be enough to confirm preterm labor
B- Presence of contractions in the CTG
C- Pelvic examination to see the cervix

192
346- Which of the following is consistent with rheumatic heart disease? A- Mitral stenosis
B- Mitral regurgitation

347- a 68 year old patient was recently diagnosed with hepatocellular carcinoma ..
What&apos;s the next step?
A-colonoscopy
B- chest CT
C - CT liver
D - Surgery

348- Long scenario about a young woman with Abdominal pain...with a ultrasound finding..

A- Endometrial thinking
B- ovarian cyst

349- A 9 weeks gestation, presented with minimal vaginal bleeding and abdominal cramps,
on transvaginal ultrasound: Crown rump length equals 9 weeks gestation and fetal
heartbeat is positive. What is the most appropriate next step? ( 382)(This is the full
scenario, no vitals no labs) A- Pelvic assessment (Literally like this)
B- Admit for observation
C- IV progesterone
D- Ultrasonography for placental localization

350- The question of the child with widening of the joints, I remember the question with
numbers exactly:
Ca 2.2 (Normal 2.2 - 2.7)
Phosphate 2.1 (Normal 1.3- 2.3)
ALP >1000 very high
It was exactly like this, no PTH given
What is the most likely diagnosis?
A- Hypophosphatasia
B- Renal osteodystrophy
C- Vit D deficiency rickets
D- Familial hypophosphatemic rickets

351- Pregnant 8 weeks GA come to hospital with spontaneous abortion, and ask you what
most common lead to spontaneous abortion in this GA
A- incompetent cervix
B- chromosomal abnormalities

352- RTA patient was transfused with 4 L of blood, which of the following would be the
complication?
A- Citrate toxicity
B- Hypocalcemia
(Both answers were within options, others were irrelevant)

193
353- ‫ صورة جاتني‬sucking reflex
‫جاني سوالین‬

354
croup ‫ ومكتوب بالحرف انھ اخذ العالجات حقت ال‬croup ‫سناریو طویل وتعالج على انھ المناسبھ وال تحسن وش تعطي‬
A- antibiotics
‫ مع دوز‬predenesoln- B
‫ مع دوز‬dexamtheasome- C
D- recimic epinephrine

355-
‫والسوال الي بعده نفس السناریو قال وش تطلب لھ‬
A- lateral neck x-ray
B- chest inspiration and expiration x ray
C- visualize by laryngeoscopy
D- chest ct

356- pt diagnosed as trachiomalecia ask about what is the best diagnostic tool (496)
A-bronchoscopy
B-ct chest
C-x ray

357- Man whats to travel and seeking travelers diarrhea treatment labs were given showed
abnormal KFT what treatment to give ? (453)
A. Fluco....
B. Bismuth
C. No need
D. Probiotics

358- Patient has asthma on medications and controlled, he complain of sudden stridor and
i think wheeze? and other symptoms, what to do?
I chose otorhino evaluation

359- a 45 YO male pt with obstructive sleep apnea . which of the following is the most
effective way of treatment in adults?
a. CPAP
b. reduction of BMI to less than 30 kg/m2
c. upper airway surgery
d. mandibular advancement

360- Postpartum female who is not sleeping and not eating, she has postpartum
depression what, way to optimize her treatment?
Involve family members

361 - 22 YO female pt presented with meningeal irritation signs , CSF showed : high WBCs
, maily lymphocytes . mildly elevated protien , normal level glucose , your most likely
diagnosis ?

194
a. diabetic coma
b. TB meningitis
C, viral meningitis
D. bacterial meningitis

362- 50 YO female k/c of IHD and DM admitted to ICU with severe pneumonia and was
treated with ABX . after 3 days of admission , she developed hypotention and was treated
with hydration and inotropes . on admission his labs were normal. after 3 days , LFT was
abnormal , total bil 20 , AST 1600 , ALT1945 , LDH453 , US unremarkable , what is most
likely diagnosis ? (317)
a. ischemic hepatitis
b. intravascular hemolysis
c. ICU related jaundice
d. acalculous cholecystitis

363- young male patient presented with raised liver enzymes , bilateral cataracts and
ketones in his urine , which one of the following is the best investigation to diagnose this pt
?
a. abdominal US
b. liver biopsy (galactosemia )
c. abdominal CT
d. MRCP

364- 30 YO female medically free presented with jaundice for 2 weeks , no flapping tremors
or stigmata of chronic liver . AST 1400 ALT 1300 GGT 70 total bil : 60 , what is the best next
step ?
a. discharge with F/U
b. admit and start IV dextrose
c. order pt and INR
d. refer her for liver transplantation

365- 56 YO patient had an upper GI bleeding due to esophageal varices and was given iv
fluid , which should be given to reduce mortality ?
a. vasopressin
b. octreotide
c. nadolo
d. ceftriaxone

365- (case of WAGR) Child with painless mass and absent iris and undescended testis,
what is the diagnosis?
Nephroplastoma

366- A female presented with back pain and fever, she was given NSAIDs and advised for
bed rest at home, then she presented with inability to move her lower limbs, O/E there was
tenderness on the midback.
Spinal MRI: Diffuse densities on T6,
What is the most appropriate next step?
A- Reassurance

195
B- Specimen from T6 vertebra
C- Brucella titer
D- Bone marrow aspiration for culture

22 September
- Female she came complaining of itching and bad smell after delivery and episiotomy, she
went to a doctor he suspect uti and gave her antibiotic but still she didn’t improve, then she
went to another doctor after examination her said there is a missed gauze inside, what is
the mistake that the first doctor did?
A- he let the midwife help in delivery
B- second doctor communicate result to the first doctor


"‫ "تشخيص حقيقي والفرق بين الدكتورين كيف فحصوا‬C- genuine something
D- didn’t check the safety protocol before leaving OR

- A long scenario about an infant who’s at high risk of developing IDA. The question was, in
such patients who have ((high risk)) they’re screened at which age?
A. 12 months
B. 4 months
C. 8 months

- Child has scrotal pain , on exploration found cord is edematous and inflamed with red right
hemiscrtum:
A) testicular torsion
B) Inguinal hernia
C) testicular appendages torsion
D) epidiymoorchitia

- Case of child with confirmed pyloric stenosis, vomiting, labs show Hyponatremia and
hypochloremia + Metabolic alkalosiis . What is the best initial step?
A - NPO
B - NGT
C- 0.9% NS

- Patient with pyloric stenosis and olive shaped mass, resasitation done, what’s the definitve
management?
A- Pyloromyotomy

- Female brought her baby to the well baby clinic, he is still formula/breast fed, the doctor
advised her baby on iron supplement 1 month after this appointment. How old is the infant
now?
A- 1m
B- 2m
C- 3m
D- 6m

- case and mention the diagnosis was pyloric stenosis, what is the diagnostic test?
A- X-ray
B- Ultrasound
C- Endoscopy

196
- 30-year- old female patient c/o right cervical lump.
O/E : she has enlarged cervical lymph node with normal thyroid gland. Biopsy taken ,
Histopathology reported (follicular thyroid cells) what is the diagnosis :
A- metastasis of thyroid gland
B- Follicular thyroid carcinoma
C- Papillary thyroid carcinoma
D- Lateral aberrant thyroid tissue

- old man after episod of cuogh have SOB and abdominal pain on Xray there is air fluid
level in neck and under diaphragm and left pleural effusion Pt vital Bp: 90/65 HR : 115 RR :
25 O2 sat : 90 on room air what is most important management?
A- bulla of lung
B- laparotomy
C- chest tube drainage
D- observation

- Woman complaining of spider veins on her hips for cosmetic reasons, no symptoms and
no other varicoses, How is the ddx established?
A-Compression ultrasound
B-Venogram
C-Clinical diagnosis no need more investigation

- Infant age of screening for IDA At birth


A- 2m
B-4m
C- 12 m
D- 10

- Child had Type1 diabetes and was consulting the family regarding celiac screening, which
of the following is true:
A- Screen at diagnosis then every 5 years
B- Screen at diagnosis then every 2 years if asymptotic
C- Screen annually
- Case of bedsores ttt before 3 day

now pt have infected bedsore greenish discoloration and bad smell pt confusion and low
BP:
A- Septic shock

Fetus with breach presentation flexing hip and extended knees, what is the position?
A-Frank breech.
B-Complete breech
C-Incomplete breech

case of DM on medication, she has been using NSAID to relieve RA symptoms, Na normal,
K below borderline, Cr higher than normal, vitale Bp: 150/100
what do think the cause of HTN?
A. Essential hypertension
B. pheochromocytoma
C. NSAID induced HTN

197
D. Primary hyperaldosteronism

A 45- year old man presents to the emergency department with a 3 day history of fever,
productive cough of yellowish sputum of yellowish sputum Medical history significant for
type 2 diabetes melitis. He is a smoker. His chest examination reveals decreased breath
sounds throughout and crackles at the right lung base . Other a systemic examination is
unremarkable.
Chest radiograph shows -Right lower lobe infiltrate
Which of the following is the most appropriate management?
A. admit him start Ceftriaxone & Azithromycin
B. Admit him, start intravenous amoxicillin
C.Outpatient treatment with azithromycin
D. Outpatient treatment with CEFUROXIME & AZITHROMYCIN ✅
-12 years old girl with enlarged thyroid nodule everything is normal?
A- Reassurance
B- Repeat fna
Other recall:
Girl with 4cm thyroid lump normal TSH FNA inadicuate what to do; A-Follow up
B- Repeat FNA
C- Thyroidecromy
D- US

Baby i think 1 year with history of 4days URTI bresented with increase hypoxia and sever
ryspiratory symptoms auscultation: fine crackles all over the lungs xray showed puffy
infiltrates bilaterally :
A- Bronchopneumonia
B- Bronchiactesis
C- Forgot other option

2 month baby with fever , cough , fine crackles , on xray puffy infiltration A)
bronchopneumonia
B) cystic fibrosis

-Teen came because of scrotal pain. On examination, the scrotum was high riding, and
tender to palpate. Next?
A. Explore
B. Elevate testes & IV Abx
C. Doppler

70 yo male had Endoscopic variceal vein ligation 1 month back, what’s the best long term
prevention for rebleeding?
A-Beta blocker
B-H2 blocker
C-PPI D-No need

- solider will go to south region what malaria prophylaxis to give?


A-Atovaquone/proguanil

- Pt with liver cirrhosis have UGIB what to give before endoscopy

198
A- octreotide
other recall :

- Patient k/c of Hep C came with oesophageal varices, ten ascites and drowsy. Which drugs
will reduce mortality?
A. Pantoprazole infusion B. Octreotide
C. IV ceftriaxone
D. Blood transferred

- female pregnant didn’t remember his LMP ? Crown rump length


Other recall:
Female present for regular check up in clinic, she didn't remember when was her last
menstrual period, what is the most accurate way to calculate the
gastrointestinal age ?
A-History and examination of utrus funds length
B. Bitemporal lengths
C - Femur lenght
D. fetal crown-rump length.

- How to know fetal weight intrapartum in 37 w?


A- femur length
B- head circumflex
C-Biparietal diameter
D- abdominal circumference

- 38wks delivered by cs presented with tachyepnia tachycardia?


A-TTN
B-ARDS
C-MA

- Divorced 20yo when to do pap screen?


A. After 1 yese
B. No need
C. Now

- An or 25 y/o female worried about cervical cancer. She took her first dose of HPV vaccine
3 months ago. What the best thing to do at this visit today?
A-Schedule app after 3 months
B-No need to do anything at this visit
C-Give 2nd dose at this visit D-Repeat 1st dose

- Woman 35 years old , smoker 10- 15 cig per day , has history of sub fertility and has family
history of diabetes, which is the strongest factor for the fertility ?
A-Age
B-Hx of subfertility
C-Family hx of dm
D-Smoking

- years old patient come with medial leg ulcer. The most likely diagnosis is:
A. Diabetic

199
B. Venous hypertension
C. Atherosclerosis
D. Buerger&apos;s disease

40- yo girl presented complaining of SOB and exercise intolerance and 11 !weight. on
examination there is gallops rythem BP: high 150systolic Hr: 160 RR:40
whats the best next management ?
A-TSH
B- cBC
C- Electrolytes

79-Female with fishy-smell discharge, analysis: PH: 5.1 What type of cells will you see?
A- Multinucleated giant cells
B- Overproduction of lactobacillus
C- Single nucleated cells Review
D- Granulated epithelial cells

- Which is a clue for the diagnosis of eosinophilic esophagitis?


A.excessive chewing
B.snoring
C.pain at night Forgot the last one

- Child came with oral white lesion on the base of the tongue and soft palate, pediatrician
prescribed him nystatin, what’s most likely dx?
A. Erythematous candidiasis
B. Oral thrush

Pt present with vaginal discharge, itching and urinary symptoms I think they also mentioned
that the discharge was cottage cheese like Asking about the treatment?
A) topical anti fungal
B)topical antibiotic
C)oral anti fungal
D) oral antibiotic

- Diabetic lady present with viginal oderless discharge ,edematose labia major Microscopic
show spores
A- candida albican
B-pudencal

- Which of the following is the species of mosquitoes in malaria?


A. aedes aegypti
B. aedes albopictus
C. Anopheles

- Pelvic Fracture with injury to membranous part of urethra. Blood seen in external meatus.
What is the most appropriate action.
A) Cystoscopy
B) Folly’s Catheter
C) Suprapubic catheter
D) CT pelvis

200
43-Sle case asking about confirmatory marker
A - anti Smith
B - anti ds DNA
other recall:

Patient with malar rash, rynoid phenomenon, joint pain of MCP joints, healed ulcer in top of
2nd finger, proximal muscle weakness pt can’t climb the stairs, what is diagnostic AB?
1- anti ccp
2- anti smith
3- Anti ds dna
4- anti RNP

- A 35 year old lady presents with a left nipple bloody discharge, by imaging it was
suggestive of Intraductal papilloma. What to do next?
A. Central Duct excision
B. Observation
C. Mastectomy
D. Core needle biopsy

Patient with PCOS, what should be investigated:


A-Glucose tolerance and lipid profile
B-Thyroid profile

- which of the following is not part of nephrotic syndrome


A ⁃ HTN
B⁃ Edema
C⁃ Low serum albumin

Patient has trauma, lucid interval:


A- Epidural hematoma
B- Subdural hematoma

-Nuchal translucency on US, what it may indicate?


- turnur syndrome
- cardiac congenital DISEASE

- Woman complaining of spider veins on her hips for cosmetic reasons, no symptoms and
no other varicoses, How is the ddx established?
A -Compression ultrasound
B-Venogram
C-Clinical diagnosis no need more investigation

- Patient RA case on methotrexate now came with erythema and joint pain in the
morningmor than one joint and in hand there is some change in xray , take NSAIDs not
improve, high RA factor, how to treat ?
⁃ Hydroxycholoquine

- The mediator released in burn leasions?


A- Bradykinin

201
- Patient eith multiple truma and mandibular fractures how to maintain airway?
A- Cricothyroidectomy

- Pt recieving IVIG what vaccination can u give?


A- DTap

- Burn patients calculate fluid resus using parkland folmula??

- What is decreased in preeclampsia?


A- Urea
B- Createnin
C- Plasma volume
D- (No platelets)

- 55 yr Smoker came with SOB, he is medically and surgically free, chest xray was normal,
what is the next investigation?
A- HRCT
B- spirometry

- 4 year old can stand on 1 foot for less than 10 sec and can tell story but not ise future
tense and draw square but can’t put cubes together...
A- Normal development
B- Only language delay
C- Global development

- Pedia Patient who had an allergic reaction to a vaccine came today for his next shot but
mother is worried that it might happen again, what do do?
A- Do allergics screening
B- Give vaccination then and wait 1 hour
C- No need for vaccination

- Mother rh negative gave birth to a healthy th positive baby, what do do next?


A- give mother anti d
B- Give baby and mama anti d
C- No need to give because she already took at 28 wks

- Patient with history of PE wants contraception?


A- IUD

- Patient with pelvic pain and yellow vaginal discharge and flagellated protozoa, what
is the organism?

- Female with decrease sensation in left calf and lateral side of the left foot, what is
the level of spinal stenosis
A.L3-L4
B.L5-L5
C.L5-S1

202
- years old with headache, unilateral, preceded with nausea, she has similar attacks 5
times per week, each lasting 3-4 hrs, CT of head is normal, what will you give her?
A. Sumatriptan

The operation is successful and the patient is given postoperative insulin and IV d
extrose. 2 days after the operation he becomes very agitated. Which of the following
is the most likely cause?
A.Water overload
B. Addison&apos;s disease
C. Diabetic nephropathy
D. SIADH

- An 8yearold boy’s parents complain that he has episodes where he blinks multiple
times and becomes okay after that he is conscious and responsive during those
episodes. The most likely diagnosis?
A. Tics
B. Tourette syndrome.
C. Blinking disorder.

- When to stop metaformen?


A. GFR <5
B. GFR <30
C. GFR <50
D. GFR <60

Case of COPD , absent gag reflex (nothing about consciousness ) on sever


respiratory distress Ph:7.25 High pCO2 and low O O sat: 83 What is your next best
step:
A. Intubation and ventilation
B. Non invasive.

A patient with renal failure presents with low calcium, high alkaline phosphatase, and
high PTH. What is the best form of vitamin D to give this patient?
A. calcitriol.
B.vitamin D2
C.vitamin D3.
D.vitamin D1

Non-Hodgkin lymphoma on Tx, developed tumor lysis syndrome, what labs expected
to see?
A- Hyper-kalemia and Hypo-calcemia

patient with fever and productive cough and right lower and middle lobe
consolidation, best therapy?
A. azithro & ceftriaxone
B. azithro & ceftriaxone +oseltamivir
C. azithro & ceftriaxone +thoracentesis.

Ortho case what type of dot prophylaxis?

203
A-ENOXAPARIN 40 mg SC
B-IM ASPIRIN
C-Heparin 10,000 U
D-IV Fondaparinux 20 mg

Case of 14 months old baby with hx of abdominal pain and passing red currant jelly
stool, whats is most common site of this pathology?
A. iliojojnal
B. jojnoilial
C. iliocolic
D. ilioilial

Stabwound/ open wond in hand with nerve and tendon enjary.. Antibiotics given..
what to do?
A debradment and primary ..
B debradment and 2ndry..
C. . ...
D debradment and graf

Post sleeve abdominal pain what to do?


A- CT

Pt had history of ectopic pregnancy 2 years back treated with methotrexate, now
present to preconception clinic , how to minimize her next ?
A. Wait for one year more
B. Folic acid
C. Early detection pregnancy

How can we diagnose the pt as cervical insufficiency based on transvaginal US?


A.<5mm
B.<10mm
C.<25mm
D.<30mm.

Female Patient with nausea & vomiting with each period last one was vomiting with
minimal amounts of blood now she is stable what the management ?
A. Arrange for urgent endoscope
B. Reassure & till her to come back if she had the symptom again.

Lady doesn’t come for visits she is 38 weeks in labor , u felt heart beat in mother
umbilical level , *flexed hips* & *extended knees* ?
A frank
B incomplete
C complete

204
D comound

Greenish color of amniotic fluid during delivery, whats the cause?


A- Fetal distress

Infertalty consltation, Male healthy and athlic Fm opeas , tri for 3 year. what to do ?
A. Do Infertalty infestation for both, and hlthy food & loss wt to Fm.
B. Infertalty infestation for Fm as she....... .
"No start with semen analysis in the other options."

82- 17 yo athlete girl who developed breast later than her friends.Never had her
period..?
A. Hypogonadotropic hypogonadism
B. Imperforated hymen /Transverse vaginal septum
C-Gonadal agenesis
D-Testicular feminization

First indicator for hemorrhagic shock :


A- pulse pressure

45 female pt came to the primary clinic because she is scared of cancer she give
history of smoking which screen you will do for her :
A- Ct chest
B- Colonoscopy
D- Mammogram

Two ECG pictures :


- supraventricular tachycardia - inferior STMI
- Influenza vaccine is very limited so which group need it more : - school age

- pregnant woman I forgot the rest !


Pt with prostatic heart valve + infective endocarditis what is the common organism
Milestone: walk steadily, two body parts , says 6 words and immature pencil grasp :
A. 12 months
B. 14 months
C. 17 months
D. 24 months

55-year-old k/c of bronchial asthma , DM , HTN came to ER with acute episode of


palpitation . HR : 160 bpm with irregular rythm , BP 120/80 SPO2 92% on RA what of
the following is most appropriated next step :
A) Cardioversion
B) Amiodarone
C) Adenosin
D) Propranolol

18 months boy brought by his mother, c/o fever , rhinorrhea since two days and can’t
move left lower limb , O/E left knee swelling and limitation of motion .(No more
informations) What of the following is ONLY ONE investigation can help you :

205
A. Blood culture
B. Joint fluid analysis
C. Complete blood count
D. Knee x ray

35-year- old male brought to ER after RTA c/o right side chest pain. O/E:
consciousness, alert , oriented . Emphysema in upper chest and neck . CXR: Rt side
pneumothorax and pneumomediastinum
What is the disgnosis :
A. Tension pneumothorax
B. Open pneumothorax
C. Tracheobronchial injury
D. Hemopneumothorax

45-year-old male patient underwent appendectomy after signs and symptoms of


appendicitis . Histopathology report : tip of the appendix carcinoma. What is the
appropriate management :
A. Observation.
B. Chemotherapy
C. Radiotherapy
D. Right hemicolectomy

Adult Patient came with hematemesis diagnosed ad peptic ulcer. Therapeutic


endoscopy done and bleeding stopped. Which of the following most appropriated :
A) discharge home on oral PPI
B) discharge home on high dose h2 antagonist
C) PPI infusion for 24 hours then oral PPI
D) PPI infusion for 72 hours then oral PPI

Which of the following most correct about B-hCG :


A) .........
B) high B-hcg during 2nd trimester indicates Molar pregnancy
C) high B-hcg during 2nd trimester is the most sensitive test in case of Down Syndrome
D) B-hcg has no rule after miscarriage

Female with vaginal discharge and in examination there is *red strawberry cervix* :
A. chlamydia
B. neisseria gonorrhoeae
C. trichomoniasis vaginali

20years old not married female want to do pap smear. When to do ?


A.now
B.after one year.
C.when she bcame sexuly active.

Case and Ask about what infestgation to Dx Endometriosis?


A : pibcy

Reduce prolactinoma?
A : Aspire

206
- Average 3 years old what can he do?
A.Triangle
B. Climb stairs
C. Known Week days
D. Can catch a ball with alternate leg

101- Breastfeeding mother with bloody discharge?


102- Breastfeeding mother with pain?

- patient immune compromised what to avoid?


A.Varicell
B.Pertussis
C. PV
D.Influenza

10-year-old child came with fever and vesicular rash on the palms associated with
painful mouth ulcer. The majority of his classmates have the same issue. What
additional findings
you’ll see in this patient?
A.Enlarged congested tonsils
B. Rash on the soles

child with B wheezing and foreign body ingestion (+s&s) what to do ?


A. bronchoscop.
B. Intubation

How to diagnose a 12yrs girl with HTN?


A. Above 90th percentile BP for age and gender
B. Above 95th percentile BP for age and gender
C. Above 140/90 mmHg.

child with *pica*,pale, low socioeconomic state , low hemoglobin hand lead was 2 (in
tha labs resolute)threm, iron profile was normal(in tha labs resolute also), x-ray
report :dense metaphyseal bands , what is the treatment?
A. vit-K
B. D-penicillamine
C. Iron

Mather came to ask when to give Solid food to her paby, she only feed him with
*breastfeeding* no other milk, the Doctor told her she can after one month. What age
the paby new?
A.1
B.2
C.3
D.5

207
Pediatric patient brought by his parents; X ray showed widening of the ends of his
bones (growth plates): Calcium (low) PTH (mildly high) Alkaline phosphatase (very
high) What does the patient have? A.Hypophosphatemia B.Primary hyperparathyroidism
C.Vitamin
D deficiency rickets
D. Renal osteodystrophy .

Lady with bruises because her husband beat her, what to do?
A- reassurance
B- inform authorities
C- talk to the husband

Dr ask you to prepare the RT kidney for op but u are sure it was the LT one what
should u do:
a- tell the surgeon
b- tell chief intern
c- review imaging scan
d- ethical comm

A patient coming from an endemic area for malaria. On Quinine(or Quinidine?!). Later
on, he was admitted to the hospital with Heart Failure. Doctor did not ask the patient
about travel or medication hx. Thus, he ordered the nurse to give digoxin. The patient
went into deterioration. How could it be prevented?
A. Computerized drug prescription system.
B. Nurse failed to write drug formularies.
C. Pharmacist who failed to check prescription.
D. Patient who did not bring his medicine pack/ report his medicines.

120- There was one question about child Pugh score - Female did D&C and now
infertile
A - Asherman ( other options were irrelevant )

most common abdominal pediatric tumor :


A-Glioblostoma
B- Ewing
C- Willms

Pt cant swallow liquids more than solid food What is the FIRST investigation?
A-Upper endoscope
B-Barium swallow
C- Chest CT
D- No manometry in options

PIC of steping reflexes asking about what age it disappears at :


A- 2 m

32 y pt ( i think) did curettage before 2 months, now she comes to complain that her
period didn&apos;t come for the last 2 months , what layer of uterus is damaged by
curettage?
A- functional layer

208
B - basal layer

Pt come with green vaginal discharge , organism is?


A, streptococcus
B pseudomonas

sandpaper like rash organism:


A- Group A steptococcus

child hypoglycemia & seizure & metabolic acidosis & characteristics smell & positive
ketone Diagnosis?
A-: marple syrup

A case of HUS. What’s the diagnostic culture test?


A. Blood
B. Urine
C. MRI
D. Stool

What is true about hydroxychloroquine?


A-Stop during pregnancy
B-Stop during lactation
C-Interferes with Mycophenolate mofetil
D-Interferes with smoking

TB pt with high liver enzymes what will you do


A. stop all
B. stop pyrazenamide
C. stop rifambicin
D. stop INH

46 years old female complaining of joint pain for 6 month, with significantly stiffness
in the morning. Pain now localized on MCP joint & PP joint, associated with bilateral
severe knee pain started 3 weeks ago. What is your management? (Complete case)
A- Methotrexate
B- Methotrexate & Hydroxychloroquine
C- Methotrexate & steroids
D- Methotrexate & Hydroxy & steroids

-Most cancer should be screened in population ?


A. Colon
B. Lung

-Child present with sever dehydration and fever ( sunken eyes and decrease skin
turgor ) septic work up ordered, what to do next ?
A- 20ml/kg IVF bolus
B- Maintenance IVF until results come back

-Infant brought with signs of measles In immunisation record, the father said that he
refused the vaccination (or he is against vaccination)

209
A. Inform child protection
B. Educate the father about vaccines
C. Call ethic committee
D. Focus in treating the disease

Pt had history of ectopic pregnancy 2 years back treated with methotrexate, now
present to preconception clinic , how to minimize her next pregnancy risk ?
A-Wait for one year more
B-Folic acid
C-Early detection or diagnosis of pregnancy

Pt after 10 days follow gastric sleeve develop sob and diminish popliteal pulse what
next step do for this pt??
A.TPA
B.heparin
C.IVC filter
D.aspirin

Breast erythema US 2*3 collection suspect cyst or abscess strat ABX what next?
A-observation
B-incision and drainage

-What’s the type of polyp that has the highest risk of being malignant?
A- Villous

-ECV contraindications?
Toronto: absolute contraindications: where CD is required (placenta previa, previous
classical CD), previous myomectomy, PROM, uteroplacental insufficiency, nuchal cord,
non-reactive NST, multiple gestation

-When to diagnose pedia as hypertension?


A- bp > 120/80
B- bp > 150
C- >90% depends age and sex
D- >95% depends age and sex

Patient post MVA complains of SOB, signs of respiratory distress and distended neck
veins. Chest examination reveals hyperresonance . What is the next step in
management?
A- needle decompression in intercostal space
B- chest tube in intercostal space
C- Thoracotomy
D- tube drainage in intercostal space

-Unvaccinated boy and the mother said the vaccine not good . What is the best next
step ?
A-Explain
B-Ignore and treat
C-Inform child protection

210
-Pregnant in labor and signs of meconium stain how to manage baby ?
A- ICQ Suction Oropharynx before deliver the body
B-NICU
C-TOCOLYTIC
D-Intratracheal suctioning

-Researcher while analyzing data omitted data to change the outcome results?
A- Falsification
B- Fibrication

-Resident didn’t close the curtain and exposed the abdomen ?


A- Privacy
B- Dignity

-What indicates a doctors’ respect of a patents’ autonomy?


A- Patent information
B- Covering patient’s body parts
C- Treat all patient’s equally
D- Patient signing a consent

Pt for surgery, but didn’t stop warfarin and nurse finds out so surgeon postpones:
A- Near miss
B- Adverse event
C- Medical negligence

-Calculate CHADS2 score In 64 yrs old male, DM, HTN, hypercholesterolemia, hx of


ischemic stroke, smoker
A. 3
B. 4
C. 5

Appropriate site for thoracocenthesis?


A: 8-9th ICS

Most common type of leukemia?


A- ALL

Pregnant lady in c/a and doctor couldn’t stop bleeding he calls assistant consultant
and the conclusion is to do hysterectomy to safe her life
A. Consent from husband
B. Do only what’s written in intervention consent (basically do nothing)
C. Do it without consent
D. Idk call someone or something

-A Pregnant , know case of HTN, which is following is contraindicated?


A- Nifedipine
B- Methyldopa
C- Lisinopril
D- Labetalol

211
-A 50-YEAR-old male coming to the clinic, asking about the best way to prevent colon
cancer?
A Low fibre diet
B Annual colonoscopy
C Exercise and healthy life style

-Most accurate method to make sure full placental separation?


A. pressure and massage uterus
B. manual removal
C. wait for spontaneous
D. with a sponge

A pregnant in her 20 weeks found not to have taken MMR what’s the appropriate
management?
A-Give at 28 weeks
B-Give now
C-No need to give

-Most common organism in infective endocarditis (clinical presentation only with


splinter hemorrhage ):
A. Staph aureus
B. Streptococcus

-Pregnant with DIC and placenta abruption with sever bleeding and pain what is the
most appropriate thing to do:
A. Give her blood transfusions
B. Call multidisciplinary team

-How would you assess if there is response to therapy in asthma?


A-ABG
B-CXR
C-Peak Flow meter
D- Pulses paradox

-34 y/o female 30 gestation with painless vaginal bleeding, did vaginal examination
found suspicious mass (see report) Report: US shows that the fetus corresponds to
the Gestational age Next step:
A. Colposcopy
B. Cone biopsy
C. Pap smear
D. Endocervical curettage

-Seven weeks pregnant lady C/O vaginal bleeding with tissue. Her cervix was open
and you can see some product of conception. Her fundal height is equal to 7-8
weeks.
A. threatened abortion
B. incomplete abortion
C. missed abortion
D. molar pregnancy

212
-42 years old female patient want to do full workup cancer screening . She has no
medical, surgical or family history of cancer. what to do?
A- CT chest
B-Mamogram
C-colonoscopy
D- CT chest Mamogram ,colonoscopy

-Government is planning to do campaign to reduce the risk of smoking and CVD risk
factors along with other efforts what kind of prevention?
A- primary
B- secondary
C- tertiary
D- primordial

-Female patient has menorahgia . On pelvis US it showed a Mass. where is the lesion(
exactly like this short question )
A- intramural
B- subserosal
C- submucosal
E- cervical

-Postmenopausal winsome for 3years has on/off bleeding she is taking on estrogen
hormone. How to manage ?
A- add progesterone hormone
B- switch to oral contraceptive pill
C- contraceptive IUD
D-oestrogen patch

-Postmenopausal winsome for 3years has on/off bleeding she is taking on estrogen
hormone. How to manage ?
A- add progesterone hormone
B- switch to oral contraceptive pill
C- contraceptive IUD
D-oestrogen patch

-+Old patient known case of A fib presented with mild diffuse abdominal pain,
hemodynamically stable. What is the best next?
A-US
B-CT ( like this)
C- diagnostic laparoscopy
D- explorAtory laparoscopy
No colonoscopy in answers

-Female patient has a history headache, mastalgia and other symptoms not
remembers for 10 days before periods, all goes away with the onset of her cycleWhat
is the Dx?
A-Primary dysmenorrhea
B-pelvic congestion syndrome

213
C-premenestrul tension syndrome
D-PMS

-Patient came from India with features suggesting TB, what is the best next?
A- discharge
B- Acid fast bacilli
C- TB 4 medication
D- not remembered
Not isolation in answers

-Women after delivery developed uterine inversion and was returned. Which placenta
location increases risk of this condition?
-Anterio wall
-Fundal
-Posterior wall

-Young male previously healthy presented with fatigue, body ache, cough with
whitish sputum and some times associated with blood. Dx?
A- Bronchitis
B- pulmonary edema
C- hereditary hemorrhagic
d. telengiectesia
D-?

-Ectopic pregnancy case , bHCG Day1 1000


Day 4 1400
Day 7 1900
Day 14 2200
What is the most appropriate next step?
1- second dose methotrexate
2- immediate laparatomy
3- perform transvaginal US

-31 w GA Antepartum hemorrhage severe bleeding BP 90/60 “not sure about CTG
findings” Next step ?
1- CS
2- steroid
3- antibiotics
4- vaginal delivery

-scenario for acute liver failure and asking what’s considered a prognostic factor for
this condition?
-pt
-albumin
-bilirubin
-ast or alt (not sure)

214
Q6;eldery female (60s) with rectal bleeding, diagnosed with grade 3 hemorrhoids at 9
and 3 o’clock, what’s next;
-Rubber band
-Hemorrhoidectomy
-Colonoscopy

Q7; eldery smoker k/c of poorly controlled DM comes with ulcers on tip of three of his
toes, diminished dorsalis pedis bilaterally, however, intact popliteal pulse, what’s the
initial management;
-Amputation
-Long term anticoagulation
-Immediate surgical intervention
-Diet modification and lifestyle changes

Q8; case of pertussis (whooping cough, post-tussive vomiting...), definitive Dx;


-Culture
-Serology -Nasopharyngeal PCR

Q-Child who have URTI recently resolved but still C/O dry cough at night given
Alopuritol but no relief , What to give next :
- Cough sedative
- ICS
- oral steroids
- LTR

Q-17 yrs old primigravida came to ER c/o vaginal bleeding and abdominal cramps ,
pregnancy test positive at home US : shows its 9wks and closed cervix with blood
through ,as well as positive heart beat (they describe Treatened Abortion)
“No vitals given” , Mx :
A- Reassure and F/U after 1 wk
B- admit,stablize, prepare for possible termination of pregnancy
C- oxytocin

Q-patient was stabbed in multiple areas, came to the ER and was awake,
his clothes were full of blood, his bp is around 80/60, (O2 sat 88%)which of the
following is
best next step?
A) oxygen supplementation.
B) iv crystalloid infusion
C) Intubation

Q-P.t known case of DM1 came to ER ( with typical DKA ) and then they mentioned
Diagnosed as DKA and IV fluid start what next to give :
1- IV insulin
2- Iv insulin as well as long acting insulin 3- SC insulin
4- sliding scale insulin

215
Q about which of the following is best initial test for Celiac pt?
-Iga tissue transglutaminase antibody
-Anti-endomysial antibody
-Endoscopy

Q about which of the following is best appropriate test for Celiac pt?
-IgA tissue transglutaminase antibody
-Anti-endomysial antibody
-Endoscopy

Q-A question about a 21 YO just got out pf DKA. And ask you insulin plan for her
Type 1 DM.?
2 doses of NPH
Basal-bolus with two doses of basal insulin
*I chose NPH because it is an easier regimen to control and she had DKA

Q-Patient came with epigastric pain radiating to the back for 6 days , he has history
of multiple gallstones, now he is complaining of abdominal pain. Physical exam
confirm he has abdominal distention and sluggish bowel sounds. All labs are normal
( LFT and Bilirubin level).
Erect cxr: left pleural effusion. What is the initial next step?
- abdominal ultrasound
- abdominal CT scan
-exlap

Q-Patient diagnosed with afib 2 years ago , came with transiet ischemic attatck and
hypertension, how will you manage?
⁃ continue warfarin
⁃ Discontnue warfarin
⁃ Continue warfarin and add aspirin
⁃ Discontinue warfarin and add clopidogrel

Q-Patient with jaundice splenomegaly and history of cholecystectomy for stone. Most
diagnostic test
A. Peripheral blood smear
B. Hemoglobin electrophoresis
C. Bone marrow biopsy

St elevation in lead l,aVL,V4,v5 indicate mi in?


-Ant
-Lat
-Inf
-Post

Q-Type ofIsolation in tb case


-Airporn
-Contact

216
-Dropplet

Q-Pt in his 60’s came with flank pain, tender mass in lumber region his vital BP=
160/90 What’s your investigation ?
A-US
B-CT abdomen
C-MRI
D-Radionuclides

Q-Kid with open fracture scenario, ... What to do:


-Morphine
-Call the surgeon on duty
-Give fluid bolus
vitals his BP was 70/
——-
Q-Long scenario of pt with gerd sx and was taking antacids and still symptomatic
(Didn’t say he is fat, didn’t say he eats and sleeps)
Was only describing gerd sx:
What to do :
-Lifestyle modification
-Ppi
————
Q-Case about a lady she leaks urine when coughing but has no dysuria or urgency
Mechanism of this:
-Hypermobile urethra
————-
Q-Case of a lady complain of problems with defecation, she uses her finger to help
her pass stool:
-Ant coloporaphy
-Post colporaphy
-Classical repair
-Removal of enterocele
————
Q-case of ectopic took MTX last week, came now with severe abdominal pain ( no
vitals or anything about stability ) B-HCG is 6000?
-Diagnostic laprascopy
-Give another dose of MTX
————
Q-10 month girl brought by her parents
Due to umbilical swelling bulges during crying and return when she is asleep
-reassure
—————
Q-Child recommended exercise per day
-30 min
-60 min

—————-
Hydatid cyst, 10x15?
-Surgical
-Metro

217
-Per drainage
-Per aspiration
—————-
Q-Most common site for crohns
-Ileum
-Iliocolonic
—————-
Q-Case saying a patient has Afib (not asking about acute management) Question was
how to insure or achieve Sinus rhythm:
-Digoxin
-Bisoprolol
-Verapamil
-Amiodarone
———
Q-CKD pt wants to do CT scan but doctors are worried about his kidney function due
to contrast

-Give N acetyl cysteine


-Give NaCl 0.9% pre and post CT
————
Q-Febrile neutropenia case :

-Blood urine sputum Cx + oral antibiotics


-Blood urine sputum Cx + IV antibiotics
————-

Q-Pregnant had abruption , How to care for her antenatal plan :


-Reclassify her as “high risk” pregancy and close follow up -Reclassify her as “low risk”
—————

Q-Pedia pt has strep throat presentation, next appropriate:


-Throat Cx
-Rapid antigen test ———

Q-Pt has fibroid, doctor told her her would to a procedure to her but will give her
soemthing to reduce the size :
I chose GnRH agonist
———
Q-Pt with burn injury in the right forearm circumferential burn Pulse oxymeter in the
right index is 86, What to do :
-Escharotomy
-Check compartmental pressure

Q-2 yrs Pedia patient meningitis case


Culture showed Gram possitve cocci arranged in chains
-Ampicillin
-Ampicillin and gentamicin -Ceftriaxone and vanco
————
Q-Female patient found to have phylloyes 3 Cm
-Wide (not wire) local exicion

218
-Simple mastectomy

Q-13 months with pneumonia, vitally he was hypotensive and unable to take oral :
what makes hospitalization an option?
- age
- vital signs
- inability to take orally - the pneumonia

Milestone of a child that can jump on both legs makes blocks of 6, says 2 names and
2 wordsi think:
A-1 year
B-2 year
C-3 year
D-4 year

Q-Indications for thoracestomy or ectomy:


A-pleural glucose below 60
B-pleural ph above 7.2
C-pleura that show 90% PMN
D-pleural ldh to serum protien leas than

.Q-Pt came prenatal clinic after couple of week with no mesntrrual cycle she told you
that her last pregnancy was still birth and asking you about vaccines that can
decrease it, you did now pregnancy test it was positive what vaccine will you give?
A-rubella
B-influanza
C-bcg
Ans: B because she is pregnant

Q-Pt wants to get pregnant came for fu before getting pregnant you did thyroid test:
TSH high-
T4 normal
She is asymptomatic doesn’t complain from anything PE was normal, What will you
do:
A- No need for thyroxine treatment
B- give thyroxine immediately
C- give thyroxine during pregnancy
Ans: B

Q-pt has DM how to know or check if he has diabetic nephropathy?


A-hematuria
B-protienuria
C-Hayline casts
D-RBC casts
Ans: B

Q-Women came with severe itching white vaginal discharge and you saw
pseudohyphae what is the treatment?
A-topical flucnazole
B-metronidazole

219
Q-Old patient came to the clinic what vaccine to give?
A- pneumoccoal
B- meningiocooca
C- bcg
Ans: A

Q-Women came for mammo screening whats is the interval between screening from
50 to 75 or 70 i forgot:
A-annually
B-every 2 years
C-every 3 years
D-every 5 years

Q-Pt came with signs and symptoms of TB fever hemoptysis and so on... You did
x-ray and found cavitation, What will you do next?
A-sputum AFB
B-start INH, rifampin, ethambutol and pyrazinamid
There is no isolation in the answers Ans: A to confirm the diagnosis

Q-Chest stabbing trauma present with hyperinflated lung and raised jvp and deviated
trachea to other side
A-tention pnemothorax
B-cardic tamponade

Q-High K ( 6.5 ) with ecg provided peak T wave . What is your management?
- Iv ca glucogonate
- Iv insulin and dextrose Other 2 irrelevant

Q-Patient presented with SOB after trauma , upon exam there was dullness in the left
side . What is most likely diagnosis?
- Tensions pneumothorax
- cardiac temponade
- massive hemothorax
- pulmonary contusion

Q-Pt present to ER after trauma, x ray was done and showed multiple rib fracture,
after 2 days pt developed SOB . Another x ray was done and showed lung infiltration
in the same side of fracture. What is the dx ?
- pulmonary contusion
- flail chest

Q-Patient with MVA came with rips fracture, x-ray on admission show 3-7 rips fracture
x- ray repeated after 2 day showing lung infiltration at the site of fractured rips. What
is the diagnosis?
A-Flail chest
B-Pulmonary contusion

220
Q-Pt came from india with typical symptoms of meningitis, lab shows low glucose
and high wbc , CT done and showed no masses or hemorrhage,
What is most appropriate next step ?
- treat the one who contact with pt
- isolate in negative room pressure
- perform LP for CSF analysis and culture - give ( all drugs of tb written)
- perform LP for CSF analysis and culture - give ( all drugs of tb written)

Q-30 weeker Pregnant with premature preterm rupture of membrane only with no
signs of labour , what is the most important to give ?
- dexamethazone with nifidipine - dexa + mgso4
- dexa + ampicillin

Q-A pediatric with weight in the 6th percentile, height on 0.4 percentile, wrist
widening. What deficiency does the patient have?
A. E
B. B2orB1
C. D
D. A

Q-Pt 4 years old presented with deteriorating nausea vomiting and I think headache?
And visual loss for the past 6 weeks, symptoms increased in the early morning and
late night and gets better when walking
Best modality that will help you to diagnose and treat this condition:
- EEG
- Brain MRI
- EMG
- US abdomen

Q-11 year old boy came with the complaint of faliure to thrive, presented with
polyurea, polydepsia and 3 episodes of dehydration not related to vomiting or
diarrhea. Hx of similar symptoms in the boys of the family
-DM
-Diabetes insipidus
-Water intoxication
-Nephrotic syndrome

Q-Pt presented with history of UC and had 7 days diarrhea. Best IV fluid to give
- Normal saline
-Ringer lactate

32 years female with 3 months of absence period , what is the most appropriate
investigation:
- TSH
-FSH
- pregnancy test
-US

breast abscess 1*2 cm red tender started abx what to do next A- I&D B-observation
C- repeated aspiration

221
Breast mass on US birad 3, What to do next?
-Follow up 6m
-Cure needle biopsy

LONG scenario about anterolateral STEMI (with pic), pt is k/c of DM, HTN, recent hx of
ischemic stroke 1 month back. Labs and vitals were provided.
Asking about the next step in management:
A. Give him magnesium B.
CT angiography
C. Urgent coronary angiography
D. Thrombolytic therapy

Q-Girl 9 or 12 yrs old complaining of recurrent abdominal pain, loss of appetite, loss
of weight, associated with watery diarrhea and some time there's blood.
No vitals mentioned.
What is the most appropriate management?
A. Reassurance
B. Admit for further workup
C. Prescribe PPI and follow up
D. Forgot the last option but wasn't related

Q-Young female pregnant, unbooked, presented to the ER with vaginal bleeding, she
denies any N/V, or abdominal pain. Fundal height exam confirmed a 32 GA
‫يدوبك وصلت عالطوارئ عشان عندها مشكلة بالمواصالت‬
Vitals all normal, What is the most appropriate NEXT step?
A. Perform vaginal digital assessment B. Perform ultrasound
C. Do laboratory investigation
D. Admit the pt to the ward

Q-Female pregnant 29 wks came to the clinic complaining of leakage (literally written
like this). While taking the hx she said that leak is still ongoing with large amount of
fluid. What is the next step?
A. Vaginal digital exam
B. Sterile speculum exam
C. reassurance and discharge
D. Give steroid and ask her to come after 1 week to do something (sorry I forget)

Q2; Nitrazine paper is used to Dx PROM, what other test could be used for this Dx;
-Ferning

Q3; elderly (61yo) present with symptoms of obstruction, imagine showed left side Y-
shaped structure. What’s next;
-Fleet enema
-Colonoscopy
-Sigmodectomy

Q4; 4yo present with high relapsing 9 days fever, swollen erythematous feet and
hands, red lips, erythematous maculopapular rash in buttocks and lower extremities,
Dx;
-EBV infection

222
-Kawasaki
-Rubella
-Measls
(B)
Q6;eldery female (60s) with rectal bleeding, diagnosed with grade 3 hemorrhoids at 9
and 3 o’clock, what’s next;
-Rubber band
-Hemorrhoidectomy
-Colonoscopy

-Female with history of previous abortion, she is having multiple uterine fibroid ,
which of the following will help her the most to get pregnant

A. myomectomy
B. Uterine artery immobilization
C. Medical therapy
D. Something to decrease the size of fibroids
-Pregnant Female GA of 30 present with moderately severe vaginal bleeding US
showing that she had low laying placenta (not vitals mentioned) What is the next
appropriate step

A. IOL
B. Observation C. C/S

Q-Pregnant women found non immunized against rubella when to give here vaccine

- Now
- post partum

223
224
225
30 sep (1-5)

226
Triceps mass what to do? ( no size)
A-Incisional bx
B-excisional bx
C- core needle bx
Answer :A vs C

Degenerative change in leiomyoma during pregnancy?


A- Red
B- cystic
C- Hyalin
Answer: A

Scenario child had typical sign and symptoms of HSP aking about the causative of
limping?

A- knee and ankle


B- pelvic bone
C- sacroiliac
Answer: A

A child with yellowish discoloration


Labs: Ind. bilirubin: 140, Total: 240, Other labs within normal , What is the Dx?
A- viral hepatitis
B- Gilbert syndrome
C- obstructive jaundice
D- acute pancreatitis
Answer: B

Infant in the vaccination clinic, her mother told you he has yesterday hx of diarrhea 3
times otherwise normal no fever or cough. Regarding his vaccines?
Answer: (there is no contraindication to vaccines ->
give them)

Pt in his 60’s came with flank pain, tender mass in lumber region his vital BP= 160/90
What’s your investigation ?
A-US
B-CT abdomen
C-MRI
D-Radionuclides
Answer: B

17 yo athlete girl who developed breast later than her friends. Never had her period..?

A- Hypogonadotropic hypogonadism
B- Imperforated hymen / Transverse vaginal septum
C- Gonadal agenesis
D-Testicular feminization
Answer A

227
Lady doesn’t come for visits she is 38 weeks in labor , u felt heart beat in umbilical
level , *flexed hips* & *extended knees* ?

A- frank
B- incomplete
C- complete
D- comound
Answer: A

Female with vaginal discharge and in examination there is *red strawberry


cervix* :

A- chlamydia
B- neisseria gonorrhoeae
C- trichomoniasis vaginali
Answer :C

Female Patient with nausea & vomiting with each period last one was vomiting with
minimal amounts of blood now she is stable what the management ?

A- Arrange for urgent endoscope


B- Reassure & till her to come back if she had the symptom again.
Answer: A vs B

Elderly, 6 moths history of parkinsonism, early demintia, (forgot the third thing).

A- parkinson's disease
B- lewy body dementia
C- alzhiemer
Answer: B
-----------
Note:
-Lewy body dementia if the onset of both cognitive and motor symptoms is within 1year

-Dementia secondary to Parkinson disease if cognitive symptoms occur > 1 year after the
onset of motor symptoms

Right sided weakness, sensation intact. Which artery is affected?


A- basilar
B- vertebral
C- branch of middle cerebral artery
D- anterior cerebral artery
Answer: C

Prosthetic valve, now with endocarditis, treatment?


A- Vancomycin, ceftriaxone
B- Vanco, rifampin, gentamicin
C- Vanco

228
D- Vanco, rifampin
Answer: B

Pt with family history of diabetes, Fasting glucose high, HbA1C high


initial management?
A- Metformin
B- Liraglutide
Answer: A

20days postpartum, vaginal discharge, white- yellow no smell no itchiness,


microscope: white cell and epithelial?
A- Reassure
B- Culture discharge
C- Urine culture
D- Metronidazole
Answer: A
-----------
Note:
For the first few days after delivery, the uterine discharge (lochia) appears red (lochia
rubra), owing to the presence of erythrocytes. After 3 to 4 days, the lochia becomes
paler (lochia serosa), and by the 10th day, it assumes a white or yellow-white color
(lochia alba). Foul-smelling lochia suggests endometritis.
------------------
Most women pass lochia for about 4 weeks after delivery; in some cases, this lasts for
6–8 weeks.

20weeks Pregnant with rash and jaundice, Medically free, Elevated LFT.Likely
diagnosis?

A- Cholestasis of pregnancy

B- Hepatitis

C- Budd chiari

Answer: A

Known SLE on medication, presents with Lower limb weakness,Exam: paraplegia


and hyperreflexia, investigation?

A- Lumbar puncture and brain mri

B- Lp and brain mri, mra, mrv

C- Lp and ct brain

D- Lp and mri spine

229
Answer: D

The Patient was in a RTA 2 weeks back, Now has symptoms of pneumothorax
(Reduced air entry on right side). (I think the Radiograph image showed reduced right
lung size) Management?

A- Needle

B- Chest tube

Answer: A

------------

Note:

●Patients who are clinically stable and having their first PSP should undergo needle

aspiration if their pneumothorax is large (>3 cm rim of air on chest radiograph), or *if

they are symptomatic with chest pain or dyspnea*

●First-line treatment involves percutaneous needle aspiration. If aspiration fails or the

pneumothorax is large, a chest tube placement is usually required.

----------------------------------

Petient k/c of crohns, having refractory hypokalemia

A- Assess aldosterone angiotensin level

B- Give IV magnesium sulfate

Answer: B

Pregnant 39 weeks, in latent phase of labor (didn’t mention for how long) cervix is 2
cm, CTG shows basal HR 140, moderate variability, no acceleration or
deceleration,mild and irregular contraction.?

A-observe

B-Induce labor

B-Augment labor

D-C/S (not if this choice was there)

Answer: A

230
positive secretin stimulation test

A-VIPoma

B- Gastrinoma

C-Carcinoid

Answer: B

RTA heavy bleeding from thigh, first step?

A- Control bleeding (written like this)

B - thigh tourniquet

524-Post CBD surgery complains of chills and rigors. BP 119/? HR 80 RR 28


T38.Normal WBC ( 11 i think?).

A-sirs

B-Bacteremia

C-Sepsis

D-Severe sepsis

Answer: A

SLE patient having edema and dyspnea high JVP no gallop or heart abnormality.
Lab:high creatinine, low albumin, normal platelet, high ALT and AST. What is the
likely diagnosis?

A- liver failure

B- Renal failure

Answer: constructive pericarditis

47 years old female come with presentation of haemorrhoids and confirm the location
3,7,11 o’clock ,What you will do?

231
1-colonoscopy

2- rubber band ligation

Answer: A

Over 40 years old we must do colonoscopy to role out malignancy,Female undergo


an eventful vaginal delivery then after the placental delivery she bleed 1000 cc but
she vitally stable, her utures was firm. What is the cause:

A. atony of the uterus.

B. missed part in her uterus

Answer: Mostly B

35 female Dx with U.c since (3yrs I guess ) her father has colon cancer Her mother
has breast cancer What she should do regarding screening?

-Mammo at age of 40

-Colonoscopy at age of 50 and -mamo at age of 40

-Should do colonscopy after 8or 10 yrs after her dx of U.C

Answer: C

Although the question is not complete.

We must know the age of relative at diagnosis.

Colon cancer screening for +ev family history = at age of 40 or 10 years earlier which

one earlier

Breast cancer screening for +ev family history = 10 years earlier than age at diagnosis

( AMBOSS )

At age of 30 ( American Cancer Society )

At age of 40 ( USPSTF )

Female 20 years recently develop mass 2x2 cm that is ovale and smooth wall what is
your dx?

232
A - breast cyst

B- fibroadenoma

C - fibrocystic cancer

D - intraductal carcinoma

Answer: B

34 year old with splinter hemorrhage and fever and pansystolic murmer ,No history
of surgery or medical illness, What is the emperic treatmen will give?

A – csftriaxone

B- gentamycin

C - ceftriaxone and vancomycin

D- Can’t remember

Answer : C

Pt with burn and have edema what is the chemical mediator:


A- bradykinin
B- prostaglandin
C- serotonin
D- thromboxane A2

Answer: Mostly B Questions from other groups

Dizygotic twin result :

Mono mono of same sex

Di di of same sex

Di di of different sex

Conjugate twin

Answer: C

male (forgot the age) complaining of difficulty swallowing for liquids more than
solids.Best next investigation?

233
A- barium swallow.

B- endoscopy.

C- Two other choices I can’t remember but manometry wasn’t there for sure.

Answer: B

Typical case of RA, what is the treatment?

A- Prednisone

B- Indomethacin

C- Hydroxychloroquine with weekly methotrexate

Answer:

RTA pt have dullness on percussion over left hemithorax decrease breath sound (
most likely hemothorax ) chest tube inserted( 200 or 250) ml /h for the last 3 hrs
something like that so it’s ongoing bleeding, next step ?

A. Thoracotomy

Answer: A

pt with Persistent productive cough for 3 months with similar attacks previous 2
years dx :

A. Ch. bronchitis

234
B. Lung fibrosis

C. Bronchectisis

Answer: A

A businessman came with neurological symptoms and he had multiple sexual


activities Low WBC and positive cryptococcus:
A. HIV
B. Syphilis
C. Toxoplasmosis

Answer: A

A young boy case came with jaundice, he’s Mediterranean, It makes you think it’s
mediterranean fever but I think it was G6PD because he had high reticulocytes, What
drug not to give?
A. Amoxicillin
B. Aspirin

Answer:

Patient with hx of pancreatitis months ago and now referred to surgery clinic for mild
abdominal pain and , what initial investigation? I think it was pseudocyst
A. U/S
B. CT
C. endoscopy

Answer: A?

pediatric patient hx of recurrent low platelets and eczwma dx?


A. Wiskot Aldrich syndrome

Answer: A

multiple questions asking about what type of prevention (primoridal, 1°,2°,3°)

pediatric with rash all over but sparing the diaper involves extensors and scalp and
face , with reucrrent infections:

Plt low

A-atopic

B- Wiskot Aldrich syndrome

Answer: A?

235
abandoned baby found to have inguinal hernia what to do?
A. call the hospital's ethical committee
B. involve the social worker
C. treat his hernia without consent
D. forgot

Answer: A

- young patient c/o loud heart sounds but all labs, ECG, imaging were
normal,medically free and no cardiac hx (the ECG was not clear), asking about
management?

Answer: No option!

- which antipsychotic causes weight gain?


A. olanzapine
B. clozapine
C. risperidal
D. forgot

Answer: A vs B?

Obese wt 120 ht (not sure) around 174 he tried diet and exercise but didn’t work
A. Bariatric surgery
B. Wt loss medication

Answer: A?

best bariatric surgery for patient with GERD:


A. Roux Y Surgery

Answer: A

Most Common organism in IE in Prosthetic valve less than 60 days:


A. Staph epidermis
B. staph aureus
C. strep lviridans

Answer: B

236
Hashimoto’s thyroiditis mc cancer
A. Papillary thyroid Ca
B. thyroid lymphoma

Answer: B

plural effusion (empyema result) ask about TX?

Answer: CHEST TUBE

Pancreatic pseudocyst with early satity and fullness after observation for 3 weaks it
increase in size , what is the next step?
A. observation
B. percutaneous drainage
C. Internal drainage
D. Excision

Answer: C

237
Patient with left inflamed red hemi scrotum, on examination there was red firm
irreducible painful swelling with high riding which is extended to to left inguinal
region. Left testis couldn't be palpated. What is the diagnosis?

A. Torsion appendix testis.

B. testicular torsion

C. epididymo orchitis.

D. incarcerated inguinal hernia

Answer: D

Pt 43 with 6 years Hx of UC , what’s the recommendation about colon cancer?


A. colonoscopy at age of 50 then every 10 y
B. do colonoscopy after 8-10 y from the diagnosis of UC

Answer: B

Pt with oesophageal cancer how to do staging?


A. ct
B. oesophageal US
C. CXR

Answer: A

Pt with air in biliary tree ,Dx?


A. cholecystitis
B. Cholelithiasis
C. Gall stone ilieus

Answer: C

Pt treat as croup but without benefits, what’s the investigation to know the other Dx?
A. lateral xray
B. CXR
C. Laryngoscope

Answer: A

Copd which parameter mostly use to asses prognosis !


A. -FEv1

238
B. -Fev1/ fvc

Answer: A

How to differentiate B/w hypovolemic shock and PPH


A. -Maternal pulse pressure
B. -HCT
C. -HB

Answer: A

Pregnant with hx of genital herpes simplex what to do


A. -Speculum exam
B. -Give acyclovir
C. -Cs

Answer: A?

Pregnant lonq scenario dx is written, abruptio placentae what something you will do
itto Save her life??
A. -Admit icu
B. -FFB
C. -Rapid response team with multidisciplinary intervention
D. -Immediately delivery

Answer: C

Pt asymptomatic found to have long nodule size was <3cm ?


A. -Refer to surgical resection
B. -Review previous x-ray
C. -Ct..

Answer: B

Buisness man travelling, with oral candida, normal neutrophil high


lymphocyte,diagnosis?
A. HIV
B. Toxoplasmosis- Brucellosis

Answer: A

239
Long case Patient with abdominal pain for 2 months on meds but failed to respond,
endoscopy done and revealed multiple ulcers in antrum. What is the best treatment?

A. anterctomy and vagotomy

B. Total gastrectomy

C. partial gastrectomy

D. Vagotomy with Pyloroplasty

Answer: C

A man with right forearm circumferential burning with weak pulses. Whats best next
approach?
A. -Bedside escharatomy
B. -Compartment measurement
C. -Burning dress, iv fluid, elevate the limp
D. -Iv fluid

Answer: A

Child with hypertelorism and other manifestation, what you will ordr?
A. TORSH screening
B. Gentic screening /chromosomal analysis
C. Metabolic screening

Answer: B

7 y/o with meningeal irritation, headache, and fever. CSF (normal protein and normal
glucose and lymphocytosis). How to treat?
A. Ceftriaxone
B. vancomycin
C. steroids
D. Acyclovir

Answer: D

240
Young female pregnant, unbooked, presented to the ER with vaginal bleeding,
shedenies any N/V, or abdominal pain. Fundal height exam confirmed a 32 GA ,Vitals
all normal What is the most appropriate NEXT step?

A. Perform vaginal digital assessment

B. Perform ultrasound

C. Do laboratory investigation

D. Admit the pt to the ward

Female pregnant 29 wks came to the clinic complaining of leakage (literally


writtenlike this). While taking the hx she said that leak is still ongoing with large
amount offluid. What is the next step?

A. Vaginal digital exam

B. Sterile speculum exam

C. reassurance and discharge

D. Give steroid and ask her to come after 1 week to do something (sorry I forget)

preterm baby born at 30 week gestation weight is 1.5 kg. regarding vaccination
what’strue in his case:

-at birth

-one week after discharge

-delay until corrected age

25 years old male presented with progressive SOB and wheezing. On auscultation:
wheezy chest, no crackles. History of previous hospitalization and intubation last
year. Was given systemic steroids but no benefit which of the following is the most
appropriate rescue (something like that) drug?

- IV SABA

- Inhaled budesonide

- IV Epinephrine

- IV MgSo ✅
And another one

k/c of asthma presented to ER with sever exacerbation ,*he can’t complete one
sentence*, what is management?

A- C-PAP

241
B- mechanical ventilation

C- beta agonist + corticosteroids + theophylline + oxygen

D- Mg sulfate

Pt 25y asthma , chronic sinusitis not controlled, What to add ?

-Montlkast

-Busonide

patient (I think pediatric) came with supracondylar fracture, vitally stable but has
diminished pulse , what is the most appropriate management?

A-early K wire

B-surgical exploration

Another recall

Child with supracondylar fracture and loss of pulse

What is best next step?

A. Surgical exploration.

B. Observation

C. Closer reduction

Girl 9 or 12 yrs old complaining of recurrent abdominal pain, loss of appetite, loss of
weight, associated with watery diarrhea and some time there's blood.No vitals
mentioned., What is the most appropriate management?

A. Reassurance

B. Admit for further workup

C. Prescribe PPI and follow up

D. Forgot the last option but wasn't related

Man with tibial fracture i think, managed in the ER with cast, then he developed
swelling in the toes. (Clear scenario about compartment syndrome)What is the
EARLIEST symptom?

- pain

- paresthesia

- cold extremities

242
- couldn't remember

Another recall

male patient present in the ER after RTA with tibial fracture and he was stable, what is
our priority in regard to his management?

A. control the pain

B. Control bleeding ☘️
C.Limit soft tissue injury

LONG scenario about anterolateral STEMI (with pic), pt is k/c of DM, HTN, recent hx of
ischemic stroke 1 month back. Labs and vitals were provided.Asking about the next
step in management:

A. Give him magnesium

B. CT angiography

C. Urgent coronary angiography

D. Thrombolytic therapy

Hypertensive male on thiazide, His blood pressure is 158/?What dietary modification


will help him get better control of his blood pressure?

A. Low protein diet

B. Low salt diet

C. High fiber diet

D. Low fat diet

43 yo female hx intermittent bleeding for 6 months , come for contraception what the
most appropriate investigation?

-TSH

-trans vginal us

- FSH

What is the factor related to lncrease bnb other than heart faliuar?

A. Copd

B. Obsity

A scenario about a child who has a unilateral wheeze despite bronchodilatior use at
home and it was asking about the initial investigation?

243
-bronchoscope

-Chest x ray (AP and lateral)

Child complain of chest pain for 2 hours and loud heart beats,HR is 256 Physical
examination was normal, not in pain!What’s the highest next step?

- 12 lead ECG

- ECHO

- Chest Xray

- Cardiac enzymes

Pregnant patient on valproic acid for previous seizures. What would you expect?

-Oligohydramnios

-Polyhydramnios

-Neural tube defects

-Sorry I don't remember the last option

Patient has frequent seizures and cognitive impairment. Diagnosed to have


glioblastoma multiforme. Labs show decreased Na, decreased K, decreased Cl (I
think also decreased Ca but not sure). What is the cause of the electrolyte
abnormalities?

-Salt wasting nephropathy

-Excessive water intake

-SIADH

-Forgot the last option sorry

2 weeks old girl with 3 lesion desrcibed as flat red and contain non clear fluid. 2 small
in legs, 1 big on right eyelid. Whats ur initial management?

-Consult Ophtha

-Consult Ped surgereon

-Give AB

-Observe

A patient complaining of burning chest pain and unpleasant taste when lifting heavy
objects?

244
A. Gastritis.

B. Esophagitis.

C. Perforated peptic ulcer.

Pt on antidepressant then developed constipation, what class of antidepressants?

-Ssri

-TCA

-SNRI

-MAOI

Female Pt known to have SLE on steroids, hydroxy. etc , Has hx of hip pain for 3
examination there’s diffuse tenderness and he’s vitally stable,What’s the appropriate
next step ?

-Avascular necrosis

Elderly, 89-90 yo lives in (‫( رعاية دار اتوقع‬has a abdominal pain for 24 hrs, on
examination there’s diffuse tenderness and he’s vitally stable

What’s the appropriate next step?

-abdominal US

-sigmoidoscopy

-colonoscopy

-last choice I can’t remember if it was laparotomy or something else

milestone said baba and walk holding furniture and a lot of other features

A. 12M

B. 10M

Other recall

38- Child can set without support, cruises around furniture, uses chair to stand, say
dada, crawl stairs. What is the age of this child ?

A. 8 months.

✨B. 10 months.
C. 12 months.

D. 15 months

245
A mother brought her 9-month-old girl, she says she received all of her vaccines
since birth, what she’ll get now?

-measles and meningococcal

patient underwent colonoscopy found sigmoid polyp, histology report: Sigmoid


Adenocarcinoma with free margin with no invasion to mucosa. What is the next step?

A- observe

B- sigmoidectomy

C- sigmoidoscopy

D- ???

59 years old male complaining of of fatigue palpitation and SOB, examination showed
pallor and CBC suggestive of iron deficiency anemia, beside giving oral iron
supplements, what investigation should be requested? ]

- A- urea breath test

- B- Fecal occult blood

- C- Endoscopy and colonoscopy

83 y/o presented with abnormal uterine bleeding started 8 month ago, Hx of


3abortions, and Hx of myomectomy, in examination bulky uterus:

Pic: endometrium thickening 13 mm

What is the Dx:

A- endometrioma

B- uterine atrophy

C- endometrial cancer

D-? no adenomyosis was there

Milestones:
Q-3 years old~ climbs stairs
Q-And a question about a baby who can sit without support and holds on furniture to
stand, cab recognize his name
-10 month
-12 month

-A mother brought her 9 month old girl, she says she received all of her vaccines
since birth, what she’ll get now?

-measles and meningococcal

246
patient underwent colonoscopy found sigmoid polyp, histology report: Sigmoid
Adenocarcinoma with free margin with no invasion to mucosa. What is the next step
A- observe
B- sigmoidectomy
C- sigmoidoscopy

-55years old presented with fatigue and pallor? Lab: microcytic hypochromic anemia
, next step:
A- peripheral blood smear
B- focal occult blood

Q-83 y/o presented with abnormal uterine bleeding started 8 month ago, Hx of 3
abortions, and Hx of myomectomy, in examination bulky uterus:
Pic: endometrium thickening 13 mm
What is the Dx:
A- endometrioma
B- uterine atrophy
C- endometrial cancer

Q-Gun shot wound in the thigh, he has weak pulse in the affected limb and there’s
hematoma and he cant move and there’s paraesthesia, What to do ?
A- exploration in the
B- bedside exploration

Q- pregnant women presente with lower abdominal pain and moderate vaginal
bleeding K/C of BA + epilepsy + smocker What is the highest risk factor for abruption
placenta?
A-Age
B- Smocking
C- Bronchial Asthma
D-Epilepsy

Q-36 year old male after road traffic accident presented with chest ecchymosis. HR
normal, BP normal, RR normal
ECG: Arrythmia “written like that no image”
Chet x-ray: Sternal fracture
What is the most likely cause?
A-Aortic disruption
B-Cardiac contusion

Q-Heart failure patient. Exertion after in house activities. He is on ACEi, BB and


Frusemide. What would you like to do?
A-add Isosorbide
B-add Spironolactone

Q-Women in her mid 40s. Complain of urinary leak. It occur during cough, excercise
and (third activity also related to stress). She also have it when she get the urge to
urinateWhat is the most likely diagnosis?
-stress incontinance
-Mixed incontinance

247
-urge incontinance

Q-Elderly have SCC. Present with altered sensorium. asking about type of fluid to
give
-NS
-Half NS -Hypertonic -1/2 Dextrose

Q-Women with history of salpingostomy for ectopic. Came for (smothing I forgot). On
PE there is a protroding mass from the cervix., Her hcg serial measurment “was
decreasing in first two days then start to increase from the third day to the seventh”
What to do NEXT:
-Immediate MTX
-Bx
-Work-up for metastsis without bx

Q- Infant with inguinal hernia, not obstructed, strangulated or irreducible, the testes
at the ipsilateral side was slightly elevated
-Herniotomy
-Hernia repair with mesh
-Wait untill 6 years

Q-Preconception with SLE on hydroxychloriquine:


-hydroxychloriquine interacts with MMF and can’t be given together
-hydroxychloriquine should be discontinued in pregnancy
-hydroxychloriquine should be discontinued in lactation
-Increase the dose of hydroxychloriquine in case of smoking

The percentage by which smoking increase the risk of MI?


A.2
B.3
C.4
D.5

Q- A scenario about hydatid cyst 10*13 with dauther cysts, what is the mangement?
-surgical deroofing
-percutaneous aspiration
-Right hepatectomy

On US thyroid is normal and the swelling has cystic component. FNA shows
complete follicular cells. What is the most likely diagnosis?
-Apparent thyroid
-Metastatic cancer
-Ectopic thyroid
-Thyroglossal cyst

Q-A scenario about pt with raised JVP and muffled heart sound, imaging shows
enlarged cardia sillhouette, where is the problem?
-Pericardium

Q-after ileal resection, which element to be delivered parentally?

248
A. Iron
B. Zinc
C. Bile salts
D. Vit B12

Q-RTA victim in ER after 20min of accident with warm extremities, clear chest, and
normal abdomen, (in the vitals, bp was 90/60 and I could not remember other values),
what is the most likely type of shock?
-Septic
-Neurogenic
-Cardiogenic
-Hypovolemic

Q-Skin disease with celiac disease?


-Psoriasis
-Dermatitis herpetifprmis

Q A child presented with acute exacerbation of asthma, he was given SABA, Inhaled
corticosteroids and meganesium sulphate, he was given 100٪ oxygen but the the
oxygen saturation stills 80. What is the most important next step ?
1- add ipratropium
2-add aminophylline
3- add antibiotic ( not sure)
4- intubation and mechanical ventillation

Q-A Pregnant at 38 weeks gestation presented with 24 hours spontaneous rupture of


membrane. She has no pain and on examination the uterus is soft and no evidence of
contractions. CTG was reassuring. What is the best next step?
1- C/S
2- observation
3- induction of labour 4- corticosteroids

Q-A 8 weaks Child Presentes with two weeks history of fever, cough and SOB. The
child has poor feeding. On neonatal examination there was evidence pansystolic
murmur. Child has bilateral infiltration on chest X-ray and crackles. What is the best
step in managing this child?
1- refer to surgery
2- give diuretics immediately
3-give full antibiotic course then reevaluate 4-give steroid( not sure )

Q-Adult Patient presented to the ER with erythema and painful swelling in the dorsum
of the hand. He mentioned that he has a small knife injury oh the last few days. What
is the most likely diagnosis?
1- cellulitis
2-carbuncle
3-herpiticform dermatitis
4-fungal infection( not sure)

249
QAn adult patient underwent Right hemithyroidectomy. histology showed a cancer(
not sure which type) with clear margin. On US There was one solid nodule 0.6*0.9 and
another nodule 2*3 size. What the best next step?
A- follow up
B- FNA from the larger one
C- FNA From both
D- not remembering

Q-30 years old woman has 3 months history of joint pain in both hands, On
examination there is evidence of active inflammation ( case of rheumatoid arthritis)
what is the best management?
A- steroids( prednisone)
B- indomethacin
C- hydroxychloride and weekly methotrexate D- not remembered

Q-A clear case of UTI with positive nitrites And Leukocytes. Which one of the
following will increase the probability of uti?
A- nitrites
B- leukocytes

Q-Patient involved in road traffic accident Had multiple liver lacerations and was
haemodynamically unstable. What is the pest management?
A- hepatic packing
B- isolated artery ligation
C- partial heptectomy

Q- Child has scrotal pain , on examination ( on previous recalls was written on


exploration which makes different for me )found cord is edematous and inflamed with
red right hemiscrtum, ( slightly elevated testes not sure)
A) testicular torsion
B) Inguinal hernia
C) testicular appendages torsion
D) epidiymo-orchitia

250
PICTURES

1-

Exactly the seem picture in the exam. Asking for diagnosis?


Answer: Henoch-Schonlein Purpura (HSP)

251
Picture of hysterscope ? Endometrial polyps

Come for me exactly this picture and ask diagnosis


1_chancroid
B_herpes
C _syphilis

252
and they are concerned on fertility? what to do? I choose, the cause of
fertility

5. Picture about warts at vulva diagnosis: condyloma lata

100-CTG pic early deceleration

253
101-ECG of right bundle branch block?

254
extraintestinal manifestations of crohn's disease:

I couldn’t remember the scenario but she was preterm + with ctg showing reduced
variability. what to do:
- reassure and discharge

255
- c/s

Boot shaped (Tetrology of Fallout,ToF) they asked about Management?

Asking about the Management.

256

You might also like